Вы находитесь на странице: 1из 290

This is page i

Printer: Opaque this


Contents
CHAPTER 1
Multilinear algebra
1.1 Background . . . . . . . . . . . . . . . . . . . . . . . . . . . 1
1.2 Quotient spaces and dual spaces . . . . . . . . . . . . . . . 4
1.3 Tensors . . . . . . . . . . . . . . . . . . . . . . . . . . . . . 12
1.4 Alternating k-tensors . . . . . . . . . . . . . . . . . . . . . . 17
1.5 The space,
k
(V

) . . . . . . . . . . . . . . . . . . . . . . . 26
1.6 The wedge product . . . . . . . . . . . . . . . . . . . . . . . 31
1.7 The interior product . . . . . . . . . . . . . . . . . . . . . . 35
1.8 The pull-back operation on
k
. . . . . . . . . . . . . . . . 39
1.9 Orientations . . . . . . . . . . . . . . . . . . . . . . . . . . . 44
CHAPTER 2
Dierential forms
2.1 Vector elds and one-forms . . . . . . . . . . . . . . . . . . 49
2.2 k-forms . . . . . . . . . . . . . . . . . . . . . . . . . . . . . 65
2.3 Exterior dierentiation . . . . . . . . . . . . . . . . . . . . . 69
ii Contents
2.4 The interior product operation . . . . . . . . . . . . . . . . 75
2.5 The pull-back operation on forms . . . . . . . . . . . . . . . 80
2.6 Div, curl and grad . . . . . . . . . . . . . . . . . . . . . . . 88
2.7 Symplectic geometry and classical mechanics . . . . . . . . 94
CHAPTER 3
Integration of forms
3.1 Introduction . . . . . . . . . . . . . . . . . . . . . . . . . . . 105
3.2 The Poincare lemma for compactly supported forms on
rectangles . . . . . . . . . . . . . . . . . . . . . . . . . . . . 106
3.3 The Poincare lemma for compactly supported forms on
open subsets of R
n
. . . . . . . . . . . . . . . . . . . . . . . 112
3.4 The degree of a dierentiable mapping . . . . . . . . . . . . 114
3.5 The change of variables formula . . . . . . . . . . . . . . . . 119
3.6 Techniques for computing the degree of a mapping . . . . . 127
3.7 Appendix: Sards theorem . . . . . . . . . . . . . . . . . . . 137
CHAPTER 4
Forms on Manifolds
4.1 Manifolds . . . . . . . . . . . . . . . . . . . . . . . . . . . . 143
4.2 Tangent spaces . . . . . . . . . . . . . . . . . . . . . . . . . 154
4.3 Vector elds and dierential forms on manifolds . . . . . . . 162
4.4 Orientations . . . . . . . . . . . . . . . . . . . . . . . . . . . 173
4.5 Integration of forms over manifolds . . . . . . . . . . . . . . 186
4.6 Stokes theorem and the divergence theorem . . . . . . . . . 193
4.7 Degree theory on manifolds . . . . . . . . . . . . . . . . . . 201
4.8 Applications of degree theory . . . . . . . . . . . . . . . . . 208
CHAPTER 5
Cohomology via forms
5.1 The DeRham cohomology groups of a manifold . . . . . . . 217
5.2 The MayerVictoris theorem . . . . . . . . . . . . . . . . . 231
5.3 Good covers . . . . . . . . . . . . . . . . . . . . . . . . . . . 242
5.4 Poincare duality . . . . . . . . . . . . . . . . . . . . . . . . 251
5.5 Thom classes and intersection theory . . . . . . . . . . . . . 259
Contents iii
5.6 The Lefshetz theorem . . . . . . . . . . . . . . . . . . . . . 271
5.7 The K unneth theorem . . . . . . . . . . . . . . . . . . . . . 282
CHAPTER B
The implicit function theorem
This is page 1
Printer: Opaque this
CHAPTER 1
MULTILINEAR ALGEBRA
1.1 Background
We will list below some denitions and theorems that are part of
the curriculum of a standard theory-based sophomore level course
in linear algebra. (Such a course is a prerequisite for reading these
notes.) A vector space is a set, V , the elements of which we will refer
to as vectors. It is equipped with two vector space operations:
Vector space addition. Given two vectors, v
1
and v
2
, one can add
them to get a third vector, v
1
+v
2
.
Scalar multiplication. Given a vector, v, and a real number, , one
can multiply v by to get a vector, v.
These operations satisfy a number of standard rules: associativ-
ity, commutativity, distributive laws, etc. which we assume youre
familiar with. (See exercise 1 below.) In addition well assume youre
familiar with the following denitions and theorems.
1. The zero vector. This vector has the property that for every
vector, v, v +0 = 0 +v = v and v = 0 if is the real number, zero.
2. Linear independence. A collection of vectors, v
i
, i = 1, . . . , k, is
linearly independent if the map
(1.1.1) R
k
V , (c
1
, . . . , c
k
) c
1
v
1
+ +c
k
v
k
is 1 1.
3. The spanning property. A collection of vectors, v
i
, i = 1, . . . , k,
spans V if the map (1.1.1) is onto.
4. The notion of basis. The vectors, v
i
, in items 2 and 3 are a basis
of V if they span V and are linearly independent; in other words, if
the map (1.1.1) is bijective. This means that every vector, v, can be
written uniquely as a sum
(1.1.2) v =

c
i
v
i
.
2 Chapter 1. Multilinear algebra
5. The dimension of a vector space. If V possesses a basis, v
i
,
i = 1, . . . , k, V is said to be nite dimensional, and k is, by denition,
the dimension of V . (It is a theorem that this denition is legitimate:
every basis has to have the same number of vectors.) In this chapter
all the vector spaces well encounter will be nite dimensional.
6. A subset, U, of V is a subspace if its vector space in its own
right, i.e., for v, v
1
and v
2
in U and in R, v and v
1
+v
2
are in U.
7. Let V and W be vector spaces. A map, A : V W is linear if,
for v, v
1
and v
2
in V and R
A(v) = Av (1.1.3)
and
A(v
1
+v
2
) = Av
1
+Av
2
. (1.1.4)
8. The kernel of A. This is the set of vectors, v, in V which get
mapped by A into the zero vector in W. By (1.1.3) and (1.1.4) this
set is a subspace of V . Well denote it by Ker A.
9. The image of A. By (1.1.3) and (1.1.4) the image of A, which
well denote by ImA, is a subspace of W. The following is an
important rule for keeping track of the dimensions of Ker A and
ImA.
(1.1.5) dimV = dimKer A+ dimImA.
Example 1. The map (1.1.1) is a linear map. The v
i
s span V if its
image is V and the v
i
s are linearly independent if its kernel is just
the zero vector in R
k
.
10. Linear mappings and matrices. Let v
1
, . . . , v
n
be a basis of V
and w
1
, . . . , w
m
a basis of W. Then by (1.1.2) Av
j
can be written
uniquely as a sum,
(1.1.6) Av
j
=
m

i=1
c
i,j
w
i
, c
i,j
R.
The m n matrix of real numbers, [c
i,j
], is the matrix associated
with A. Conversely, given such an m n matrix, there is a unique
linear map, A, with the property (1.1.6).
1.1 Background 3
11. An inner product on a vector space is a map
B : V V R
having the three properties below.
(a) For vectors, v, v
1
, v
2
and w and R
B(v
1
+v
2
, w) = B(v
1
, w) +B(v
2
, w)
and
B(v, w) = B(v, w) .
(b) For vectors, v and w,
B(v, w) = B(w, v) .
(c) For every vector, v
B(v, v) 0 .
Moreover, if v = 0, B(v, v) is positive.
Notice that by property (b), property (a) is equivalent to
B(w, v) = B(w, v)
and
B(w, v
1
+v
2
) = B(w, v
1
) +B(w, v
2
) .
The items on the list above are just a few of the topics in linear al-
gebra that were assuming our readers are familiar with. Weve high-
lighted them because theyre easy to state. However, understanding
them requires a heavy dollop of that indenable quality mathe-
matical sophistication, a quality which will be in heavy demand in
the next few sections of this chapter. We will also assume that our
readers are familiar with a number of more low-brow linear algebra
notions: matrix multiplication, row and column operations on matri-
ces, transposes of matrices, determinants of n n matrices, inverses
of matrices, Cramers rule, recipes for solving systems of linear equa-
tions, etc. (See 1.1 and 1.2 of Munkres book for a quick review of
this material.)
4 Chapter 1. Multilinear algebra
Exercises.
1. Our basic example of a vector space in this course is R
n
equipped
with the vector addition operation
(a
1
, . . . , a
n
) + (b
1
, . . . , b
n
) = (a
1
+b
1
, . . . , a
n
+b
n
)
and the scalar multiplication operation
(a
1
, . . . , a
n
) = (a
1
, . . . , a
n
) .
Check that these operations satisfy the axioms below.
(a) Commutativity: v +w = w +v.
(b) Associativity: u + (v +w) = (u +v) +w.
(c) For the zero vector, 0 = (0, . . . , 0), v + 0 = 0 +v.
(d) v + (1)v = 0.
(e) 1v = v.
(f) Associative law for scalar multiplication: (ab)v = a(bv).
(g) Distributive law for scalar addition: (a +b)v = av +bv.
(h) Distributive law for vector addition: a(v +w) = av +aw.
2. Check that the standard basis vectors of R
n
: e
1
= (1, 0, . . . , 0),
e
2
= (0, 1, 0, . . . , 0), etc. are a basis.
3. Check that the standard inner product on R
n
B((a
1
, . . . , a
n
), (b
1
, . . . , b
n
)) =
n

i=1
a
i
b
i
is an inner product.
1.2 Quotient spaces and dual spaces
In this section we will discuss a couple of items which are frequently,
but not always, covered in linear algebra courses, but which well
need for our treatment of multilinear algebra in 1.1.3 1.1.8.
1.2 Quotient spaces and dual spaces 5
The quotient spaces of a vector space
Let V be a vector space and W a vector subspace of V . A W-coset
is a set of the form
v +W = {v +w, w W} .
It is easy to check that if v
1
v
2
W, the cosets, v
1
+ W and
v
2
+ W, coincide while if v
1
v
2
W, they are disjoint. Thus the
W-cosets decompose V into a disjoint collection of subsets of V . We
will denote this collection of sets by V/W.
One denes a vector addition operation on V/W by dening the
sum of two cosets, v
1
+W and v
2
+W to be the coset
(1.2.1) v
1
+v
2
+W
and one denes a scalar multiplication operation by dening the
scalar multiple of v +W by to be the coset
(1.2.2) v +W .
It is easy to see that these operations are well dened. For instance,
suppose v
1
+ W = v

1
+ W and v
2
+ W = v

2
+ W. Then v
1
v

1
and v
2
v

2
are in W; so (v
1
+ v
2
) (v

1
+ v

2
) is in W and hence
v
1
+v
2
+W = v

1
+v

2
+W.
These operations make V/W into a vector space, and one calls
this space the quotient space of V by W.
We dene a mapping
(1.2.3) : V V/W
by setting (v) = v + W. Its clear from (1.2.1) and (1.2.2) that
is a linear mapping, and that it maps V to V/W. Moreover, for
every coset, v +W, (v) = v +W; so the mapping, , is onto. Also
note that the zero vector in the vector space, V/W, is the zero coset,
0+W = W. Hence v is in the kernel of if v +W = W, i.e., v W.
In other words the kernel of is W.
In the denition above, V and W dont have to be nite dimen-
sional, but if they are, then
(1.2.4) dimV/W = dimV dimW .
by (1.1.5).
The following, which is easy to prove, well leave as an exercise.
6 Chapter 1. Multilinear algebra
Proposition 1.2.1. Let U be a vector space and A : V U a linear
map. If W Ker A there exists a unique linear map, A
#
: V/W U
with property, A = A
#
.
The dual space of a vector space
Well denote by V

the set of all linear functions, : V R. If
1
and
2
are linear functions, their sum,
1
+
2
, is linear, and if is
a linear function and is a real number, the function, , is linear.
Hence V

is a vector space. One calls this space the dual space of V .


Suppose V is n-dimensional, and let e
1
, . . . , e
n
be a basis of V .
Then every vector, v V , can be written uniquely as a sum
v = c
1
e
1
+ +c
n
e
n
c
i
R.
Let
(1.2.5) e

i
(v) = c
i
.
If v = c
1
e
1
+ + c
n
e
n
and v

= c

1
e
1
+ + c

n
e
n
then v + v

=
(c
1
+c

1
)e
1
+ + (c
n
+c

n
)e
n
, so
e

i
(v +v

) = c
i
+c

i
= e

i
(v) +e

i
(v

) .
This shows that e

i
(v) is a linear function of v and hence e

i
V

.
Claim: e

i
, i = 1, . . . , n is a basis of V

.
Proof. First of all note that by (1.2.5)
(1.2.6) e

i
(e
j
) =
_
1 , i = j
0 , i = j
.
If V

let
i
= (e
i
) and let

i
e

i
. Then by (1.2.6)
(1.2.7)

(e
j
) =

i
e

i
(e
j
) =
j
= (e
j
) ,
i.e., and

take identical values on the basis vectors, e


j
. Hence
=

.
Suppose next that

i
e

i
= 0. Then by (1.2.6),
j
= (

i
e

i
)(e
j
) =
0 for all j = 1, . . . , n. Hence the e

j
s are linearly independent.
1.2 Quotient spaces and dual spaces 7
Let V and W be vector spaces and A : V W, a linear map.
Given W

the composition, A, of A with the linear map,


: W R, is linear, and hence is an element of V

. We will denote
this element by A

, and we will denote by


A

: W

the map, A

. Its clear from the denition that


A

(
1
+
2
) = A

1
+A

2
and that
A

= A

,
i.e., that A

is linear.
Denition. A

is the transpose of the mapping A.


We will conclude this section by giving a matrix description of
A

. Let e
1
, . . . , e
n
be a basis of V and f
1
, . . . , f
m
a basis of W; let
e

1
, . . . , e

n
and f

1
, . . . , f

m
be the dual bases of V

and W

. Suppose A
is dened in terms of e
1
, . . . , e
n
and f
1
, . . . , f
m
by the mn matrix,
[a
i,j
], i.e., suppose
Ae
j
=

a
i,j
f
i
.
Claim. A

is dened, in terms of f

1
, . . . , f

m
and e

1
, . . . , e

n
by the
transpose matrix, [a
j,i
].
Proof. Let
A

i
=

c
j,i
e

j
.
Then
A

i
(e
j
) =

k
c
k,i
e

k
(e
j
) = c
j,i
by (1.2.6). On the other hand
A

i
(e
j
) = f

i
(Ae
j
) = f

i
_

a
k,j
f
k
_
=

k
a
k,j
f

i
(f
k
) = a
i,j
so a
i,j
= c
j,i
.
8 Chapter 1. Multilinear algebra
Exercises.
1. Let V be an n-dimensional vector space and W a k-dimensional
subspace. Show that there exists a basis, e
1
, . . . , e
n
of V with the
property that e
1
, . . . , e
k
is a basis of W. Hint: Induction on n k.
To start the induction suppose that nk = 1. Let e
1
, . . . , e
n1
be a
basis of W and e
n
any vector in V W.
2. In exercise 1 show that the vectors f
i
= (e
k+i
), i = 1, . . . , nk
are a basis of V/W.
3. In exercise 1 let U be the linear span of the vectors, e
k+i
, i =
1, . . . , n k.
Show that the map
U V/W , u (u) ,
is a vector space isomorphism, i.e., show that it maps U bijectively
onto V/W.
4. Let U, V and W be vector spaces and let A : V W and
B : U V be linear mappings. Show that (AB)

= B

.
5. Let V = R
2
and let W be the x
1
-axis, i.e., the one-dimensional
subspace
{(x
1
, 0) ; x
1
R}
of R
2
.
(a) Show that the W-cosets are the lines, x
2
= a, parallel to the
x
1
-axis.
(b) Show that the sum of the cosets, x
2
= a and x
2
= b is the
coset x
2
= a +b.
(c) Show that the scalar multiple of the coset, x
2
= c by the
number, , is the coset, x
2
= c.
6. (a) Let (V

)

be the dual of the vector space, V



. For every
v V , let
v
: V

R be the function,
v
() = (v). Show that
the
v
is a linear function on V

, i.e., an element of (V

)

, and show
that the map
(1.2.8) : V (V

)

v
v
is a linear map of V into (V

)

.
1.2 Quotient spaces and dual spaces 9
(b) Show that the map (1.2.8) is bijective. (Hint: dim(V

)

=
dimV

= dimV , so by (1.1.5) it suces to show that (1.2.8) is
injective.) Conclude that there is a natural identication of V with
(V

, i.e., that V and (V



)

are two descriptions of the same object.


7. Let W be a vector subspace of V and let
W

= { V

, (w) = 0 if w W} .
Show that W

is a subspace of V

and that its dimension is equal to
dimV dimW. (Hint: By exercise 1 we can choose a basis, e
1
, . . . , e
n
of V such that e
1
, . . . e
k
is a basis of W. Show that e

k+1
, . . . , e

n
is a
basis of W

.) W

is called the annihilator of W in V



.
8. Let V and V

be vector spaces and A : V V

a linear map.
Show that if W is the kernel of A there exists a linear map, B :
V/W V

, with the property: A = B , being the map (1.2.3).


In addition show that this linear map is injective.
9. Let W be a subspace of a nite-dimensional vector space, V .
From the inclusion map, : W

, one gets a transpose map,

: (V

)

(W

and, by composing this with (1.2.8), a map

: V (W

.
Show that this map is onto and that its kernel is W. Conclude from
exercise 8 that there is a natural bijective linear map
: V/W (W

with the property =

. In other words, V/W and (W

are
two descriptions of the same object. (This shows that the quotient
space operation and the dual space operation are closely related.)
10. Let V
1
and V
2
be vector spaces and A : V
1
V
2
a linear map.
Verify that for the transpose map: A

: V

2
V

1
Ker A

= (ImA)

and
ImA

= (Ker A)

.
10 Chapter 1. Multilinear algebra
11. (a) Let B : V V R be an inner product on V . For v V
let

v
: V R
be the function:
v
(w) = B(v, w). Show that
v
is linear and show
that the map
(1.2.9) L : V V

, v
v
is a linear mapping.
(b) Prove that this mapping is bijective. (Hint: Since dimV =
dimV

it suces by (1.1.5) to show that its kernel is zero. Now note


that if v = 0
v
(v) = B(v, v) is a positive number.) Conclude that if
V has an inner product one gets from it a natural identication of
V with V

.
12. Let V be an n-dimensional vector space and B : V V R
an inner product on V . A basis, e
1
, . . . , e
n
of V is orthonormal is
(1.2.10) B(e
i
, e
j
) =
_
1 i = j
0 i = j
(a) Show that an orthonormal basis exists. Hint: By induction let
e
i
, i = 1, . . . , k be vectors with the property (1.2.10) and let v be a
vector which is not a linear combination of these vectors. Show that
the vector
w = v

B(e
i
, v)e
i
is non-zero and is orthogonal to the e
i
s. Now let e
k+1
= w, where
= B(w, w)

1
2
.
(b) Let e
1
, . . . e
n
and e

1
, . . . e

n
be two orthogonal bases of V and let
(1.2.11) e

j
=

a
i,j
e
i
.
Show that
(1.2.12)

a
i,j
a
i,k
=
_
1 j = k
0 j = k
(c) Let A be the matrix [a
i,j
]. Show that (1.2.12) can be written
more compactly as the matrix identity
(1.2.13) AA
t
= I
where I is the identity matrix.
1.2 Quotient spaces and dual spaces 11
(d) Let e
1
, . . . , e
n
be an orthonormal basis of V and e

1
, . . . , e

n
the
dual basis of V

. Show that the mapping (1.2.9) is the mapping,
Le
i
= e

i
, i = 1, . . . n.
12 Chapter 1. Multilinear algebra
1.3 Tensors
Let V be an n-dimensional vector space and let V
k
be the set of all
k-tuples, (v
1
, . . . , v
k
), v
i
V . A function
T : V
k
R
is said to be linear in its i
th
variable if, when we x vectors, v
1
, . . . , v
i1
,
v
i+1
, . . . , v
k
, the map
(1.3.1) v V T(v
1
, . . . , v
i1
, v, v
i+1
, . . . , v
k
)
is linear in V . If T is linear in its i
th
variable for i = 1, . . . , k it is said
to be k-linear, or alternatively is said to be a k-tensor. We denote
the set of all k-tensors by L
k
(V ). We will agree that 0-tensors are
just the real numbers, that is L
0
(V ) = R.
Let T
1
and T
2
be functions on V
k
. It is clear from (1.3.1) that if
T
1
and T
2
are k-linear, so is T
1
+T
2
. Similarly if T is k-linear and
is a real number, T is k-linear. Hence L
k
(V ) is a vector space. Note
that for k = 1, k-linear just means linear, so L
1
(V ) = V

.
Let I = (i
1
, . . . i
k
) be a sequence of integers with 1 i
r
n,
r = 1, . . . , k. We will call such a sequence a multi-index of length k.
For instance the multi-indices of length 2 are the square arrays of
pairs of integers
(i, j) , 1 i, j n
and there are exactly n
2
of them.
Exercise.
Show that there are exactly n
k
multi-indices of length k.
Now x a basis, e
1
, . . . , e
n
, of V and for T L
k
(V ) let
(1.3.2) T
I
= T(e
i
1
, . . . , e
i
k
)
for every multi-index I of length k.
Proposition 1.3.1. The T
I
s determine T, i.e., if T and T

are
k-tensors and T
I
= T

I
for all I, then T = T

.
1.3 Tensors 13
Proof. By induction on n. For n = 1 we proved this result in 1.1.
Lets prove that if this assertion is true for n1, its true for n. For
each e
i
let T
i
be the (k 1)-tensor
(v
1
, . . . , v
n1
) T(v
1
, . . . , v
n1
, e
i
) .
Then for v = c
1
e
1
+ c
n
e
n
T(v
1
, . . . , v
n1
, v) =

c
i
T
i
(v
1
, . . . , v
n1
) ,
so the T
i
s determine T. Now apply induction.
The tensor product operation
If T
1
is a k-tensor and T
2
is an -tensor, one can dene a k+-tensor,
T
1
T
2
, by setting
(T
1
T
2
)(v
1
, . . . , v
k+
) = T
1
(v
1
, . . . , v
k
)T
2
(v
k+1
, . . . , v
k+
) .
This tensor is called the tensor product of T
1
and T
2
. We note that
if T
1
or T
2
is a 0-tensor, i.e., scalar, then tensor product with it
is just scalar multiplication by it, that is a T = T a = aT
(a R, T L
k
(V )).
Similarly, given a k-tensor, T
1
, an -tensor, T
2
and an m-tensor,
T
3
, one can dene a (k + +m)-tensor, T
1
T
2
T
3
by setting
T
1
T
2
T
3
(v
1
, . . . , v
k++m
) (1.3.3)
= T
1
(v
1
, . . . , v
k
)T
2
(v
k+1
, . . . , v
k+
)T
3
(v
k++1
, . . . , v
k++m
) .
Alternatively, one can dene (1.3.3) by dening it to be the tensor
product of T
1
T
2
and T
3
or the tensor product of T
1
and T
2
T
3
.
Its easy to see that both these tensor products are identical with
(1.3.3):
(1.3.4) (T
1
T
2
) T
3
= T
1
(T
2
T
3
) = T
1
T
2
T
3
.
We leave for you to check that if is a real number
(1.3.5) (T
1
T
2
) = (T
1
) T
2
= T
1
(T
2
)
and that the left and right distributive laws are valid: For k
1
= k
2
,
(1.3.6) (T
1
+T
2
) T
3
= T
1
T
3
+T
2
T
3
14 Chapter 1. Multilinear algebra
and for k
2
= k
3
(1.3.7) T
1
(T
2
+T
3
) = T
1
T
2
+T
1
T
3
.
A particularly interesting tensor product is the following. For i =
1, . . . , k let
i
V

and let
(1.3.8) T =
1

k
.
Thus, by denition,
(1.3.9) T(v
1
, . . . , v
k
) =
1
(v
1
) . . .
k
(v
k
) .
A tensor of the form (1.3.9) is called a decomposable k-tensor. These
tensors, as we will see, play an important role in what follows. In
particular, let e
1
, . . . , e
n
be a basis of V and e

1
, . . . , e

n
the dual basis
of V

. For every multi-index, I, of length k let


e

I
= e

i
1
e

i
k
.
Then if J is another multi-index of length k,
e

I
(e
j
1
, . . . , e
j
k
) =
_
1 , I = J
0 , I = J
(1.3.10)
by (1.2.6), (1.3.8) and (1.3.9). From (1.3.10) its easy to conclude
Theorem 1.3.2. The e

I
s are a basis of L
k
(V ).
Proof. Given T L
k
(V ), let
T

T
I
e

I
where the T
I
s are dened by (1.3.2). Then
(1.3.11) T

(e
j
1
, . . . , e
j
k
) =

T
I
e

I
(e
j
1
, . . . , e
j
k
) = T
J
by (1.3.10); however, by Proposition 1.3.1 the T
J
s determine T, so
T

= T. This proves that the e

I
s are a spanning set of vectors for
L
k
(V ). To prove theyre a basis, suppose

C
I
e

I
= 0
for constants, C
I
R. Then by (1.3.11) with T

= 0, C
J
= 0, so the
e

I
s are linearly independent.
As we noted above there are exactly n
k
multi-indices of length k
and hence n
k
basis vectors in the set, {e

I
}, so weve proved
Corollary. dimL
k
(V ) = n
k
.
1.3 Tensors 15
The pull-back operation
Let V and W be nite dimensional vector spaces and let A : V W
be a linear mapping. If T L
k
(W), we dene
A

T : V
k
R
to be the function
(1.3.12) A

T(v
1
, . . . , v
k
) = T(Av
1
, . . . , Av
k
) .
Its clear from the linearity of A that this function is linear in its
i
th
variable for all i, and hence is k-tensor. We will call A

T the
pull-back of T by the map, A.
Proposition 1.3.3. The map
(1.3.13) A

: L
k
(W) L
k
(V ) , T A

T ,
is a linear mapping.
We leave this as an exercise. We also leave as an exercise the
identity
(1.3.14) A

(T
1
T
2
) = A

T
1
A

T
2
for T
1
L
k
(W) and T
2
L
m
(W). Also, if U is a vector space and
B : U V a linear mapping, we leave for you to check that
(1.3.15) (AB)

T = B

(A

T)
for all T L
k
(W).
Exercises.
1. Verify that there are exactly n
k
multi-indices of length k.
2. Prove Proposition 1.3.3.
3. Verify (1.3.14).
4. Verify (1.3.15).
16 Chapter 1. Multilinear algebra
5. Let A : V W be a linear map. Show that if
i
, i = 1, . . . , k
are elements of W

(
1

k
) = A

1
A

k
.
Conclude that A

maps decomposable k-tensors to decomposable


k-tensors.
6. Let V be an n-dimensional vector space and
i
, i = 1, 2, ele-
ments of V

. Show that
1

2
=
2

1
if and only if
1
and
2
are linearly dependent. (Hint: Show that if
1
and
2
are linearly
independent there exist vectors, v
i
, i =, 1, 2 in V with property

i
(v
j
) =
_
1, i = j
0, i = j
.
Now compare (
1

2
)(v
1
, v
2
) and (
2

1
)(v
1
, v
2
).) Conclude that if
dimV 2 the tensor product operation isnt commutative, i.e., its
usually not true that
1

2
=
2

1
.
7. Let T be a k-tensor and v a vector. Dene T
v
: V
k1
R to
be the map
(1.3.16) T
v
(v
1
, . . . , v
k1
) = T(v, v
1
, . . . , v
k1
) .
Show that T
v
is a (k 1)-tensor.
8. Show that if T
1
is an r-tensor and T
2
is an s-tensor, then if
r > 0,
(T
1
T
2
)
v
= (T
1
)
v
T
2
.
9. Let A : V W be a linear map mapping v V to w W.
Show that for T L
k
(W), A

(T
w
) = (A

T)
v
.
1.4 Alternating k-tensors 17
1.4 Alternating k-tensors
We will discuss in this section a class of k-tensors which play an
important role in multivariable calculus. In this discussion we will
need some standard facts about the permutation group. For those
of you who are already familiar with this object (and I suspect most
of you are) you can regard the paragraph below as a chance to re-
familiarize yourselves with these facts.
Permutations
Let

k
be the k-element set: {1, 2, . . . , k}. A permutation of order k
is a bijective map, :

k


k
. Given two permutations,
1
and

2
, their product,
1

2
, is the composition of
1
and
2
, i.e., the map,
i
1
(
2
(i)) ,
and for every permutation, , one denotes by
1
the inverse per-
mutation:
(i) = j
1
(j) = i .
Let S
k
be the set of all permutations of order k. One calls S
k
the
permutation group of

k
or, alternatively, the symmetric group on
k letters.
Check:
There are k! elements in S
k
.
For every 1 i < j k, let =
i,j
be the permutation
(i) = j
(j) = i (1.4.1)
() = , = i, j .
is called a transposition, and if j = i +1, is called an elementary
transposition.
Theorem 1.4.1. Every permutation can be written as a product of
nite number of transpositions.
18 Chapter 1. Multilinear algebra
Proof. Induction on k: k = 2 is obvious. The induction step: k1
implies k: Given S
k
, (k) = i
ik
(k) = k. Thus
ik
is, in
eect, a permutation of

k1
. By induction,
ik
can be written as
a product of transpositions, so
=
ik
(
ik
)
can be written as a product of transpositions.
Theorem 1.4.2. Every transposition can be written as a product of
elementary transpositions.
Proof. Let =
ij
, i < j. With i xed, argue by induction on j.
Note that for j > i + 1

ij
=
j1,j

i,j1

j1,j
.
Now apply induction to
i,j1
.
Corollary. Every permutation can be written as a product of ele-
mentary transpositions.
The sign of a permutation
Let x
1
, . . . , x
k
be the coordinate functions on R
k
. For S
k
we
dene
(1.4.2) (1)

i<j
x
(i)
x
(j)
x
i
x
j
.
Notice that the numerator and denominator in this expression are
identical up to sign. Indeed, if p = (i) < (j) = q, the term, x
p
x
q
occurs once and just once in the numerator and one and just one
in the denominator; and if q = (i) > (j) = p, the term, x
p
x
q
,
occurs once and just once in the numerator and its negative, x
q
x
p
,
once and just once in the numerator. Thus
(1.4.3) (1)

= 1 .
1.4 Alternating k-tensors 19
Claim:
For , S
k
(1.4.4) (1)

= (1)

(1)

.
Proof. By denition,
(1)

i<j
x
(i)
x
(j)
x
i
x
j
.
We write the right hand side as a product of
(1.4.5)

i<j
x
(i)
x
(j)
x
i
x
j
= (1)

and
(1.4.6)

i<j
x
(i)
x
(j)
x
(i)
x
(j)
For i < j, let p = (i) and q = (j) when (i) < (j) and let p = (j)
and q = (i) when (j) < (i). Then
x
(i)
x
(j)
x
(i)
x
(j)
=
x
(p)
x
(q)
x
p
x
q
(i.e., if (i) < (j), the numerator and denominator on the right
equal the numerator and denominator on the left and, if (j) < (i)
are negatives of the numerator and denominator on the left). Thus
(1.4.6) becomes

p<q
x
(p)
x
(q)
x
p
x
q
= (1)

.
Well leave for you to check that if is a transposition, (1)

= 1
and to conclude from this:
Proposition 1.4.3. If is the product of an odd number of trans-
positions, (1)

= 1 and if is the product of an even number of


transpositions (1)

= +1.
20 Chapter 1. Multilinear algebra
Alternation
Let V be an n-dimensional vector space and T L

(v) a k-tensor.
If S
k
, let T

(V ) be the k-tensor
(1.4.7) T

(v
1
, . . . , v
k
) = T(v

1
(1)
, . . . , v

1
(k)
) .
Proposition 1.4.4. 1. If T =
1

k
,
i
V

, then T

(1)

(k)
.
2. The map, T L
k
(V ) T

L
k
(V ) is a linear map.
3. T

= (T

.
Proof. To prove 1, we note that by (1.4.7)
(
1

k
)

(v
1
, . . . , v
k
)
=
1
(v

1
(1)
)
k
(v

1
(k)
) .
Setting
1
(i) = q, the i
th
term in this product is
(q)
(v
q
); so the
product can be rewritten as

(1)
(v
1
) . . .
(k)
(v
k
)
or
(
(1)

(k)
)(v
1
, . . . , v
k
) .
The proof of 2 well leave as an exercise.
Proof of 3: By item 2, it suces to check 3 for decomposable
tensors. However, by 1
(
1

k
)

=
(1)

(k)
= (
(1)

(k)
)

= ((
1
)

.
Denition 1.4.5. T L
k
(V ) is alternating if T

= (1)

T for all
S
k
.
We will denote by A
k
(V ) the set of all alternating k-tensors in
L
k
(V ). By item 2 of Proposition 1.4.4 this set is a vector subspace
of L
k
(V ).
1.4 Alternating k-tensors 21
It is not easy to write down simple examples of alternating k-
tensors; however, there is a method, called the alternation operation,
for constructing such tensors: Given T L

(V ) let
(1.4.8) Alt T =

S
k
(1)

.
We claim
Proposition 1.4.6. For T L
k
(V ) and S
k
,
1. (Alt T)

= (1)

Alt T
2. if T A
k
(V ) , Alt T = k!T.
3. Alt T

= (Alt T)

4. the map
Alt : L
k
(V ) L
k
(V ) , T Alt (T)
is linear.
Proof. To prove 1 we note that by Proposition (1.4.4):
(Alt T)

(1)

(T

)
= (1)

(1)

.
But as runs over S
k
, runs over S
k
, and hence the right hand
side is (1)

Alt (T).
Proof of 2. If T A
k
Alt T =

(1)

(1)

(1)

T
= k! T .
Proof of 3.
Alt T

(1)

= (1)

(1)

= (1)

Alt T = (Alt T)

.
22 Chapter 1. Multilinear algebra
Finally, item 4 is an easy corollary of item 2 of Proposition 1.4.4.
We will use this alternation operation to construct a basis for
A
k
(V ). First, however, we require some notation:
Let I = (i
1
, . . . , i
k
) be a multi-index of length k.
Denition 1.4.7. 1. I is repeating if i
r
= i
s
for some r = s.
2. I is strictly increasing if i
1
< i
2
< < i
r
.
3. For S
k
, I

= (i
(1)
, . . . , i
(k)
) .
Remark: If I is non-repeating there is a unique S
k
so that I

is strictly increasing.
Let e
1
, . . . , e
n
be a basis of V and let
e

I
= e

i
1
e

i
k
and

I
= Alt (e

I
) .
Proposition 1.4.8. 1.
I
= (1)

I
.
2. If I is repeating,
I
= 0.
3. If I and J are strictly increasing,

I
(e
j
1
, . . . , e
j
k
) =
_
1 I = J
0 I = J
.
Proof. To prove 1 we note that (e

I
)

= e

I
; so
Alt (e

I
) = Alt (e

I
)

= (1)

Alt (e

I
) .
Proof of 2: Suppose I = (i
1
, . . . , i
k
) with i
r
= i
s
for r = s. Then if
=
ir,is
, e

I
= e

I
r so

I
=
I
r = (1)

I
=
I
.
1.4 Alternating k-tensors 23
Proof of 3: By denition

I
(e
j
1
, . . . , e
j
k
) =

(1)

I
(e
j
1
, . . . , e
j
k
) .
But by (1.3.10)
e

I
(e
j
1
, . . . , e
j
k
) =
_
1 if I

= J
0 if I

= J
. (1.4.9)
Thus if I and J are strictly increasing, I

is strictly increasing if and


only if I

= I, and (1.4.9) is non-zero if and only if I = J.


Now let T be in A
k
. By Proposition 1.3.2,
T =

a
J
e

J
, a
J
R.
Since
k!T = Alt (T)
T =
1
k!

a
J
Alt (e

J
) =

b
J

J
.
We can discard all repeating terms in this sum since they are zero;
and for every non-repeating term, J, we can write J = I

, where I
is strictly increasing, and hence
J
= (1)

I
.
Conclusion:
We can write T as a sum
(1.4.10) T =

c
I

I
,
with Is strictly increasing.
Claim.
The c
I
s are unique.
24 Chapter 1. Multilinear algebra
Proof. For J strictly increasing
(1.4.11) T(e
j
1
, . . . , e
j
k
) =

c
I

I
(e
j
1
, . . . , e
j
k
) = c
J
.
By (1.4.10) the
I
s, I strictly increasing, are a spanning set of vec-
tors for A
k
(V ), and by (1.4.11) they are linearly independent, so
weve proved
Proposition 1.4.9. The alternating tensors,
I
, I strictly increas-
ing, are a basis for A
k
(V ).
Thus dimA
k
(V ) is equal to the number of strictly increasing multi-
indices, I, of length k. We leave for you as an exercise to show that
this number is equal to
(1.4.12)
_
n
k
_
=
n!
(n k)!k!
= n choose k
if 1 k n.
Hint: Show that every strictly increasing multi-index of length k
determines a k element subset of {1, . . . , n} and vice-versa.
Note also that if k > n every multi-index
I = (i
1
, . . . , i
k
)
of length k has to be repeating: i
r
= i
s
for some r = s since the i
p
s
lie on the interval 1 i n. Thus by Proposition 1.4.6

I
= 0
for all multi-indices of length k > 0 and
(1.4.13) A
k
= {0} .
Exercises.
1. Show that there are exactly k! permutations of order k. Hint: In-
duction on k: Let S
k
, and let (k) = i, 1 i k. Show that

ik
leaves k xed and hence is, in eect, a permutation of

k1
.
2. Prove that if S
k
is a transposition, (1)

= 1 and deduce
from this Proposition 1.4.3.
1.4 Alternating k-tensors 25
3. Prove assertion 2 in Proposition 1.4.4.
4. Prove that dimA
k
(V ) is given by (1.4.12).
5. Verify that for i < j 1

i,j
=
j1,j

i,j1
,
j1,j
.
6. For k = 3 show that every one of the six elements of S
3
is either
a transposition or can be written as a product of two transpositions.
7. Let S
k
be the cyclic permutation
(i) = i + 1 , i = 1, . . . , k 1
and (k) = 1. Show explicitly how to write as a product of trans-
positions and compute (1)

. Hint: Same hint as in exercise 1.


8. In exercise 7 of Section 3 show that if T is in A
k
, T
v
is in A
k1
.
Show in addition that for v, w V and T A
k
, (T
v
)
w
= (T
w
)
v
.
9. Let A : V W be a linear mapping. Show that if T is in
A
k
(W), A

T is in A
k
(V ).
10. In exercise 9 show that if T is in L
k
(W), Alt (A

T) = A

(Alt (T)),
i.e., show that the Alt operation commutes with the pull-back op-
eration.
26 Chapter 1. Multilinear algebra
1.5 The space,
k
(V

)
In 1.4 we showed that the image of the alternation operation, Alt :
L
k
(V ) L
k
(V ) is A
k
(V ). In this section we will compute the kernel
of Alt .
Denition 1.5.1. A decomposable k-tensor
1

k
,
i
V

,
is redundant if for some index, i,
i
=
i+1
.
Let I
k
be the linear span of the set of reductant k-tensors.
Note that for k = 1 the notion of redundant doesnt really make
sense; a single vector L
1
(V

) cant be redundant so we decree


I
1
(V ) = {0} .
Proposition 1.5.2. If T I
k
, Alt (T) = 0.
Proof. Let T =
k

k
with
i
=
i+1
. Then if =
i,i+1
, T

= T
and (1)

= 1. Hence Alt (T) = Alt (T

) = Alt (T)

= Alt (T);
so Alt (T) = 0.
To simplify notation lets abbreviate L
k
(V ), A
k
(V ) and I
k
(V ) to
L
k
, A
k
and I
k
.
Proposition 1.5.3. If T I
r
and T

L
s
then T T

and T

T
are in I
r+s
.
Proof. We can assume that T and T

are decomposable, i.e., T =

1

r
and T

s
and that T is redundant:
i
=
i+1
.
Then
T T

=
1

i1

i

r

s
is redundant and hence in I
r+s
. The argument for T

T is similar.
Proposition 1.5.4. If T L
k
and S
k
, then
(1.5.1) T

= (1)

T +S
where S is in I
k
.
1.5 The space,
k
(V

) 27
Proof. We can assume T is decomposable, i.e., T =
1

k
.
Lets rst look at the simplest possible case: k = 2 and =
1,2
.
Then
T

()

T =
1

2
+
2

1
= ((
1
+
2
) (
1
+
2
)
1

2
)/2 ,
and the terms on the right are redundant, and hence in I
2
. Next
let k be arbitrary and =
i,i+1
. If T
1
=
1

i2
and T
2
=

i+2

k
. Then
T (1)

T = T
1
(
i

i+1
+
i+1

i
) T
2
is in I
k
by Proposition 1.5.3 and the computation above.
The general case: By Theorem 1.4.2, can be written as a product
of m elementary transpositions, and well prove (1.5.1) by induction
on m.
Weve just dealt with the case m = 1.
The induction step: m1 implies m. Let = where is a
product of m 1 elementary transpositions and is an elementary
transposition. Then
T

= (T

= (1)

+
= (1)

(1)

T +
= (1)

T +
where the dots are elements of I
k
, and the induction hypothesis
was used in line 2.
Corollary. If T L
k
, the
(1.5.2) Alt (T) = k!T +W ,
where W is in I
k
.
Proof. By denition Alt (T) =

(1)

, and by Proposition 1.5.4,


T

= (1)

T +W

, with W

I
k
. Thus
Alt (T) =

(1)

(1)

T +

(1)

= k!T +W
where W =

(1)

.
28 Chapter 1. Multilinear algebra
Corollary. I
k
is the kernel of Alt .
Proof. Weve already proved that if T I
k
, Alt (T) = 0. To prove
the converse assertion we note that if Alt (T) = 0, then by (1.5.2)
T =
1
k!
W .
with W I
k
.
Putting these results together we conclude:
Theorem 1.5.5. Every element, T, of L
k
can be written uniquely
as a sum, T = T
1
+T
2
where T
1
A
k
and T
2
I
k
.
Proof. By (1.5.2), T = T
1
+T
2
with
T
1
=
1
k!
Alt (T)
and
T
2
=
1
k!
W .
To prove that this decomposition is unique, suppose T
1
+ T
2
= 0,
with T
1
A
k
and T
2
I
k
. Then
0 = Alt (T
1
+T
2
) = k!T
1
so T
1
= 0, and hence T
2
= 0.
Let
(1.5.3)
k
(V

) = L
k
(V

)/I
k
(V

) ,
i.e., let
k
=
k
(V

) be the quotient of the vector space L
k
by the
subspace, I
k
, of L
k
. By (1.2.3) one has a linear map:
(1.5.4) : L
k

k
, T T +I
k
which is onto and has I
k
as kernel. We claim:
Theorem 1.5.6. The map, , maps A
k
bijectively onto
k
.
Proof. By Theorem 1.5.5 every I
k
coset, T +I
k
, contains a unique
element, T
1
, of A
k
. Hence for every element of
k
there is a unique
element of A
k
which gets mapped onto it by .
1.5 The space,
k
(V

) 29
Remark. Since
k
and A
k
are isomorphic as vector spaces many
treatments of multilinear algebra avoid mentioning
k
, reasoning
that A
k
is a perfectly good substitute for it and that one should,
if possible, not make two dierent denitions for what is essentially
the same object. This is a justiable point of view (and is the point
of view taken by Spivak and Munkres
1
). There are, however, some
advantages to distinguishing between A
k
and
k
, as well see in 1.6.
Exercises.
1. A k-tensor, T, L
k
(V ) is symmetric if T

= T for all S
k
.
Show that the set, S
k
(V ), of symmetric k tensors is a vector subspace
of L
k
(V ).
2. Let e
1
, . . . , e
n
be a basis of V . Show that every symmetric 2-
tensor is of the form

a
ij
e

i
e

j
where a
i,j
= a
j,i
and e

1
, . . . , e

n
are the dual basis vectors of V

.
3. Show that if T is a symmetric k-tensor, then for k 2, T is
in I
k
. Hint: Let be a transposition and deduce from the identity,
T

= T, that T has to be in the kernel of Alt .


4. Warning: In general S
k
(V ) = I
k
(V ). Show, however, that if
k = 2 these two spaces are equal.
5. Show that if V

and T I
k2
, then T is in I
k
.
6. Show that if
1
and
2
are in V

and T is in I
k2
, then
1

T
2
+
2
T
1
is in I
k
.
7. Given a permutation S
k
and T I
k
, show that T

I
k
.
8. Let W be a subspace of L
k
having the following two properties.
(a) For S S
2
(V ) and T L
k2
, S T is in W.
(b) For T in W and S
k
, T

is in W.
1
and by the author of these notes in his book with Alan Pollack, Dierential Topol-
ogy
30 Chapter 1. Multilinear algebra
Show that W has to contain I
k
and conclude that I
k
is the small-
est subspace of L
k
having properties a and b.
9. Show that there is a bijective linear map
:
k
A
k
with the property
(1.5.5) (T) =
1
k!
Alt (T)
for all T L
k
, and show that is the inverse of the map of A
k
onto

k
described in Theorem 1.5.6 (Hint: 1.2, exercise 8).
10. Let V be an n-dimensional vector space. Compute the dimen-
sion of S
k
(V ). Some hints:
(a) Introduce the following symmetrization operation on tensors
T L
k
(V ):
Sym(T) =

S
k
T

.
Prove that this operation has properties 2, 3 and 4 of Proposi-
tion 1.4.6 and, as a substitute for property 1, has the property:
(SymT)

= SymT.
(b) Let
I
= Sym(e

I
), e

I
= e

i
1
e

in
. Prove that {
I
, I
non-decreasing} form a basis of S
k
(V ).
(c) Conclude from (b) that dimS
k
(V ) is equal to the number of
non-decreasing multi-indices of length k: 1 i
1
i
2

k
n.
(d) Compute this number by noticing that
(i
1
, . . . , i
n
) (i
1
+ 0, i
2
+ 1, . . . , i
k
+k 1)
is a bijection between the set of these non-decreasing multi-indices
and the set of increasing multi-indices 1 j
1
< < j
k
n+k 1.
1.6 The wedge product 31
1.6 The wedge product
The tensor algebra operations on the spaces, L
k
(V ), which we dis-
cussed in Sections 1.2 and 1.3, i.e., the tensor product operation
and the pull-back operation, give rise to similar operations on the
spaces,
k
. We will discuss in this section the analogue of the tensor
product operation. As in 4 well abbreviate L
k
(V ) to L
k
and
k
(V )
to
k
when its clear which V is intended.
Given
i

k
i
, i = 1, 2 we can, by (1.5.4), nd a T
i
L
k
i
with

i
= (T
i
). Then T
1
T
2
L
k
1
+k
2
. Let
(1.6.1)
1

2
= (T
1
T
2
)
k
1
+k
2
.
Claim.
This wedge product is well dened, i.e., doesnt depend on our choices
of T
1
and T
2
.
Proof. Let (T
1
) = (T

1
) =
1
. Then T

1
= T
1
+W
1
for some W
1

I
k
1
, so
T

1
T
2
= T
1
T
2
+W
1
T
2
.
But W
1
I
k
1
implies W
1
T
2
I
k
1
+k
2
and this implies:
(T

1
T
2
) = (T
1
T
2
) .
A similar argument shows that (1.6.1) is well-dened independent of
the choice of T
2
.
More generally let
i

k
i
, i = 1, 2, 3, and let
i
= (T
i
), T
i

L
k
i
. Dene

1

2

3

k
1
+k
2
+k
3
by setting

1

2

3
= (T
1
T
2
T
3
) .
As above its easy to see that this is well-dened independent of the
choice of T
1
, T
2
and T
3
. It is also easy to see that this triple wedge
product is just the wedge product of
1

2
with
3
or, alternatively,
the wedge product of
1
with
2

3
, i.e.,
(1.6.2)
1

2

3
= (
1

2
)
3
=
1
(
2

3
).
32 Chapter 1. Multilinear algebra
We leave for you to check:
For R
(1.6.3) (
1

2
) = (
1
)
2
=
1
(
2
)
and verify the two distributive laws:
(
1
+
2
)
3
=
1

3
+
2

3
(1.6.4)
and

1
(
2
+
3
) =
1

2
+
1

3
. (1.6.5)
As we noted in 1.4, I
k
= {0} for k = 1, i.e., there are no non-zero
redundant k tensors in degree k = 1. Thus
(1.6.6)
1
(V

) = V

= L
1
(V

).
A particularly interesting example of a wedge product is the fol-
lowing. Let
i
V

=
1
(V

), i = 1, . . . , k. Then if T =
1

k
(1.6.7)
1

k
= (T)
k
(V

) .
We will call (1.6.7) a decomposable element of
k
(V

).
We will prove that these elements satisfy the following wedge prod-
uct identity. For S
k
:
(1.6.8)
(1)

(k)
= (1)

1

k
.
Proof. For every T L
k
, T = (1)

T + W for some W I
k
by
Proposition 1.5.4. Therefore since (W) = 0
(1.6.9) (T

) = (1)

(T) .
In particular, if T =
1

k
, T

=
(1)

(k)
, so
(T

) =
(1)

(k)
= (1)

(T)
= (1)

1

k
.
In particular, for
1
and
2
V

(1.6.10)
1

2
=
2

1
1.6 The wedge product 33
and for
1
,
2
and
3
V

(1.6.11)
1

2

3
=
2

1

3
=
2

3

1
.
More generally, its easy to deduce from (1.6.8) the following result
(which well leave as an exercise).
Theorem 1.6.1. If
1

r
and
2

s
then
(1.6.12)
1

2
= (1)
rs

2

1
.
Hint: It suces to prove this for decomposable elements i.e., for

1
=
1

r
and
2
=

s
. Now make rs applications
of (1.6.10).
Let e
1
, . . . , e
n
be a basis of V and let e

1
, . . . , e

n
be the dual basis
of V

. For every multi-index, I, of length k,
(1.6.13) e

i
1
e

i
k
= (e

I
) = (e

i
1
e

i
k
) .
Theorem 1.6.2. The elements (1.6.13), with I strictly increasing,
are basis vectors of
k
.
Proof. The elements

I
= Alt (e

I
) , I strictly increasing,
are basis vectors of A
k
by Proposition 3.6; so their images, (
I
),
are a basis of
k
. But
(
I
) =

(1)

(e

I
)

(1)

(e

I
)

(1)

(1)

(e

I
)
= k!(e

I
) .
Exercises:
1. Prove the assertions (1.6.3), (1.6.4) and (1.6.5).
2. Verify the multiplication law, (1.6.12) for wedge product.
34 Chapter 1. Multilinear algebra
3. Given
r
let
k
be the k-fold wedge product of with
itself, i.e., let
2
= ,
3
= , etc.
(a) Show that if r is odd then for k > 1,
k
= 0.
(b) Show that if is decomposable, then for k > 1,
k
= 0.
4. If and are in
2r
prove:
( +)
k
=
k

=0
_
k


k
.
Hint: As in freshman calculus prove this binomial theorem by induc-
tion using the identity:
_
k

_
=
_
k1
1
_
+
_
k1

_
.
5. Let be an element of
2
. By denition the rank of is k if

k
= 0 and
k+1
= 0. Show that if
= e
1
f
1
+ +e
k
f
k
with e
i
, f
i
V

, then is of rank k. Hint: Show that

k
= k!e
1
f
1
e
k
f
k
.
6. Given e
i
V

, i = 1, . . . , k show that e
1
e
k
= 0 if and
only if the e
i
s are linearly independent. Hint: Induction on k.
1.7 The interior product 35
1.7 The interior product
Well describe in this section another basic product operation on the
spaces,
k
(V

). As above well begin by dening this operator on
the L
k
(V )s. Given T L
k
(V ) and v V let
v
T be the be the
(k 1)-tensor which takes the value
(1.7.1)

v
T(v
1
, . . . , v
k1
) =
k

r=1
(1)
r1
T(v
1
, . . . , v
r1
, v, v
r
, . . . , v
k1
)
on the k 1-tuple of vectors, v
1
, . . . , v
k1
, i.e., in the r
th
summand
on the right, v gets inserted between v
r1
and v
r
. (In particular
the rst summand is T(v, v
1
, . . . , v
k1
) and the last summand is
(1)
k1
T(v
1
, . . . , v
k1
, v).) Its clear from the denition that if v =
v
1
+ v
2

v
T =
v
1
T +
v
2
T , (1.7.2)
and if T = T
1
+T
2

v
T =
v
T
1
+
v
T
2
, (1.7.3)
and we will leave for you to verify by inspection the following two
lemmas:
Lemma 1.7.1. If T is the decomposable k-tensor
1

k
then
(1.7.4)
v
T =

(1)
r1

r
(v)
1

r

k
where the cap over
r
means that its deleted from the tensor prod-
uct ,
and
Lemma 1.7.2. If T
1
L
p
and T
2
L
q
(1.7.5)
v
(T
1
T
2
) =
v
T
1
T
2
+ (1)
p
T
1

v
T
2
.
We will next prove the important identity
(1.7.6)
v
(
v
T) = 0 .
Proof. It suces by linearity to prove this for decomposable tensors
and since (1.7.6) is trivially true for T L
1
, we can by induction
36 Chapter 1. Multilinear algebra
assume (1.7.6) is true for decomposible tensors of degree k 1. Let

1

k
be a decomposable tensor of degree k. Setting T =

1

k1
and =
k
we have

v
(
1

k
) =
v
(T )
=
v
T + (1)
k1
(v)T
by (1.7.5). Hence

v
(
v
(T )) =
v
(
v
T) + (1)
k2
(v)
v
T
+(1)
k1
(v)
v
T .
But by induction the rst summand on the right is zero and the two
remaining summands cancel each other out.
From (1.7.6) we can deduce a slightly stronger result: For v
1
, v
2

V
(1.7.7)
v
1

v
2
=
v
2

v
1
.
Proof. Let v = v
1
+ v
2
. Then
v
=
v
1
+
v
2
so
0 =
v

v
= (
v
1
+
v
2
)(
v
1
+
v
2
)
=
v
1

v
1
+
v
1

v
2
+
v
2

v
1
+
v
2

v
2
=
v
1

v
2
+
v
2

v
1
since the rst and last summands are zero by (1.7.6).
Well now show how to dene the operation,
v
, on
k
(V

). Well
rst prove
Lemma 1.7.3. If T L
k
is redundant then so is
v
T.
Proof. Let T = T
1
T
2
where is in V

, T
1
is in L
p
and T
2
is in L
q
. Then by (1.7.5)

v
T =
v
T
1
T
2
+(1)
p
T
1

v
( ) T
2
+(1)
p+2
T
1

v
T
2
.
1.7 The interior product 37
However, the rst and the third terms on the right are redundant
and

v
( ) = (v) (v)
by (1.7.4).
Now let be the projection (1.5.4) of L
k
onto
k
and for =
(T)
k
dene
(1.7.8)
v
= (
v
T) .
To show that this denition is legitimate we note that if = (T
1
) =
(T
2
), then T
1
T
2
I
k
, so by Lemma 1.7.3
v
T
1

v
T
2
I
k1
and
hence
(
v
T
1
) = (
v
T
2
) .
Therefore, (1.7.8) doesnt depend on the choice of T.
By denition
v
is a linear mapping of
k
(V

) into
k1
(V

).
We will call this the interior product operation. From the identities
(1.7.2)(1.7.8) one gets, for v, v
1
, v
2
V
k
,
1

p
and

2

2

(v
1
+v
2
)
=
v
1
+
v
2
(1.7.9)

v
(
1

2
) =
v

1

2
+ (1)
p

1

v

2
(1.7.10)

v
(
v
) = 0 (1.7.11)
and

v
1

v
2
=
v
2

v
1
. (1.7.12)
Moreover if =
1

k
is a decomposable element of
k
one
gets from (1.7.4)
(1.7.13)
v
=
k

r=1
(1)
r1

r
(v)
1

r

k
.
In particular if e
1
, . . . , e
n
is a basis of V , e

1
, . . . , e

n
the dual basis of
V

and
I
= e

i
1
e

i
k
, 1 i
1
< < i
k
n, then (e
j
)
I
= 0
if j / I and if j = i
r
(1.7.14) (e
j
)
I
= (1)
r1

Ir
where I
r
= (i
1
, . . . ,

i
r
, . . . , i
k
) (i.e., I
r
is obtained from the multi-
index I by deleting i
r
).
38 Chapter 1. Multilinear algebra
Exercises:
1. Prove Lemma 1.7.1.
2. Prove Lemma 1.7.2.
3. Show that if T A
k
, i
v
= kT
v
where T
v
is the tensor (1.3.16).
In particular conclude that i
v
T A
k1
. (See 1.4, exercise 8.)
4. Assume the dimension of V is n and let be a non-zero element
of the one dimensional vector space
n
. Show that the map
(1.7.15) : V
n1
, v
v
,
is a bijective linear map. Hint: One can assume = e

1
e

n
where e
1
, . . . , e
n
is a basis of V . Now use (1.7.14) to compute this
map on basis elements.
5. (The cross-product.) Let V be a 3-dimensional vector space, B
an inner product on V and a non-zero element of
3
. Dene a map
V V V
by setting
(1.7.16) v
1
v
2
=
1
(Lv
1
Lv
2
)
where is the map (1.7.15) and L : V V

the map (1.2.9). Show
that this map is linear in v
1
, with v
2
xed and linear in v
2
with v
1
xed, and show that v
1
v
2
= v
2
v
1
.
6. For V = R
3
let e
1
, e
2
and e
3
be the standard basis vectors and
B the standard inner product. (See 1.1.) Show that if = e

1
e

2
e

3
the cross-product above is the standard cross-product:
e
1
e
2
= e
3
e
2
e
3
= e
1
(1.7.17)
e
3
e
1
= e
2
.
Hint: If B is the standard inner product Le
i
= e

i
.
Remark 1.7.4. One can make this standard cross-product look even
more standard by using the calculus notation: e
1
=

i, e
2
=

j and
e
3
=

k
1.8 The pull-back operation on
k
39
1.8 The pull-back operation on
k
Let V and W be vector spaces and let A be a linear map of V into
W. Given a k-tensor, T L
k
(W), the pull-back, A

T, is the k-tensor
(1.8.1) A

T(v
1
, . . . , v
k
) = T(Av
1
, . . . , Av
k
)
in L
k
(V ). (See 1.3, equation 1.3.12.) In this section well show how
to dene a similar pull-back operation on
k
.
Lemma 1.8.1. If T I
k
(W), then A

T I
k
(V ).
Proof. It suces to verify this when T is a redundant k-tensor, i.e., a
tensor of the form
T =
1

k
where
r
W

and
i
=
i+1
for some index, i. But by (1.3.14)
A

T = A

1
A

k
and the tensor on the right is redundant since A

i
= A

i+1
.
Now let be an element of
k
(W

) and let = (T) where T is


in L
k
(W). We dene
(1.8.2) A

= (A

T) .
Claim:
The left hand side of (1.8.2) is well-dened.
Proof. If = (T) = (T

), then T = T

+ S for some S I
k
(W),
and A

= A

T +A

S. But A

S I
k
(V ), so
(A

) = (A

T) .
Proposition 1.8.2. The map
A

:
k
(W

)
k
(V

) ,
mapping to A

is linear. Moreover,
40 Chapter 1. Multilinear algebra
(i) If
i

k
i
(W), i = 1, 2, then
(1.8.3) A

(
1

2
) = A

1
A

2
.
(ii) If U is a vector space and B : U V a linear map, then
for
k
(W

),
(1.8.4) B

= (AB)

.
Well leave the proof of these three assertions as exercises. Hint:
They follow immediately from the analogous assertions for the pull-
back operation on tensors. (See (1.3.14) and (1.3.15).)
As an application of the pull-back operation well show how to
use it to dene the notion of determinant for a linear mapping. Let
V be a n-dimensional vector space. Then dim
n
(V

) =
_
n
n
_
= 1;
i.e.,
n
(V

) is a one-dimensional vector space. Thus if A : V V


is a linear mapping, the induced pull-back mapping:
A

:
n
(V

)
n
(V

) ,
is just multiplication by a constant. We denote this constant by
det(A) and call it the determinant of A, Hence, by denition,
(1.8.5) A

= det(A)
for all in
n
(V

). From (1.8.5) its easy to derive a number of basic
facts about determinants.
Proposition 1.8.3. If A and B are linear mappings of V into V ,
then
(1.8.6) det(AB) = det(A) det(B) .
Proof. By (1.8.4) and
(AB)

= det(AB)
= B

(A

) = det(B)A

= det(B) det(A) ,
so, det(AB) = det(A) det(B).
1.8 The pull-back operation on
k
41
Proposition 1.8.4. If I : V V is the identity map, Iv = v for
all v V , det(I) = 1.
Well leave the proof as an exercise. Hint: I

is the identity map


on
n
(V

).
Proposition 1.8.5. If A : V V is not onto, det(A) = 0.
Proof. Let W be the image of A. Then if A is not onto, the dimension
of W is less than n, so
n
(W

) = {0}. Now let A = I


W
B where I
W
is the inclusion map of W into V and B is the mapping, A, regarded
as a mapping from V to W. Thus if is in
n
(V

), then by (1.8.4)
A

= B

and since I

W
is in
n
(W) it is zero.
We will derive by wedge product arguments the familiar matrix
formula for the determinant. Let V and W be n-dimensional vector
spaces and let e
1
, . . . , e
n
be a basis for V and f
1
, . . . , f
n
a basis for
W. From these bases we get dual bases, e

1
, . . . , e

n
and f

1
, . . . , f

n
,
for V

and W

. Moreover, if A is a linear map of V into W and


[a
i,j
] the nn matrix describing A in terms of these bases, then the
transpose map, A

: W

V

, is described in terms of these dual
bases by the n n transpose matrix, i.e., if
Ae
j
=

a
i,j
f
i
,
then
A

j
=

a
j,i
e

i
.
(See 2.) Consider now A

(f

1
f

n
). By (1.8.3)
A

(f

1
f

n
) = A

1
A

n
=

(a
1,k
1
e

k
1
) (a
n,kn
e

kn
)
the sum being over all k
1
, . . . , k
n
, with 1 k
r
n. Thus,
A

(f

1
f

n
) =

a
1,k
1
. . . a
n,kn
e

k
1
e

kn
.
42 Chapter 1. Multilinear algebra
If the multi-index, k
1
, . . . , k
n
, is repeating, then e

k
1
e

kn
is zero,
and if its not repeating then we can write
k
i
= (i) i = 1, . . . , n
for some permutation, , and hence we can rewrite A

(f

1
f

n
)
as the sum over S
n
of

a
1,(1)
a
n,(n)
(e

1
e

n
)

.
But
(e

1
e

n
)

= (1)

1
e

n
so we get nally the formula
(1.8.7) A

(f

1
f

n
) = det[a
i,j
]e

1
e

n
where
(1.8.8) det[a
i,j
] =

(1)

a
1,(1)
a
n,(n)
summed over S
n
. The sum on the right is (as most of you know)
the determinant of [a
i,j
].
Notice that if V = W and e
i
= f
i
, i = 1, . . . , n, then = e

1

e

n
= f

1
f

n
, hence by (1.8.5) and (1.8.7),
(1.8.9) det(A) = det[a
i,j
] .
Exercises.
1. Verify the three assertions of Proposition 1.8.2.
2. Deduce from Proposition 1.8.5 a well-known fact about deter-
minants of nn matrices: If two columns are equal, the determinant
is zero.
3. Deduce from Proposition 1.8.3 another well-known fact about
determinants of n n matrices: If one interchanges two columns,
then one changes the sign of the determinant.
Hint: Let e
1
, . . . , e
n
be a basis of V and let B : V V be the
linear mapping: Be
i
= e
j
, Be
j
= e
i
and Be

= e

, = i, j. What is
B

(e

1
e

n
)?
1.8 The pull-back operation on
k
43
4. Deduce from Propositions 1.8.3 and 1.8.4 another well-known
fact about determinants of n n matrix. If [b
i,j
] is the inverse of
[a
i,j
], its determinant is the inverse of the determinant of [a
i,j
].
5. Extract from (1.8.8) a well-known formula for determinants of
2 2 matrices:
det
_
a
11
, a
12
a
21
, a
22
_
= a
11
a
22
a
12
a
21
.
6. Show that if A = [a
i,j
] is an n n matrix and A
t
= [a
j,i
] is its
transpose det A = det A
t
. Hint: You are required to show that the
sums

(1)

a
1,(1)
. . . a
n,(n)
S
n
and

(1)

a
(1),1
. . . a
(n),n
S
n
are the same. Show that the second sum is identical with

(1)

a
(1),1
. . . a
(n),n
summed over =
1
S
n
.
7. Let A be an n n matrix of the form
A =
_
B
0 C
_
where B is a k k matrix and C the matrix and the bottom
k block is zero. Show that
det A = det Bdet C .
Hint: Show that in (1.8.8) every non-zero term is of the form
(1)

b
1,(1)
. . . b
k,(k)
c
1,(1)
. . . c
,()
where S
k
and S

.
8. Let V and W be vector spaces and let A : V W be a linear
map. Show that if Av = w then for
p
(w

),
A

(w) = (v)A

.
(Hint: By (1.7.10) and proposition 1.8.2 it suces to prove this for

1
(W

), i.e., for W

.)
44 Chapter 1. Multilinear algebra
1.9 Orientations
We recall from freshman calculus that if R
2
is a line through the
origin, then {0} has two connected components and an orientation
of is a choice of one of these components (as in the gure below).

More generally, if L is a one-dimensional vector space then L{0}


consists of two components: namely if v is an element of L[0}, then
these two components are
L
1
= {v > 0}
and
L
2
= {v, < 0} .
An orientation of L is a choice of one of these components. Usu-
ally the component chosen is denoted L
+
, and called the positive
component of L {0} and the other component, L

, the negative
component of L {0}.
Denition 1.9.1. A vector, v L, is positively oriented if v is in
L
+
.
More generally still let V be an n-dimensional vector space. Then
L =
n
(V

) is one-dimensional, and we dene an orientation of V
to be an orientation of L. One important way of assigning an orien-
tation to V is to choose a basis, e
1
, . . . , e
n
of V . Then, if e

1
, . . . , e

n
is
the dual basis, we can orient
n
(V

) by requiring that e

1
e

n
be
in the positive component of
n
(V

). If V has already been assigned


an orientation we will say that the basis, e
1
, . . . , e
n
, is positively ori-
ented if the orientation we just described coincides with the given
orientation.
Suppose that e
1
, . . . , e
n
and f
1
, . . . , f
n
are bases of V and that
(1.9.1) e
j
=

a
i,j,
f
i
.
1.9 Orientations 45
Then by (1.7.7)
f

1
f

n
= det[a
i,j
]e

1
e

n
so we conclude:
Proposition 1.9.2. If e
1
, . . . , e
n
is positively oriented, then f
1
, . . . , f
n
is positively oriented if and only if det[a
i,j
] is positive.
Corollary 1.9.3. If e
1
, . . . , e
n
is a positively oriented basis of V , the
basis: e
1
, . . . , e
i1
, e
i
, e
i+1
, . . . , e
n
is negatively oriented.
Now let V be a vector space of dimension n > 1 and W a sub-
space of dimension k < n. We will use the result above to prove the
following important theorem.
Theorem 1.9.4. Given orientations on V and V/W, one gets from
these orientations a natural orientation on W.
Remark What we mean by natural will be explained in the course
of the proof.
Proof. Let r = n k and let be the projection of V onto V/W
. By exercises 1 and 2 of 2 we can choose a basis e
1
, . . . , e
n
of V
such that e
r+1
, . . . , e
n
is a basis of W and (e
1
), . . . , (e
r
) a basis
of V/W. Moreover, replacing e
1
by e
1
if necessary we can assume
by Corollary 1.9.3 that (e
1
), . . . , (e
r
) is a positively oriented basis
of V/W and replacing e
n
by e
n
if necessary we can assume that
e
1
, . . . , e
n
is a positively oriented basis of V . Now assign to W the
orientation associated with the basis e
r+1
, . . . , e
n
.
Lets show that this assignment is natural (i.e., doesnt depend
on our choice of e
1
, . . . , e
n
). To see this let f
1
, . . . , f
n
be another
basis of V with the properties above and let A = [a
i,j
] be the matrix
(1.9.1) expressing the vectors e
1
, . . . , e
n
as linear combinations of the
vectors f
1
, . . . f
n
. This matrix has to have the form
(1.9.2) A =
_
B C
0 D
_
where B is the rr matrix expressing the basis vectors (e
1
), . . . , (e
r
)
of V/W as linear combinations of (f
1
), . . . , (f
r
) and D the k k
matrix expressing the basis vectors e
r+1
, . . . , e
n
of W as linear com-
binations of f
r+1
, . . . , f
n
. Thus
det(A) = det(B) det(D) .
46 Chapter 1. Multilinear algebra
However, by Proposition 1.9.2, det A and det B are positive, so det D
is positive, and hence if e
r+1
, . . . , e
n
is a positively oriented basis of
W so is f
r+1
, . . . , f
n
.
As a special case of this theorem suppose dimW = n 1. Then
the choice of a vector v V W gives one a basis vector, (v),
for the one-dimensional space V/W and hence if V is oriented, the
choice of v gives one a natural orientation on W.
Next let V
i
, i = 1, 2 be oriented n-dimensional vector spaces and
A : V
1
V
2
a bijective linear map. A is orientation-preserving if,
for
n
(V

2
)
+
, A

is in
n
(V

+
)
+
. For example if V
1
= V
2
then
A

= det(A) so A is orientation preserving if and only if det(A) >


0. The following proposition well leave as an exercise.
Proposition 1.9.5. Let V
i
, i = 1, 2, 3 be oriented n-dimensional
vector spaces and A
i
: V
i
V
i+1
, i = 1, 2 bijective linear maps.
Then if A
1
and A
2
are orientation preserving, so is A
2
A
1
.
Exercises.
1. Prove Corollary 1.9.3.
2. Show that the argument in the proof of Theorem 1.9.4 can be
modied to prove that if V and W are oriented then these orienta-
tions induce a natural orientation on V/W.
3. Similarly show that if W and V/W are oriented these orienta-
tions induce a natural orientation on V .
4. Let V be an n-dimensional vector space and W V a k-
dimensional subspace. Let U = V/W and let : W V and
: V U be the inclusion and projection maps. Suppose V and U
are oriented. Let be in
nk
(U

)
+
and let be in
n
(V

)
+
. Show
that there exists a in
k
(V

) such that

= . Moreover
show that

is intrinsically dened (i.e., doesnt depend on how


we choose ) and sits in the positive part,
k
(W

)
+
, of
k
(W).
5. Let e
1
, . . . , e
n
be the standard basis vectors of R
n
. The standard
orientation of R
n
is, by denition, the orientation associated with
this basis. Show that if W is the subspace of R
n
dened by the
1.9 Orientations 47
equation, x
1
= 0, and v = e
1
W then the natural orientation of W
associated with v and the standard orientation of R
n
coincide with
the orientation given by the basis vectors, e
2
, . . . , e
n
of W.
6. Let V be an oriented n-dimensional vector space and W an
n1-dimensional subspace. Show that if v and v

are in V W then
v

= v +w, where w is in W and R {0}. Show that v and v

give rise to the same orientation of W if and only if is positive.


7. Prove Proposition 1.9.5.
8. A key step in the proof of Theorem 1.9.4 was the assertion that
the matrix A expressing the vectors, e
i
, as linear combinations of the
vectors, f
i
, had to have the form (1.9.2). Why is this the case?
9. (a) Let V be a vector space, W a subspace of V and A : V
V a bijective linear map which maps W onto W. Show that one gets
from A a bijective linear map
B : V/W V/W
with property
A = B ,
being the projection of V onto V/W.
(b) Assume that V , W and V/W are compatibly oriented. Show
that if A is orientation-preserving and its restriction to W is orien-
tation preserving then B is orientation preserving.
10. Let V be a oriented n-dimensional vector space, W an (n1)-
dimensional subspace of V and i : W V the inclusion map. Given

b
(V )
+
and v V W show that for the orientation of W
described in exercise 5, i

(
v
)
n1
(W)
+
.
11. Let V be an n-dimensional vector space, B : V V R an
inner product and e
1
, . . . , e
n
a basis of V which is positively oriented
and orthonormal. Show that the volume element
vol = e

1
e

n

n
(V

)
is intrinsically dened, independent of the choice of this basis. Hint:
(1.2.13) and (1.8.7).
48 Chapter 1. Multilinear algebra
12. (a) Let V be an oriented n-dimensional vector space and B an
inner product on V . Fix an oriented orthonormal basis, e
1
, . . . , e
n
,
of V and let A : V V be a linear map. Show that if
Ae
i
= v
i
=

a
j,i
e
j
and b
i,j
= B(v
i
, v
j
), the matrices A = [a
i,j
] and B = [b
i,j
] are related
by: B = A
+
A.
(b) Show that if is the volume form, e

1
e

n
, and A is orien-
tation preserving
A

= (det B)
1
2
.
(c) By Theorem 1.5.6 one has a bijective map

n
(V

)

= A
n
(V ) .
Show that the element, , of A
n
(V ) corresponding to the form, ,
has the property
|(v
1
, . . . , v
n
)|
2
= det([b
i,j
])
where v
1
, . . . , v
n
are any n-tuple of vectors in V and b
i,j
= B(v
i
, v
j
).
This is page 49
Printer: Opaque this
CHAPTER 2
DIFFERENTIAL FORMS
2.1 Vector elds and one-forms
The goal of this chapter is to generalize to n dimensions the basic
operations of three dimensional vector calculus: div, curl and grad.
The div, and grad operations have fairly straight forward gener-
alizations, but the curl operation is more subtle. For vector elds
it doesnt have any obvious generalization, however, if one replaces
vector elds by a closely related class of objects, dierential forms,
then not only does it have a natural generalization but it turns out
that div, curl and grad are all special cases of a general operation on
dierential forms called exterior dierentiation.
In this section we will review some basic facts about vector elds
in n variables and introduce their dual objects: one-forms. We will
then take up in 2.2 the theory of k-forms for k greater than one.
We begin by xing some notation.
Given p R
n
we dene the tangent space to R
n
at p to be the set
of pairs
(2.1.1) T
p
R
n
= {(p, v)} ; v R
n
.
The identication
(2.1.2) T
p
R
n
R
n
, (p, v) v
makes T
p
R
n
into a vector space. More explicitly, for v, v
1
and v
2
R
n
and R we dene the addition and scalar multiplication operations
on T
p
R
n
by the recipes
(p, v
1
) + (p, v
2
) = (p, v
1
+ v
2
)
and
(p, v) = (p, v) .
Let U be an open subset of R
n
and f : U R
m
a C
1
map. We
recall that the derivative
Df(p) : R
n
R
m
50 Chapter 2. Dierential forms
of f at p is the linear map associated with the mn matrix
_
f
i
x
j
(p)
_
.
It will be useful to have a base-pointed version of this denition
as well. Namely, if q = f(p) we will dene
df
p
: T
p
R
n
T
q
R
m
to be the map
(2.1.3) df
p
(p, v) = (q, Df(p)v) .
Its clear from the way weve dened vector space structures on T
p
R
n
and T
q
R
m
that this map is linear.
Suppose that the image of f is contained in an open set, V , and
suppose g : V R
k
is a C
1
map. Then the base-pointed version
of the chain rule asserts that
(2.1.4) dg
q
df
p
= d(f g)
p
.
(This is just an alternative way of writing Dg(q)Df(p) = D(g
f)(p).)
In 3-dimensional vector calculus a vector eld is a function which
attaches to each point, p, of R
3
a base-pointed arrow, (p, v). The
n-dimensional version of this denition is essentially the same.
Denition 2.1.1. Let U be an open subset of R
n
. A vector eld on
U is a function, v, which assigns to each point, p, of U a vector v(p)
in T
p
R
n
.
Thus a vector eld is a vector-valued function, but its value at p
is an element of a vector space, T
p
R
n
that itself depends on p.
Some examples.
1. Given a xed vector, v R
n
, the function
(2.1.5) p R
n
(p, v)
is a vector eld. Vector elds of this type are constant vector elds.
2. In particular let e
i
, i = 1, . . . , n, be the standard basis vectors
of R
n
. If v = e
i
we will denote the vector eld (2.1.5) by /x
i
. (The
reason for this derivation notation will be explained below.)
2.1 Vector elds and one-forms 51
3. Given a vector eld on U and a function, f : U R well
denote by fv the vector eld
p U f(p)v(p) .
4. Given vector elds v
1
and v
2
on U, well denote by v
1
+v
2
the
vector eld
p U v
1
(p) +v
2
(p) .
5. The vectors, (p, e
i
), i = 1, . . . , n, are a basis of T
p
R
n
, so if
v is a vector eld on U, v(p) can be written uniquely as a linear
combination of these vectors with real numbers, g
i
(p), i = 1, . . . , n,
as coecients. In other words, using the notation in example 2 above,
v can be written uniquely as a sum
(2.1.6) v =
n

i=1
g
i

x
i
where g
i
: U R is the function, p g
i
(p).
Well say that v is a C

vector eld if the g


i
s are in C

(U).
A basic vector eld operation is Lie dierentiation. If f C
1
(U)
we dene L
v
f to be the function on U whose value at p is given by
(2.1.7) Df(p)v = L
v
f(p)
where v(p) = (p, v). If v is the vector eld (2.1.6) then
(2.1.8) L
v
f =

g
i

x
i
f
(motivating our derivation notation for v).
Exercise.
Check that if f
i
C
1
(U), i = 1, 2, then
(2.1.9) L
v
(f
1
f
2
) = f
1
L
v
f
2
+f
1
L
v
f
2
.
Next well generalize to n-variables the calculus notion of an in-
tegral curve of a vector eld.
52 Chapter 2. Dierential forms
Denition 2.1.2. A C
1
curve : (a, b) U is an integral curve of
v if for all a < t < b and p = (t)
_
p,
d
dt
(t)
_
= v(p)
i.e., if v is the vector eld (2.1.6) and g : U R
n
is the function
(g
1
, . . . , g
n
) the condition for (t) to be an integral curve of v is that
it satisfy the system of dierential equations
(2.1.10)
d
dt
(t) = g((t)) .
We will quote without proof a number of basic facts about systems
of ordinary dierential equations of the type (2.1.10). (A source for
these results that we highly recommend is BirkhoRota, Ordinary
Dierential Equations, Chapter 6.)
Theorem 2.1.3 (Existence). Given a point p
0
U and a R, there
exists an interval I = (a T, a +T), a neighborhood, U
0
, of p
0
in U
and for every p U
0
an integral curve,
p
: I U with
p
(a) = p.
Theorem 2.1.4 (Uniqueness). Let
i
: I
i
U, i = 1, 2, be integral
curves. If a I
1
I
2
and
1
(a) =
2
(a) then
1

2
on I
1
I
2
and
the curve : I
1
I
2
U dened by
(t) =
_

1
(t) , t I
1

2
(t) , t I
2
is an integral curve.
Theorem 2.1.5 (Smooth dependence on initial data). Let v be a
C

-vector eld, on an open subset, V , of U, I R an open interval,


a I a point on this interval and h : V I U a mapping with the
properties:
(i) h(p, a) = p.
(ii) For all p V the curve

p
: I U
p
(t) = h(p, t)
is an integral curve of v.
Then the mapping, h, is C

.
2.1 Vector elds and one-forms 53
One important feature of the system (2.1.11) is that it is an au-
tonomous system of dierential equations: the function, g(x), is a
function of x alone, it doesnt depend on t. One consequence of this
is the following:
Theorem 2.1.6. Let I = (a, b) and for c R let I
c
= (a c, b c).
Then if : I U is an integral curve, the reparametrized curve
(2.1.11)
c
: I
c
U ,
c
(t) = (t +c)
is an integral curve.
We recall that a C
1
-function : U R is an integral of the system
(2.1.11) if for every integral curve (t), the function t ((t)) is
constant. This is true if and only if for all t and p = (t)
0 =
d
dt
((t)) = (D)
p
_
d
dt
_
= (D)
p
(v)
where (p, v) = v(p). But by (2.1.6) the term on the right is L
v
(p).
Hence we conclude
Theorem 2.1.7. C
1
(U) is an integral of the system (2.1.11) if
and only if L
v
= 0.
Well now discuss a class of objects which are in some sense dual
objects to vector elds. For each p R
n
let (T
p
R)

be the dual vec-


tor space to T
p
R
n
, i.e., the space of all linear mappings, : T
p
R
n

R.
Denition 2.1.8. Let U be an open subset of R
n
. A one-form on U
is a function, , which assigns to each point, p, of U a vector,
p
,
in (T
p
R
n
)

.
Some examples:
1. Let f : U R be a C
1
function. Then for p U and
c = f(p) one has a linear map
(2.1.12) df
p
: T
p
R
n
T
c
R
and by making the identication,
T
c
R = {c, R} = R
54 Chapter 2. Dierential forms
df
p
can be regarded as a linear map from T
p
R
n
to R, i.e., as an
element of (T
p
R
n
)

. Hence the assignment


(2.1.13) p U df
p
(T
p
R
n
)

denes a one-form on U which well denote by df.


2. Given a one-form and a function, : U R the product
of with is the one-form, p U (p)
p
.
3. Given two one-forms
1
and
2
their sum,
1
+
2
is the
one-form, p U
1
(p) +
2
(p).
4. The one-forms dx
1
, . . . , dx
n
play a particularly important
role. By (2.1.12)
(2.1.14) (dx
i
)
_

x
j
_
p
=
ij
i.e., is equal to 1 if i = j and zero if i = j. Thus (dx
1
)
p
, . . . , (dx
n
)
p
are the basis of (T

p
R
n
)

dual to the basis (/x


i
)
p
. Therefore,
if is any one-form on U,
p
can be written uniquely as a sum

p
=

f
i
(p)(dx
i
)
p
, f
i
(p) R
and can be written uniquely as a sum
(2.1.15) =

f
i
dx
i
where f
i
: U R is the function, p f
i
(p). Well say that
is a C

one-form if the f
i
s are C

.
Exercise.
Check that if f : U R is a C

function
(2.1.16) df =

f
x
i
dx
i
.
Suppose now that v is a vector eld and a one-form on U. Then
for every p U the vectors, v
p
T
p
R
n
and
p
(T
p
R
n
)

can be
paired to give a number, (v
p
)
p
R, and hence, as p varies, an
2.1 Vector elds and one-forms 55
R-valued function, (v), which we will call the interior product of v
with . For instance if v is the vector eld (2.1.6) and the one-form
(2.1.15) then
(2.1.17) (v) =

f
i
g
i
.
Thus if v and are C

so is the function (v). Also notice that if


C

(U), then as we observed above


d =


x
i

x
i
so if v is the vector eld (2.1.6)
(2.1.18) (v) d =

g
i

x
i
= L
v
.
Coming back to the theory of integral curves, let U be an open
subset of R
n
and v a vector eld on U. Well say that v is complete
if, for every p U, there exists an integral curve, : R U with
(0) = p, i.e., for every p there exists an integral curve that starts
at p and exists for all time. To see what completeness involves, we
recall that an integral curve
: [0, b) U ,
with (0) = p, is called maximal if it cant be extended to an interval
[0, b

), b

> b. (See for instance BirkhoRota, 6.11.) For such curves


its known that either
i. b = +
or
ii. |(t)| + as t b
or
iii. the limit set of
{(t) , 0 t, b}
contains points on the boundary of U.
Hence if we can exclude ii. and iii. well have shown that an integral
curve with (0) = p exists for all positive time. A simple criterion
for excluding ii. and iii. is the following.
56 Chapter 2. Dierential forms
Lemma 2.1.9. The scenarios ii. and iii. cant happen if there exists
a proper C
1
-function, : U R with L
v
= 0.
Proof. L
v
= 0 implies that is constant on (t), but if (p) = c
this implies that the curve, (t), lies on the compact subset,
1
(c),
of U; hence it cant run o to innity as in scenario ii. or run o
the boundary as in scenario iii.
Applying a similar argument to the interval (b, 0] we conclude:
Theorem 2.1.10. Suppose there exists a proper C
1
-function, :
U R with the property L
v
= 0. Then v is complete.
Example.
Let U = R
2
and let v be the vector eld
v = x
3

y
y

x
.
Then (x, y) = 2y
2
+x
4
is a proper function with the property above.
Another hypothesis on v which excludes ii. and iii. is the following.
Well dene the support of v to be the set
suppv = q U , v(q) = 0} ,
and will say that v is compactly supported if this set is compact. We
will prove
Theorem 2.1.11. If v is compactly supported it is complete.
Proof. Notice rst that if v(p) = 0, the constant curve,
0
(t) = p,
< t < , satises the equation
d
dt

0
(t) = 0 = v(p) ,
so it is an integral curve of v. Hence if (t), a < t < b, is any
integral curve of v with the property, (t
0
) = p, for some t
0
, it has
to coincide with
0
on the interval, a < t < a, and hence has to be
the constant curve, (t) = p, on this interval.
Now suppose the support, A, of v is compact. Then either (t) is
in A for all t or is in U A for some t
0
. But if this happens, and
2.1 Vector elds and one-forms 57
p = (t
0
) then v(p) = 0, so (t) has to coincide with the constant
curve,
0
(t) = p, for all t. In neither case can it go o to or o to
the boundary of U as t b.
One useful application of this result is the following. Suppose v is
a vector eld on U, and one wants to see what its integral curves
look like on some compact set, A U. Let C

0
(U) be a bump
function which is equal to one on a neighborhood of A. Then the
vector eld, w = v, is compactly supported and hence complete,
but it is identical with v on A, so its integral curves on A coincide
with the integral curves of v.
If v is complete then for every p, one has an integral curve,
p
:
R U with
p
(0) = p, so one can dene a map
f
t
: U U
by setting f
t
(p) =
p
(t). If v is C

, this mapping is C

by the
smooth dependence on initial data theorem, and by denition f
0
is
the identity map, i.e., f
0
(p) =
p
(0) = p. We claim that the f
t
s also
have the property
(2.1.19) f
t
f
a
= f
t+a
.
Indeed if f
a
(p) = q, then by the reparametrization theorem,
q
(t)
and
p
(t + a) are both integral curves of v, and since q =
q
(0) =

p
(a) = f
a
(p), they have the same initial point, so

q
(t) = f
t
(q) = (f
t
f
a
)(p)
=
p
(t +a) = f
t+a
(p)
for all t. Since f
0
is the identity it follows from (2.1.19) that f
t
f
t
is the identity, i.e.,
f
t
= f
1
t
,
so f
t
is a C

dieomorphism. Hence if v is complete it generates a


one-parameter group, f
t
, < t < , of C

-dieomorphisms.
For v not complete there is an analogous result, but its trickier to
formulate precisely. Roughly speaking v generates a one-parameter
group of dieomorphisms, f
t
, but these dieomorphisms are not de-
ned on all of U nor for all values of t. Moreover, the identity (2.1.19)
only holds on the open subset of U where both sides are well-dened.
58 Chapter 2. Dierential forms
Well devote the remainder of this section to discussing some func-
torial properties of vector elds and one-forms. Let U and W be
open subsets of R
n
and R
m
, respectively, and let f : U W be a
C

map. If v is a C

-vector eld on U and w a C

-vector eld on
W we will say that v and w are f-related if, for all p U and
q = f(p)
(2.1.20) df
p
(v
p
) = w
q
.
Writing
v =
n

i=1
v
i

x
i
, v
i
C
k
(U)
and
w =
m

j=1
w
j

y
j
, w
j
C
k
(V )
this equation reduces, in coordinates, to the equation
(2.1.21) w
i
(q) =

f
i
x
j
(p)v
j
(p) .
In particular, if m = n and f is a C

dieomorphism, the formula


(3.2) denes a C

-vector eld on W, i.e.,


w =
n

j=1
w
i

y
j
is the vector eld dened by the equation
(2.1.22) w
i
=
n

j=1
_
f
i
x
j
v
j
_
f
1
.
Hence weve proved
Theorem 2.1.12. If f : U W is a C

dieomorphism and v a
C

-vector eld on U, there exists a unique C

vector eld, w, on W
having the property that v and w are f-related.
Well denote this vector eld by f

v and call it the push-forward


of v by f.
Ill leave the following assertions as easy exercises.
2.1 Vector elds and one-forms 59
Theorem 2.1.13. Let U
i
, i = 1, 2, be open subsets of R
n
i
, v
i
a
vector eld on U
i
and f : U
1
U
2
a C

-map. If v
1
and v
2
are
f-related, every integral curve
: I U
1
of v
1
gets mapped by f onto an integral curve, f : I U
2
, of v
2
.
Corollary 2.1.14. Suppose v
1
and v
2
are complete. Let (f
i
)
t
: U
i

U
i
, < t < , be the one-parameter group of dieomorphisms
generated by v
i
. Then f (f
1
)
t
= (f
2
)
t
f.
Hints:
1. Theorem 4 follows from the chain rule: If p = (t) and q = f(p)
df
p
_
d
dt
(t)
_
=
d
dt
f((t)) .
2. To deduce Corollary 5 from Theorem 4 note that for p U,
(f
1
)
t
(p) is just the integral curve,
p
(t) of v
1
with initial point
p
(0) =
p.
The notion of f-relatedness can be very succinctly expressed in
terms of the Lie dierentiation operation. For C

(U
2
) let f

be the composition, f, viewed as a C

function on U
1
, i.e., for
p U
1
let f

(p) = (f(p)). Then


(2.1.23) f

L
v
2
= L
v
1
f

.
(To see this note that if f(p) = q then at the point p the right hand
side is
(d)
q
df
p
(v
1
(p))
by the chain rule and by denition the left hand side is
d
q
(v
2
(q)) .
Moreover, by denition
v
2
(q) = df
p
(v
1
(p))
so the two sides are the same.)
Another easy consequence of the chain rule is:
60 Chapter 2. Dierential forms
Theorem 2.1.15. Let U
i
, i = 1, 2, 3, be open subsets of R
n
i
, v
i
a
vector eld on U
i
and f
i
: U
i
U
i+1
, i = 1, 2 a C

-map. Suppose
that, for i = 1, 2, v
i
and v
i+1
are f
i
-related. Then v
1
and v
3
are
f
2
f
1
-related.
In particular, if f
1
and f
2
are dieomorphisms and v = v
1
(f
2
)

(f
1
)

v = (f
2
f
1
)

v .
The results we described above have dual analogues for one-
forms. Namely, let U and V be open subsets of R
n
and R
m
, respec-
tively, and let f : U V be a C

-map. Given a one-form, , on V


one can dene a pull-back one-form, f

, on U by the following
method. For p U let q = f(p). By denition (q) is a linear map
(2.1.24) (q) : T
q
R
m
R
and by composing this map with the linear map
df
p
: T
p
R
n
T
q
R
n
we get a linear map

q
df
p
: T
p
R
n
R,
i.e., an element
q
df
p
of T

p
R
n
.
Denition 2.1.16. The one-form f

is the one-form dened by


the map
p U (
q
df
p
) T

p
R
n
where q = f(p).
Note that if : V R is a C

-function and = d then

q
df
p
= d
q
df
p
= d( f)
p
i.e.,
(2.1.25) f

= d f .
In particular if is a one-form of the form, = d, with
C

(V ), f

is C

. From this it is easy to deduce


Theorem 2.1.17. If is any C

one-form on V , its pull-back, f

,
is C

. (See exercise 1.)


2.1 Vector elds and one-forms 61
Notice also that the pull-back operation on one-forms and the
push-forward operation on vector elds are somewhat dierent in
character. The former is dened for all C

maps, but the latter is


only dened for dieomorphisms.
Exercises.
1. Let U be an open subset of R
n
, V an open subset of R
n
and
f : U V a C
k
map.
(a) Show that for C

(V ) (2.1.25) can be rewritten


f

d = df

. (2.1.25

)
(b) Let be the one-form
=
m

i=1

i
dx
i

i
C

(V )
on V . Show that if f = (f
1
, . . . , f
m
) then
f

=
m

i=1
f

i
df
i
.
(c) Show that if is C

and f is C

, f

is C

.
2. Let v be a complete vector eld on U and f
t
: U U, the one
parameter group of dieomorphisms generated by v. Show that if
C
1
(U)
L
v
=
_
d
dt
f

t

_
t=0
.
3. (a) Let U = R
2
and let v be the vector eld, x
1
/x
2

x
2
/x
1
. Show that the curve
t R (r cos(t +) , r sin(t +))
is the unique integral curve of v passing through the point, (r cos , r sin ),
at t = 0.
62 Chapter 2. Dierential forms
(b) Let U = R
n
and let v be the constant vector eld:

c
i
/x
i
.
Show that the curve
t R a +t(c
1
, . . . , c
n
)
is the unique integral curve of v passing through a R
n
at t = 0.
(c) Let U = R
n
and let v be the vector eld,

x
i
/x
i
. Show that
the curve
t R e
t
(a
1
, . . . , a
n
)
is the unique integral curve of v passing through a at t = 0.
4. Show that the following are one-parameter groups of dieomor-
phisms:
(a) f
t
: R R, f
t
(x) = x +t
(b) f
t
: R R, f
t
(x) = e
t
x
(c) f
t
: R
2
R
2
, f
t
(x, y) = (cos t x sin t y , sin t x + cos t y)
5. Let A : R
n
R
n
be a linear mapping. Show that the series
exp tA = I +tA+
t
2
2!
A
2
+
t
3
3!
A
3
+
converges and denes a one-parameter group of dieomorphisms of
R
n
.
6. (a) What are the innitesimal generators of the one-parameter
groups in exercise 13?
(b) Show that the innitesimal generator of the one-parameter group
in exercise 14 is the vector eld

a
i,j
x
j

x
i
where [a
i,j
] is the dening matrix of A.
7. Let v be the vector eld on R, x
2 d
dx
Show that the curve
x(t) =
a
a at
2.1 Vector elds and one-forms 63
is an integral curve of v with initial point x(0) = a. Conclude that
for a > 0 the curve
x(t) =
a
1 at
, 0 < t <
1
a
is a maximal integral curve. (In particular, conclude that v isnt
complete.)
8. Let U be an open subset of R
n
and v
1
and v
2
vector elds on U.
Show that there is a unique vector eld, w, on U with the property
L
w
= L
v
1
(L
v
2
) L
v
2
(L
v
1
)
for all C

(U).
9. The vector eld w in exercise 8 is called the Lie bracket of
the vector elds v
1
and v
2
and is denoted [v
1
, v
2
]. Verify that Lie
bracket satises the identities
[v
1
, v
2
] = [v
2
, v
1
]
and
[v
1
[v
2
, v
3
]] + [v
2
, [v
3
, v
1
]] + [v
3
, [v
1
, v
2
]] = 0 .
Hint: Prove analogous identities for L
v
1
, L
v
2
and L
v
3
.
10. Let v
1
= /x
i
and v
2
=

g
j
/x
j
. Show that
[v
1
, v
2
] =


x
i
g
i

x
j
.
11. Let v
1
and v
2
be vector elds and f a C

function. Show that


[v
1
, fv
2
] = L
v
1
fv
2
+f[v
1
, v
2
] .
12. Let U and V be open subsets of R
n
and f : U V a dieo-
morphism. If w is a vector eld on V , dene the pull-back, f

w of
w to U to be the vector eld
f

w = (f
1

w) .
Show that if is a C

function on V
f

L
w
= L
f

w
f

.
Hint: (2.1.26).
64 Chapter 2. Dierential forms
13. Let U be an open subset of R
n
and v and w vector elds on U.
Suppose v is the innitesimal generator of a one-parameter group of
dieomorphisms
f
t
: U U , < t < .
Let w
t
= f

t
w. Show that for C

(U)
L
[v,w]
= L
w

where

w =
d
dt
f

t
w|
t=0
.
Hint: Dierentiate the identity
f

t
L
w
= L
wt
f

t

with respect to t and show that at t = 0 the derivative of the left
hand side is
L
v
L
w

by exercise 2 and the derivative of the right hand side is


L
w
+L
w
(L
v
) .
14. Conclude from exercise 13 that
(2.1.26) [v, w] =
d
dt
f

t
w|
t=0
.
15. Let U be an open subset of R
n
and let : [a, b] U, t
(
1
(t), . . . ,
n
(t)) be a C
1
curve. Given =

f
i
dx
i

1
(U), dene
the line integral of over to be the integral
_

=
n

i=1
_
b
a
f
i
((t))
d
i
dt
dt .
Show that if = df for some f C

(U)
_

= f((b)) f((a)) .
In particular conclude that if is a closed curve, i.e., (a) = (b),
this integral is zero.
2.2 k-forms 65
16. Let
=
x
1
dx
2
x
2
dx
1
x
2
1
+x
2
2

1
(R
2
{0}) ,
and let : [0, 2] R
2
{0} be the closed curve, t (cos t, sin t).
Compute the line integral,
_

, and show that its not zero. Conclude


that cant be d of a function, f C

(R
2
{0}).
17. Let f be the function
f(x
1
, x
2
) =
_

_
arctan
x
2
x
1
, x
1
> 0

2
, x
1
= 0 , x
2
> 0
arctan
x
2
x
1
+ , x
1
< 0
where, we recall:

2
< arctan t <

2
. Show that this function is C

and that df is the 1-form, , in the previous exercise. Why doesnt


this contradict what you proved in exercise 16?
2.2 k-forms
One-forms are the bottom tier in a pyramid of objects whose k
th
tier
is the space of k-forms. More explicitly, given p R
n
we can, as in
1.5, form the k
th
exterior powers
(2.2.1)
k
(T

p
R
n
) , k = 1, 2, 3, . . . , n
of the vector space, T

p
R
n
, and since
(2.2.2)
1
(T

p
R
n
) = T

p
R
n
one can think of a one-form as a function which takes its value at p
in the space (2.2.2). This leads to an obvious generalization.
Denition 2.2.1. Let U be an open subset of R
n
. A k-form, , on
U is a function which assigns to each point, p, in U an element (p)
of the space (2.2.1) .
The wedge product operation gives us a way to construct lots of
examples of such objects.
Example 1.
66 Chapter 2. Dierential forms
Let
i
, i = 1, . . . , k be one-forms. Then
1

k
is the k-form
whose value at p is the wedge product
(2.2.3)
1
(p)
k
(p) .
Notice that since
i
(p) is in
1
(T

p
R
n
) the wedge product (2.2.3)
makes sense and is an element of
k
(T

p
R
n
).
Example 2.
Let f
i
, i = 1, . . . , k be a real-valued C

function on U. Letting

i
= df
i
we get from (2.2.3) a k-form
(2.2.4) df
1
df
k
whose value at p is the wedge product
(2.2.5) (df
1
)
p
(df
k
)
p
.
Since (dx
1
)
p
, . . . , (dx
n
)
p
are a basis of T

p
R
n
, the wedge products
(2.2.6) (dx
i
1
)
p
(dx
1
k
)
p
, 1 i
1
< < i
k
n
are a basis of
k
(T

p
). To keep our multi-index notation from getting
out of hand, well denote these basis vectors by (dx
I
)
p
, where I =
(i
1
, . . . , i
k
) and the Is range over multi-indices of length k which
are strictly increasing. Since these wedge products are a basis of

k
(T

p
R
n
) every element of
k
(T

p
R
n
) can be written uniquely as a
sum

c
I
(dx
I
)
p
, c
I
R
and every k-form, , on U can be written uniquely as a sum
(2.2.7) =

f
I
dx
I
where dx
I
is the k-form, dx
i
1
dx
i
k
, and f
I
is a real-valued
function,
f
I
: U R.
Denition 2.2.2. The k-form (2.2.7) is of class C
r
if each of the
f
I
s is in C
r
(U).
Henceforth well assume, unless otherwise stated, that all the k-
forms we consider are of class C

, and well denote the space of


these k-forms by
k
(U).
We will conclude this section by discussing a few simple operations
on k-forms.
2.2 k-forms 67
1. Given a function, f C

(U) and a k-form


k
(U) we dene
f
k
(U) to be the k-form
p U f(p)
p

k
(T

p
R
n
) .
2. Given
i

k
(U), i = 1, 2 we dene
1
+
2

k
(U) to be
the k-form
p U (
1
)
p
+ (
2
)
p

k
(T

p
R
n
) .
(Notice that this sum makes sense since each summand is in
k
(T

p
R
n
).)
3. Given
1

k
1
(U) and
2

k
2
(U) we dene their wedge
product,
1

2

k
1
+k
2
(u) to be the (k
1
+k
2
)-form
p U (
1
)
p
(
2
)
p

k
1
+k
2
(T

p
R
n
) .
We recall that
0
(T

p
R
n
) = R, so a zero-form is an R-valued function
and a zero form of class C

is a C

function, i.e.,

0
(U) = C

(U) .
A fundamental operation on forms is the d-operation which as-
sociates to a function f C

(U) the 1-form df. Its clear from the


identity (2.1.10) that df is a 1-form of class C

, so the d-operation
can be viewed as a map
(2.2.8) d :
0
(U)
1
(U) .
We will show in the next section that an analogue of this map exists
for every
k
(U).
Exercises.
1. Let
2
(R
4
) be the 2-form, dx
1
dx
2
+dx
3
dx
4
. Compute
.
2. Let
i

1
(R
3
), i = 1, 2, 3 be the 1-forms

1
= x
2
dx
3
x
3
dx
2

2
= x
3
dx
1
x
1
dx
3
and

3
= x
1
dx
2
x
2
dx
1
.
Compute
68 Chapter 2. Dierential forms
(a)
1

2
.
(b)
2

3
.
(c)
3

1
.
(d)
1

2

3
.
3. Let U be an open subset of R
n
and f
i
C

(U), i = 1, . . . , n.
Show that
df
1
df
n
= det
_
f
i
x
j
_
dx
1
dx
n
.
4. Let U be an open subset of R
n
. Show that every (n 1)-form,

n1
(U), can be written uniquely as a sum
n

i=1
f
i
dx
1


dx
i
dx
n
where f
i
C

(U) and the cap over dx


i
means that dx
i
is to be
deleted from the product, dx
1
dx
n
.
5. Let =
n

i=1
x
i
dx
i
. Show that there exists an (n 1)-form,

n1
(R
n
{0}) with the property
= dx
1
dx
n
.
6. Let J be the multi-index (j
1
, . . . , j
k
) and let dx
J
= dx
j
1

dx
j
k
. Show that dx
J
= 0 if j
r
= j
s
for some r = s and show that if
the j
r
s are all distinct
dx
J
= (1)

dx
I
where I = (i
1
, . . . , i
k
) is the strictly increasing rearrangement of
(j
1
, . . . , j
k
) and is the permutation
j
1
i
1
, . . . , j
k
i
k
.
7. Let I be a strictly increasing multi-index of length k and J a
strictly increasing multi-index of length . What can one say about
the wedge product dx
I
dx
J
?
2.3 Exterior dierentiation 69
2.3 Exterior dierentiation
Let U be an open subset of R
n
. In this section we are going to dene
an operation
(2.3.1) d :
k
(U)
k+1
(U) .
This operation is called exterior dierentiation and is the fundamen-
tal operation in n-dimensional vector calculus.
For k = 0 we already dened the operation (2.3.1) in 2.1. Before
dening it for the higher ks we list some properties that we will
require to this operation to satisfy.
Property I. For
1
and
2
in
k
(U), d(
1
+
2
) = d
1
+ d
2
.
Property II. For
1

k
(U) and
2

(U)
(2.3.2) d(
1

2
) = d
1

2
+ (1)
k

1
d
2
.
Property III. For
k
(U)
(2.3.3) d(d) = 0 .
Lets point out a few consequences of these properties. First note
that by Property III
(2.3.4) d(df) = 0
for every function, f C

(U). More generally, given k functions,


f
i
C

(U), i = 1, . . . , k, then by combining (2.3.4) with (2.3.2) we


get by induction on k:
(2.3.5) d(df
1
df
k
) = 0 .
Proof. Let = df
2
df
k
. Then by induction on k, d = 0; and
hence by (2.3.2) and (2.3.4)
d(df
1
) = d(d
1
f) + (1) df
1
d = 0 ,
as claimed.)
70 Chapter 2. Dierential forms
In particular, given a multi-index, I = (i
1
, . . . , i
k
) with 1 i
r
n
(2.3.6) d(dx
I
) = d(dx
i
1
dx
i
k
) = 0 .
Recall now that every k-form,
k
(U), can be written uniquely
as a sum
=

f
I
dx
I
, f
I
C

(U)
where the multi-indices, I, are strictly increasing. Thus by (2.3.2)
and (2.3.6)
(2.3.7) d =

df
I
dx
I
.
This shows that if there exists a d with properties IIII, it has to
be given by the formula (2.3.7). Hence all we have to show is that
the operator dened by this formula has these properties. Property I
is obvious. To verify Property II we rst note that for I strictly
increasing (2.3.6) is a special case of (2.3.7). (Take f
I
= 1 and f
J
=
0 for J = I.) Moreover, if I is not strictly increasing it is either
repeating, in which case dx
I
= 0, or non-repeating in which case I

is strictly increasing for some permutation, S


k
, and
(2.3.8) dx
I
= (1)

dx
I
.
Hence (2.3.7) implies (2.3.6) for all multi-indices I. The same argu-
ment shows that for any sum over indices, I, for length k

f
I
dx
I
one has the identity:
(2.3.9) d(

f
I
dx
I
) =

df
I
dx
I
.
(As above we can ignore the repeating Is, since for these Is, dx
I
=
0, and by (2.3.8) we can make the non-repeating Is strictly increas-
ing.)
Suppose now that
1

k
(U) and
2

(U). Writing

1
=

f
I
dx
I
and

2
=

g
J
dx
J
2.3 Exterior dierentiation 71
with f
I
and g
J
in C

(U) we get for the wedge product

1

2
=

f
I
g
J
dx
I
dx
J
(2.3.10)
and by (2.3.9)
d(
1

2
) =

d(f
I
g
J
) dx
I
dx
J
. (2.3.11)
(Notice that if I = (i
1
, , i
k
) and J = (j
i
, . . . , i

), dx
I
dx
J
=
dx
K
, K being the multi-index, (i
1
, . . . , i
k
, j
1
, . . . , j

). Even if I and
J are strictly increasing, K wont necessarily be strictly increasing.
However in deducing (2.3.11) from (2.3.10) weve observed that this
doesnt matter .) Now note that by (2.1.11)
d(f
I
g
J
) = g
J
df
I
+f
I
dg
J
,
and by the wedge product identities of (1.6),
dg
J
dx
I
= dg
J
dx
i
1
dx
i
k
= (1)
k
dx
I
dg
J
,
so the sum (2.3.11) can be rewritten:

df
I
dx
I
g
J
dx
J
+ (1)
k

f
I
dx
I
dg
J
dx
J
,
or
_

df
I
dx
I
_

g
J
dx
J
_
+ (1)
k
_

dg
J
dx
J
_
,
or nally:
d
1

2
+ (1)
k

1
d
2
.
Thus the d dened by (2.3.7) has Property II. Lets now check that
it has Property III. If =

f
I
dx
I
, f
I
C

(U), then by denition,


d =

df
I
dx
I
and by (2.3.6) and (2.3.2)
d(d) =

d(df
I
) dx
I
,
so it suces to check that d(df
I
) = 0, i.e., it suces to check (2.3.4)
for zero forms, f C

(U). However, by (2.1.9)


df =
n

j=1
f
x
j
dx
j
72 Chapter 2. Dierential forms
so by (2.3.7)
d(df) =
n

j=1
d
_
f
x
j
_
dx
j
=
n

j=1
_
n

i=1

2
f
x
i
x
j
dx
i
_
dx
j
=

i,j

2
f
x
i
x
j
dx
i
dx
j
.
Notice, however, that in this sum, dx
i
dx
j
= dx
j
dx
i
and

2
f
x
i
x
j
=

2
f
x
j
x
i
so the (i, j) term cancels the (j, i) term, and the total sum is zero.
A form,
k
(U), is said to be closed if d = 0 and is said to be
exact if = d for some
k1
(U). By Property III every exact
form is closed, but the converse is not true even for 1-forms. (See
2.1, exercise 8). In fact its a very interesting (and hard) question
to determine if an open set, U, has the property: For k > 0 every
closed k-form is exact.
1
Some examples of sets with this property are described in the
exercises at the end of 2.5. We will also sketch below a proof of the
following result (and ask you to ll in the details).
Lemma 2.3.1 (Poincares Lemma.). If is a closed form on U of
degree k > 0, then for every point, p U, there exists a neighborhood
of p on which is exact.
(See exercises 5 and 6 below.)
Exercises:
1. Compute the exterior derivatives of the forms below.
1
For k = 0, df = 0 doesnt imply that f is exact. In fact exactness doesnt make
much sense for zero forms since there arent any 1 forms. However, if f C

(U)
and df = 0 then f is constant on connected components of U. (See 2.1, exercise 2.)
2.3 Exterior dierentiation 73
(a) x
1
dx
2
dx
3
(b) x
1
dx
2
x
2
dx
1
(c) e
f
df where f =

n
i=1
x
2
i
(d)

n
i=1
x
i
dx
i
(e)

n
i=1
(1)
i
x
i
dx
1


dx
i
dx
n
2. Solve the equation: d = for
1
(R
3
), where is the
2-form
(a) dx
2
dx
3
(b) x
2
dx
2
dx
3
(c) (x
2
1
+x
2
2
) dx
1
dx
2
(d) cos x
1
dx
1
dx
3
3. Let U be an open subset of R
n
.
(a) Show that if
k
(U) is exact and

(U) is closed then


is exact. Hint: The formula (2.3.2).
(b) In particular, dx
1
is exact, so if

(U) is closed dx
1
=
d. What is ?
4. Let Q be the rectangle, (a
1
, b
1
) (a
n
, b
n
). Show that if
is in
n
(Q), then is exact.
Hint: Let = f dx
1
dx
n
with f C

(Q) and let g be the


function
g(x
1
, . . . , x
n
) =
_
x
1
a
1
f(t, x
2
, . . . , x
n
) dt .
Show that = d(g dx
2
dx
n
).
5. Let U be an open subset of R
n1
, A R an open interval
and (x, t) product coordinates on U A. We will say that a form,

(U A) is reduced if it can be written as a sum


(2.3.12) =

f
I
(x, t) dx
I
,
(i.e., no terms involving dt).
74 Chapter 2. Dierential forms
(a) Show that every form,
k
(U A) can be written uniquely
as a sum:
(2.3.13) = dt +
where and are reduced.
(b) Let be the reduced form (2.3.12) and let
d
dt
=

d
dt
f
I
(x, t) dx
I
and
d
U
=

I
_
n

i=1

x
i
f
I
(x, t) dx
i
_
dx
I
.
Show that
d = dt
d
dt
+d
U
.
(c) Let be the form (2.3.13). Show that
d = dt d
U
+ dt
d
dt
+ d
U

and conclude that is closed if and only if


d
dt
= d
U
(2.3.14)
d
U
= 0 .
(d) Let be a reduced (k 1)-form. Show that there exists a re-
duced (k 1)-form, , such that
(2.3.15)
d
dt
= .
Hint: Let =

f
I
(x, t) dx
I
and =

g
I
(x, t) dx
I
. The equa-
tion (2.3.15) reduces to the system of equations
(2.3.16)
d
dt
g
I
(x, t) = f
I
(x, t) .
Let c be a point on the interval, A, and using freshman calculus show
that (2.3.16) has a unique solution, g
I
(x, t), with g
I
(x, c) = 0.
2.4 The interior product operation 75
(e) Show that if is the form (2.3.13) and a solution of (2.3.15)
then the form
(2.3.17) d
is reduced.
(f) Let
=

h
I
(x, t) dx)I
be a reduced k-form. Deduce from (2.3.14) that if is closed then
d
dt
= 0 and d
U
= 0. Conclude that h
I
(x, t) = h
I
(x) and that
=

h
I
(x) dx
I
is eectively a closed k-form on U. Now prove: If every closed k-form
on U is exact, then every closed k-form on U A is exact. Hint: Let
be a closed k-form on U A and let be the form (2.3.17).
6. Let Q R
n
be an open rectangle. Show that every closed form
on Q of degree k > 0 is exact. Hint: Let Q = (a
1
, b
1
) (a
n
, b
n
).
Prove this assertion by induction, at the n
th
stage of the induction
letting U = (a
1
, b
1
) (a
n1
, b
n1
) and A = (a
n
, b
n
).
2.4 The interior product operation
In 2.1 we explained how to pair a one-form, , and a vector eld,
v, to get a function, (v). This pairing operation generalizes: If one
is given a k-form, , and a vector eld, v, both dened on an open
subset, U, one can dene a (k 1)-form on U by dening its value
at p U to be the interior product
(2.4.1) (v(p))(p) .
Note that v(p) is in T
p
R
n
and (p) in
k
(T

p
R
n
), so by denition
of interior product (see 1.7), the expression (2.4.1) is an element of

k1
(T

p
R
n
). We will denote by (v) the (k 1)form on U whose
value at p is (2.4.1). From the properties of interior product on vector
spaces which we discussed in 1.7, one gets analogous properties for
this interior product on forms. We will list these properties, leaving
their verication as an exercise. Let v and be vector elds, and
1
76 Chapter 2. Dierential forms
and
2
k-forms, a k-form and an -form. Then (v) is linear in
:
(2.4.2) (v)(
1
+
2
) = (v)
1
+(v)
2
,
linear in v:
(2.4.3) (v +w) = (v) +z(w) ,
has the derivation property:
(2.4.4) (v)( ) = (v) + (1)
k
(v)
satises the identity
(2.4.5) (v)((w)) = (w)((v))
and, as a special case of (2.4.5), the identity,
(2.4.6) (v)((v)) = 0 .
Moreover, if is decomposable i.e., is a wedge product of one-
forms
=
1

k
, (2.4.7)
then
(v) =
k

r=1
(1)
r1
((v)
r
)
1

r

k
. (2.4.8)
We will also leave for you to prove the following two assertions, both
of which are special cases of (2.4.8). If v = /x
r
and = dx
I
=
dx
i
1
dx
i
k
then
(v) =
k

r=1
(1)
r

i
ir
dx
Ir
(2.4.9)
where

i
ir
=
_
1 i = i
r
0 , i = i
r
.
2.4 The interior product operation 77
and I
r
= (i
1
, . . . ,

i
r
, . . . , i
k
) and if v =

f
i
/x
i
and = dx
1

dx
n
then
(2.4.10) (v) =

(1)
r1
f
r
dx
1


dx
r
dx
n
.
By combining exterior dierentiation with the interior product op-
eration one gets another basic operation of vector elds on forms: the
Lie dierentiation operation. For zero-forms, i.e., for C

functions,
, we dened this operation by the formula (2.1.14). For k-forms
well dene it by the slightly more complicated formula
(2.4.11) L
v
= (v) d + d(v) .
(Notice that for zero-forms the second summand is zero, so (2.4.11)
and (2.1.14) agree.) If is a k-form the right hand side of (2.4.11)
is as well, so L
v
takes k-forms to k-forms. It also has the property
(2.4.12) dL
v
= L
v
d
i.e., it commutes with d, and the property
(2.4.13) L
v
( ) = L
v
+ L
v

and from these properties it is fairly easy to get an explicit formula


for L
v
. Namely let be the k-form
=

f
I
dx
I
, f
I
C

(U)
and v the vector eld

g
i
/x
i
, g
i
C

(U) .
By (2.4.13)
L
v
(f
I
dx
I
) = (L
v
f
I
) dx
I
+f
I
(L
v
dx
I
)
and
L
v
dx
I
=
k

r=1
dx
i
1
L
v
dx
ir
dx
i
k
,
and by (2.4.12)
L
v
dx
ir
= dL
v
x
ir
78 Chapter 2. Dierential forms
so to compute L
v
one is reduced to computing L
v
x
ir
and L
v
f
I
.
However by (2.4.13)
L
v
x
ir
= g
ir
and
L
v
f
I
=

g
i
f
I
x
i
.
We will leave the verication of (2.4.12) and (2.4.13) as exercises,
and also ask you to prove (by the method of computation that weve
just sketched) the divergence formula
(2.4.14) L
v
(dx
1
dx
n
) =

_
g
i
x
i
_
dx
1
dx
n
.
Exercises:
1. Verify the assertions (2.4.2)(2.4.7).
2. Show that if is the k-form, dx
I
and v the vector eld, /x
r
,
then (v) is given by (2.4.9).
3. Show that if is the n-form, dx
1
dx
n
, and v the vector
eld,

f
i
/x
i
, (v) is given by (2.4.10).
4. Let U be an open subset of R
n
and v a C

vector eld on U.
Show that for
k
(U)
dL
v
= L
v
d
and

v
L
v
= L
v

v
.
Hint: Deduce the rst of these identities from the identity d(d) = 0
and the second from the identity (v)((v)) = 0 .)
5. Given
i

k
i
(U), i = 1, 2, show that
L
v
(
1

2
) = L
v

1

2
+
1
L
v

2
.
Hint: Plug =
1

2
into (2.4.11) and use (2.3.2) and (2.4.4)to
evaluate the resulting expression.
2.4 The interior product operation 79
6. Let v
1
and v
2
be vector elds on U and let w be their Lie
bracket. Show that for
k
(U)
L
w
= L
v
1
(L
v
2
) L
v
2
(L
v
1
) .
Hint: By denition this is true for zero-forms and by (2.4.12) for
exact one-forms. Now use the fact that every form is a sum of wedge
products of zero-forms and one-forms and the fact that L
v
satises
the product identity (2.4.13).
7. Prove the divergence formula (2.4.14).
8. (a) Let =
k
(R
n
) be the form
=

f
I
(x
1
, . . . , x
n
) dx
I
and v the vector eld, /x
n
. Show that
L
v
=


x
n
f
I
(x
1
, . . . , x
n
) dx
I
.
(b) Suppose (v) = L
v
= 0. Show that only depends on
x
1
, . . . , x
k1
and dx
1
, . . . , dx
k1
, i.e., is eectively a k-form on R
n1
.
(c) Suppose (v) = d = 0. Show that is eectively a closed
k-form on R
n1
.
(d) Use these results to give another proof of the Poincare lemma
for R
n
. Prove by induction on n that every closed form on R
n
is
exact.
Hints:
i. Let be the form in part (a) and let
g
I
(x
1
, . . . , x
n
) =
_
xn
0
f
I
(x
1
, . . . , x
n1
, t) dt .
Show that if =

g
I
dx
I
, then L
v
= .
ii. Conclude that
(*) d(v) = (v) d .
iii. Suppose d = 0. Conclude from (*) and from the formula (2.4.6)
that the form = (v) d satises d = (v) = 0.
iv. By part c, is eectively a closed form on R
n1
, and by induc-
tion, = d. Thus by (*)
= d(v) + d.
80 Chapter 2. Dierential forms
2.5 The pull-back operation on forms
Let U be an open subset of R
n
, V an open subset of R
m
and f :
U V a C

map. Then for p U and q = f(p), the derivative of f


at p
df
p
: T
p
R
n
T
q
R
m
is a linear map, so (as explained in 7 of Chapter 1) one gets from
it a pull-back map
(2.5.1) df

p
:
k
(T

q
R
m
)
k
(T

p
R
n
) .
In particular, let be a k-form on V . Then at q V , takes the
value

q

k
(T

q
R
m
) ,
so we can apply to it the operation (2.5.1), and this gives us an
element:
(2.5.2) df

q

k
(T

p
R
n
) .
In fact we can do this for every point p U, so this gives us a
function,
(2.5.3) p U (df
p
)

q
, q = f(p) .
By the denition of k-form such a function is a k-form on U. We will
denote this k-form by f

and dene it to be the pull-back of by


the map f. A few of its basic properties are described below.
1. Let be a zero-form, i.e., a function, C

(V ). Since

0
(T

p
) =
0
(T

q
) = R
the map (2.5.1) is just the identity map of R onto R when k is equal
to zero. Hence for zero-forms
(2.5.4) (f

)(p) = (q) ,
i.e., f

is just the composite function, f C

(U).
2. Let
1
(V ) be the 1-form, = d. By the chain rule (2.5.2)
unwinds to:
(2.5.5) (df
p
)

d
q
= (d)
q
df
p
= d( f)
p
and hence by (2.5.4)
(2.5.6) f

d = df

.
2.5 The pull-back operation on forms 81
3. If
1
and
2
are in
k
(V ) we get from (2.5.2)
(df
p
)

(
1
+
2
)
q
= (df
p
)

(
1
)
q
+ (df
p
)

(
2
)
q
,
and hence by (2.5.3)
f

(
1
+
2
) = f

1
+f

2
.
4. We observed in 1.7 that the operation (2.5.1) commutes with
wedge-product, hence if
1
is in
k
(V ) and
2
is in

(V )
df

p
(
1
)
q
(
2
)
q
= df

p
(
1
)
q
df

p
(
2
)
q
.
In other words
(2.5.7) f

1

2
= f

1
f

2
.
5. Let W be an open subset of R
k
and g : V W a C

map.
Given a point p U, let q = f(p) and w = g(q). Then the composi-
tion of the map
(df
p
)

:
k
(T

q
)
k
(T

p
)
and the map
(dg
q
)

:
k
(T

w
)
k
(T

q
)
is the map
(dg
q
df
p
)

:
k
(T

w
)
k
(T

p
)
by formula (1.7.4) of Chapter 1. However, by the chain rule
(dg
q
) (df)
p
= d(g f)
p
so this composition is the map
d(g f)

p
:
k
(T

w
)
k
(T

p
) .
Thus if is in
k
(W)
(2.5.8) f

(g

) = (g f)

.
Lets see what the pull-back operation looks like in coordinates.
Using multi-index notation we can express every k-form,
k
(V )
as a sum over multi-indices of length k
(2.5.9) =

I
dx
I
,
82 Chapter 2. Dierential forms
the coecient,
I
, of dx
I
being in C

(V ). Hence by (2.5.4)
f

I
f

(dx
I
)
where f

I
is the function of f. What about f

dx
I
? If I is the
multi-index, (i
1
, . . . , i
k
), then by denition
dx
I
= dx
i
1
dx
i
k
so
d

dx
I
= f

dx
i
f

dx
i
k
by (2.5.7), and by (2.5.6)
f

dx
i
= df

x
i
= df
i
where f
i
is the i
th
coordinate function of the map f. Thus, setting
df
I
= df
i
1
df
i
k
,
we get for each multi-index, I,
(2.5.10) f

dx
I
= df
I
and for the pull-back of the form (2.5.9)
(2.5.11) f

I
df
I
.
We will use this formula to prove that pull-back commutes with
exterior dierentiation:
(2.5.12) d f

= f

d .
To prove this we recall that by (2.2.5), d( df
I
) = 0, hence by (2.2.2)
and (2.5.10)
d f

d f

I
df
I
=

d
I
df

dx
I
= f

d
I
dx
I
= f

d .
2.5 The pull-back operation on forms 83
A special case of formula (2.5.10) will be needed in Chapter 4: Let
U and V be open subsets of R
n
and let = dx
1
dx
n
. Then
by (2.5.10)
f

p
= (df
1
)
p
(df
n
)
p
for all p U. However,
(df
i
)
p
=

f
i
x
j
(p)(dx
j
)
p
and hence by formula (1.7.7) of Chapter 1
f

p
= det
_
f
i
x
j
(p)
_
(dx
1
dx
n
)
p
.
In other words
(2.5.13) f

dx
1
dx
n
= det
_
f
i
x
j
_
dx
1
dx
n
.
We will outline in exercises 4 and 5 below the proof of an important
topological property of the pull-back operation. Let U be an open
subset of R
n
, V an open subset of R
m
, A R an open interval
containing 0 and 1 and f
i
: U V , i = 0, 1, a C

map.
Denition 2.5.1. A C

map, F : U A V , is a homotopy
between f
0
and f
1
if F(x, 0) = f
0
(x) and F(x, 1) = f
1
(x).
Thus, intuitively, f
0
and f
1
are homotopic if there exists a family
of C

maps, f
t
: U V , f
t
(x) = F(x, t), which smoothly deform
f
0
into f
1
. In the exercises mentioned above you will be asked to
verify that for f
0
and f
1
to be homotopic they have to satisfy the
following criteria.
Theorem 2.5.2. If f
0
and f
1
are homotopic then for every closed
form,
k
(V ), f

1
f

0
is exact.
This theorem is closely related to the Poincare lemma, and, in fact,
one gets from it a slightly stronger version of the Poincare lemma
than that described in exercises 56 in 2.2.
Denition 2.5.3. An open subset, U, of R
n
is contractable if, for
some point p
0
U, the identity map
f
1
: U U , f(p) = p ,
84 Chapter 2. Dierential forms
is homotopic to the constant map
f
0
: U U , f
0
(p) = p
0
.
From the theorem above its easy to see that the Poincare lemma
holds for contractable open subsets of R
n
. If U is contractable every
closed k-form on U of degree k > 0 is exact. (Proof: Let be such a
form. Then for the identity map f

0
= and for the constant map,
f

0
= 0.)
Exercises.
1. Let f : R
3
R
3
be the map
f(x
1
, x
2
, x
3
) = (x
1
x
2
, x
2
x
2
3
, x
3
3
) .
Compute the pull-back, f

for
(a) = x
2
dx
3
(b) = x
1
dx
1
dx
3
(c) = x
1
dx
1
dx
2
dx
3
2. Let f : R
2
R
3
be the map
f(x
1
, x
2
) = (x
2
1
, x
2
2
, x
1
x
2
) .
Complete the pull-back, f

, for
(a) = x
2
dx
2
+x
3
dx
3
(b) = x
1
dx
2
dx
3
(c) = dx
1
dx
2
dx
3
3. Let U be an open subset of R
n
, V an open subset of R
m
, f :
U V a C

map and : [a, b] U a C

curve. Show that for



1
(V )
_

=
_

where
1
: [a, b] V is the curve,
1
(t) = f((t)). (See 2.1,
exercise 7.)
2.5 The pull-back operation on forms 85
4. Let U be an open subset of R
n
, A R an open interval con-
taining the points, 0 and 1, and (x, t) product coordinates on U A.
Recall ( 2.2, exercise 5) that a form,

(U A) is reduced if it
can be written as a sum
(2.5.14) =

f
I
(x, t) dx
I
(i.e., none of the summands involve dt). For a reduced form, , let
Q

(U) be the form


(2.5.15) Q =
_

_
1
0
f
I
(x, t) dt
_
dx
I
and let
i

(U), i = 0, 1 be the forms

0
=

f
I
(x, 0) dx
I
(2.5.16)
and

1
=

f
I
(x, 1) dx
I
. (2.5.17)
Now recall that every form,
k
(U A) can be written uniquely
as a sum
(2.5.18) = dt +
where and are reduced. (See exercise 5 of 2.3, part a.)
(a) Prove
Theorem 2.5.4. If the form (2.5.18) is closed then
(2.5.19)
0

1
= dQ.
Hint: Formula (2.3.14).
(b) Let
0
and
1
be the maps of U into U A dened by
0
(x) =
(x, 0) and
1
(x) = (x, 1). Show that (2.5.19) can be rewritten
(2.5.20)

1
= dQ.
5. Let V be an open subset of R
m
and f
i
: U V , i = 0, 1, C

maps. Suppose f
0
and f
1
are homotopic. Show that for every closed
form,
k
(V ), f

1
f

0
is exact. Hint: Let F : U A V be a
86 Chapter 2. Dierential forms
homotopy between f
0
and f
1
and let = F

. Show that is closed


and that f

0
=

0
and f

1
=

1
. Conclude from (2.5.20) that
(2.5.21) f

0
f

1
= dQ
where = dt + and and are reduced.
6. Show that if U R
n
is a contractable open set, then the
Poincare lemma holds: every closed form of degree k > 0 is exact.
7. An open subset, U, of R
n
is said to be star-shaped if there exists
a point p
0
U, with the property that for every point p U, the
line segment,
tp + (1 t)p
0
, 0 t 1 ,
joining p to p
0
is contained in U. Show that if U is star-shaped it is
contractable.
8. Show that the following open sets are star-shaped:
(a) The open unit ball
{x R
n
, x < 1} .
(b) The open rectangle, I
1
I
n
, where each I
k
is an open
subinterval of R.
(c) R
n
itself.
(d) Product sets
U
1
U
2
R
n
= R
n
1
R
n
2
where U
i
is a star-shaped open set in R
n
i
.
9. Let U be an open subset of R
n
, f
t
: U U, t R, a one-
parameter group of dieomorphisms and v its innitesimal generator.
Given
k
(U) show that at t = 0
(2.5.22)
d
dt
f

t
= L
v
.
Here is a sketch of a proof:
2.5 The pull-back operation on forms 87
(a) Let (t) be the curve, (t) = f
t
(p), and let be a zero-form,
i.e., an element of C

(U). Show that


f

t
(p) = ((t))
and by dierentiating this identity at t = 0 conclude that (2.4.40)
holds for zero-forms.
(b) Show that if (2.4.40) holds for it holds for d. Hint: Dier-
entiate the identity
f

t
d = df

t

at t = 0.
(c) Show that if (2.4.40) holds for
1
and
2
it holds for
1

2
.
Hint: Dierentiate the identity
f

t
(
1

2
) = f

t

1
f

t

2
at t = 0.
(d) Deduce (2.4.40) from a, b and c. Hint: Every k-form is a sum
of wedge products of zero-forms and exact one-forms.
10. In exercise 9 show that for all t
(2.5.23)
d
dt
f

t
= f

t
L
v
= L
v
f

t
.
Hint: By the denition of one-parameter group, f
s+t
= f
s
f
t
=
f
r
f
s
, hence:
f

s+t
= f

t
(f

s
) = f

s
(f

t
) .
Prove the rst assertion by dierentiating the rst of these identities
with respect to s and then setting s = 0, and prove the second
assertion by doing the same for the second of these identities.
In particular conclude that
(2.5.24) f

t
L
v
= L
v
f

t
.
11. (a) By massaging the result above show that
d
dt
f

t
= dQ
t
+Q
t
d (2.5.25)
where
Q
t
= f

t
(v) . (2.5.26)
Hint: Formula (2.4.11).
88 Chapter 2. Dierential forms
(b) Let
Q =
_
1
0
f

t
(v) dt .
Prove the homotopy indentity
(2.5.27) f

1
f

0
= dQ +Qd .
12. Let U be an open subset of R
n
, V an open subset of R
m
, v a
vector eld on U, w a vector eld on V and f : U V a C

map.
Show that if v and w are f-related
(v)f

= f

(w) .
Hint: Chapter 1, 1.7, exercise 8.
2.6 Div, curl and grad
The basic operations in 3-dimensional vector calculus: grad, curl and
div are, by denition, operations on vector elds. As well see below
these operations are closely related to the operations
(2.6.1) d :
k
(R
3
)
k+1
(R
3
)
in degrees k = 0, 1, 2. However, only two of these operations: grad
and div, generalize to n dimensions. (They are essentially the d-
operations in degrees zero and n 1.) And, unfortunately, there is
no simple description in terms of vector elds for the other n 2 d-
operations. This is one of the main reasons why an adequate theory
of vector calculus in n-dimensions forces on one the dierential form
approach that weve developed in this chapter. Even in three dimen-
sions, however, there is a good reason for replacing grad, div and curl
by the three operations, (2.6.1). A problem that physicists spend a
lot of time worrying about is the problem of general covariance: for-
mulating the laws of physics in such a way that they admit as large
a set of symmetries as possible, and frequently these formulations
involve dierential forms. An example is Maxwells equations, the
fundamental laws of electromagnetism. These are usually expressed
as identities involving div and curl. However, as well explain below,
there is an alternative formulation of Maxwells equations based on
2.6 Div, curl and grad 89
the operations (2.6.1), and from the point of view of general covari-
ance, this formulation is much more satisfactory: the only symmetries
of R
3
which preserve div and curl are translations and rotations,
whereas the operations (2.6.1) admit all dieomorphisms of R
3
as
symmetries.
To describe how grad, div and curl are related to the opera-
tions (2.6.1) we rst note that there are two ways of converting vector
elds into forms. The rst makes use of the natural inner product,
B(v, w) =

v
i
w
i
, on R
n
. From this inner product one gets by 1.2,
exercise 9 a bijective linear map:
(2.6.2) L : R
n
(R
n
)

with the dening property: L(v) = (w) = B(v, w). Via the
identication (2.1.2) B and L can be transferred to T
p
R
n
, giving one
an inner product, B
p
, on T
p
R
n
and a bijective linear map
(2.6.3) L
p
: T
p
R
n
T

p
R
n
.
Hence if were given a vector eld, v, on U we can convert it into a
1-form, v

, by setting
(2.6.4) v

(p) = L
p
v(p)
and this sets up a oneone correspondence between vector elds and
1-forms. For instance
(2.6.5) v =

x
i
v

= dx
i
,
(see exercise 3 below) and, more generally,
(2.6.6) v =

f
i

x
i
v

f
i
dx
i
.
In particular if f is a C

function on U the vector eld grad f is


by denition
(2.6.7)

f
x
i

x
i
and this gets converted by (2.6.8) into the 1-form, df. Thus the
grad operation in vector calculus is basically just the operation,
d :
0
(U)
1
(U).
90 Chapter 2. Dierential forms
The second way of converting vector elds into forms is via the
interior product operation. Namely let be the n-form, dx
1

dx
n
. Given an open subset, U of R
n
and a C

vector eld,
(2.6.8) v =

f
i

x
i
on U the interior product of v with is the (n 1)-form
(2.6.9) (v) =

(1)
r1
f
r
dx
1


dx
r
dx
n
.
Moreover, every (n1)-form can be written uniquely as such a sum,
so (2.6.8) and (2.6.9) set up a one-one correspondence between vector
elds and (n 1)-forms. Under this correspondence the d-operation
gets converted into an operation on vector elds
(2.6.10) v d(v).
Moreover, by (2.4.11)
d(v) = L
v

and by (2.4.14)
L
v
= div(v)
where
(2.6.11) div(v) =
n

i=1
f
i
x
i
.
In other words, this correspondence between (n1)-forms and vector
elds converts the d-operation into the divergence operation (2.6.11)
on vector elds.
Notice that div and grad are well-dened as vector calculus
operations in n-dimensions even though one usually thinks of them
as operations in 3-dimensional vector calculus. The curl operation,
however, is intrinsically a 3-dimensional vector calculus operation.
To dene it we note that by (2.6.9) every 2-form, , can be written
uniquely as an interior product,
(2.6.12) = (w) dx
1
dx
2
dx
3
,
for some vector eld w, and the left-hand side of this formula de-
termines w uniquely. Now let U be an open subset of R
3
and v a
2.6 Div, curl and grad 91
vector eld on U. From v we get by (2.6.6) a 1-form, v

, and hence
by (2.6.12) a vector eld, w, satisfying
(2.6.13) dv

= (w) dx
1
dx
2
dx
3
.
The curl of v is dened to be this vector eld, in other words,
(2.6.14) curl v = w,
where v and w are related by (2.6.13).
Well leave for you to check that this denition coincides with the
denition one nds in calculus books. More explicitly well leave for
you to check that if v is the vector eld
v = f
1

x
1
+f
2

x
2
+f
3

x
3
(2.6.15)
then
curl v = g
1

x
1
+g
2

x
2
+g
3

x
3
(2.6.16)
where
g
1
=
f
2
x
3

f
3
x
2
g
2
=
f
3
x
1

f
1
x
3
(2.6.17)
g
3
=
f
1
x
2

f
2
x
1
.
To summarize: the grad, curl and div operations in 3-dimensions
are basically just the three operations (2.6.1). The grad operation
is the operation (2.6.1) in degree zero, curl is the operation (2.6.1)
in degree one and div is the operation (2.6.1) in degree two. How-
ever, to dene grad we had to assign an inner product, B
p
, to the
next tangent space, T
p
R
n
, for each p in U; to dene div we had to
equip U with the 3-form, , and to dene curl, the most compli-
cated of these three operations, we needed the B
p
s and . This is
why dieomorphisms preserve the three operations (2.6.1) but dont
preserve grad, curl and div. The additional structures which one
needs to dene grad, curl and div are only preserved by translations
and rotations.
92 Chapter 2. Dierential forms
We will conclude this section by showing how Maxwells equa-
tions, which are usually formulated in terms of div and curl, can be
reset into form language. (The paragraph below is an abbreviated
version of GuilleminSternberg, Symplectic Techniques in Physics,
1.20.)
Maxwells equations assert:
div v
E
= q (2.6.18)
curl v
E
=

t
v
M
(2.6.19)
div v
M
= 0 (2.6.20)
c
2
curl v
M
= w +

t
v
E
(2.6.21)
where v
E
and v
M
are the electric and magnetic elds, q is the scalar
charge density, w is the current density and c is the velocity of light.
(To simplify (2.6.25) slightly well assume that our units of space
time are chosen so that c = 1.) As above let = dx
1
dx
2
dx
3
and let

E
= (v
E
) (2.6.22)
and

M
= (v
M
). (2.6.23)
We can then rewrite equations (2.6.18) and (2.6.20) in the form
(2.6.18

) d
E
= q
and
(2.6.20

) d
M
= 0 .
What about (2.6.19) and (2.6.21)? We will leave the following
form versions of these equations as an exercise.
(2.6.19

) dv

E
=

M
and
(2.6.21

) dv

M
= (w) +

t

E
2.6 Div, curl and grad 93
where the 1-forms, v

E
and v

M
, are obtained from v
E
and v
M
by
the operation, (2.6.4).
These equations can be written more compactly as dierential
form identities in 3 + 1 dimensions. Let
M
and
E
be the 2-forms

M
=
M
v

E
dt (2.6.24)
and

E
=
E
v

M
dt (2.6.25)
and let be the 3-form
(2.6.26) = q +(w) dt .
We will leave for you to show that the four equations (2.6.18)
(2.6.21) are equivalent to two elegant and compact (3+1)-dimensional
identities
d
M
= 0 (2.6.27)
and
d
E
= . (2.6.28)
Exercises.
1. Verify that the curl operation is given in coordinates by the
formula (2.6.17).
2. Verify that the Maxwells equations, (2.6.18) and (2.6.19) be-
come the equations (2.6.20) and (2.6.21) when rewritten in dieren-
tial form notation.
3. Show that in (3 + 1)-dimensions Maxwells equations take the
form (2.6.17)(2.6.18).
4. Let U be an open subset of R
3
and v a vector eld on U. Show
that if v is the gradient of a function, its curl has to be zero.
5. If U is simply connected prove the converse: If the curl of v
vanishes, v is the gradient of a function.
94 Chapter 2. Dierential forms
6. Let w = curl v. Show that the divergence of w is zero.
7. Is the converse statment true? Suppose the divergence of w is
zero. Is w = curl v for some vector eld v?
2.7 Symplectic geometry and classical mechanics
In this section well describe some other applications of the theory
of dierential forms to physics. Before describing these applications,
however, well say a few words about the geometric ideas that are
involved. Let x
1
, . . . , x
2n
be the standard coordinate functions on
R
2n
and for i = 1, . . . , n let y
i
= x
i+n
. The two-form
(2.7.1) =
n

i=1
dx
i
jy
i
is known as the Darboux form. From the identity
(2.7.2) = d
_

y
i
dx
i
_
.
it follows that is exact. Moreover computing the n-fold wedge
product of with itself we get

n
=
_
n

i
i
=1
dx
i
1
dy
i
1
_

_
n

in=1
dx
in
dy
in
_
=

i
1
,...,in
dx
i
1
dy
i
1
dx
in
dy
in
.
We can simplify this sum by noting that if the multi-index, I =
i
1
, . . . , i
n
, is repeating the wedge product
(2.7.3) dx
i
1
dy
i
1
dx
in
dx
in
involves two repeating dx
i
1
s and hence is zero, and if I is non-
repeating we can permute the factors and rewrite (2.7.3) in the form
dx
1
dy
1
dx
n
dy
n
.
(See 1.6, exercise 5.) Hence since these are exactly n! non-repeating
multi-indices

n
= n! dx
1
dy
1
dx
n
dy
n
2.7 Symplectic geometry and classical mechanics 95
i.e.,
1
n!

n
= (2.7.4)
where
= dx
1
dy
1
dx
n
dy
n
(2.7.5)
is the symplectic volume form on R
2n
.
Let U and V be open subsets of R
2n
. A dieomorphism f : U V
is said to be a symplectic dieomorphism (or symplectomorphism for
short) if f

= . In particular let
(2.7.6) f
t
: U U , < t <
be a one-parameter group of dieomorphisms and let v be the vector
eld generating (2.7.6). We will say that v is a symplectic vector eld
if the dieomorphisms, (2.7.6) are symplectomorphisms, i.e., for all t,
(2.7.7) f

t
= .
Lets see what such vector elds have to look like. Note that by
(2.5.23)
(2.7.8)
d
dt
f

t
= f

t
L
v
,
hence if f

t
= for all t, the left hand side of (2.7.8) is zero, so
f

t
L
v
= 0 .
In particular, for t = 0, f
t
is the identity map so f

t
L
v
= L
v
= 0.
Conversely, if L
v
= 0, then f

t
L
v
= 0 so by (2.7.8) f

t
doesnt
depend on t. However, since f

t
= for t = 0 we conclude that
f

t
= for all t. Thus to summarize weve proved
Theorem 2.7.1. Let f
t
: U U be a one-parameter group of dif-
feomorphisms and v the innitesmal generator of this group. Then v
is symplectic of and only if L
v
= 0.
There is an equivalent formulation of this result in terms of the
interior product, (v). By (2.4.11)
L
v
= d(v) +(v) d .
96 Chapter 2. Dierential forms
But by (2.7.2) d = 0 so
L
v
= d(v) .
Thus weve shown
Theorem 2.7.2. The vector eld v is symplectic if and only if (v)
is closed.
If (v) is not only closed but is exact well say that v is a Hamil-
tonian vector eld. In other words v is Hamiltonian if
(2.7.9) (v) = dH
for some C

functions, H C

(U).
Lets see what this condition looks like in coordinates. Let
(2.7.10) v =

f
i

x
i
+g
i

y
i
.
Then
(v) =

i,j
f
i

_

x
i
_
dx
j
dy
j
+

i,j
g
i

_

y
i
_
dx
j
dy
i
.
But

_

x
i
_
dx
j
=
_
1 i = i
0 i = j
and

_

x
i
_
dy
j
= 0
so the rst summand above is

f
i
dy
i
and a similar argument shows that the second summand is

g
i
dx
i
.
2.7 Symplectic geometry and classical mechanics 97
Hence if v is the vector eld (2.7.10)
(2.7.11) (v) =

f
i
dy
i
g
i
dx
i
.
Thus since
dH =

H
x
i
dx
i
+
H
y
i
dy
i
we get from (2.7.9)(2.7.11)
(2.7.12) f
i
=
H
y
i
and g
i
=
H
x
i
so v has the form:
(2.7.13) v =

H
y
i

x
i

H
x
i

y
i
.
In particular if (t) = (x(t) , y(t)) is an integral curve of v it has
to satisfy the system of dierential equations
dx
i
dt
=
H
y
i
(x(t) , y(t)) (2.7.14)
dy
i
dt
=
H
x
i
(x(t) , y(t)) .
The formulas (2.7.10) and (2.7.11) exhibit an important property of
the Darboux form, . Every one-form on U can be written uniquely
as a sum

f
i
dy
i
g
i
dx
i
with f
i
and g
i
in C

(U) and hence (2.7.10) and (2.7.11) imply


Theorem 2.7.3. The map, v (v), sets up a one-one correspon-
dence between vector eld and one-forms.
In particular for every C

function, H, we get by correspondence


a unique vector eld, v = v
H
, with the property (2.7.9).
We next note that by (1.7.6)
L
v
H = (v) dH = (v)((v)) = 0 .
Thus
(2.7.15) L
v
H = 0
98 Chapter 2. Dierential forms
i.e., H is an integral of motion of the vector eld, v. In particular
if the function, H : U R, is proper, then by Theorem 2.1.10 the
vector eld, v, is complete and hence by Theorem 2.7.1 generates a
one-parameter group of symplectomorphisms.
One last comment before we discuss the applications of these re-
sults to classical mechanics. If the one-parameter group (2.7.6) is a
group of symplectomorphisms then f

t

n
= f

t
f

t
=
n
so
by (2.7.4)
(2.7.16) f

t
=
where is the symplectic volume form (2.7.5).
The application we want to make of these ideas concerns the de-
scription, in Newtonian mechanics, of a physical system consisting of
N interacting point-masses. The conguration space of such a system
is
R
n
= R
3
R
3
(N copies)
with position coordinates, x
1
, . . . , x
n
and the phase space is R
2n
with position coordinates x
1
, . . . , x
n
and momentum coordinates,
y
1
, . . . , y
n
. The kinetic energy of this system is a quadratic function
of the momentum coordinates
(2.7.17)
1
2

1
m
i
y
2
i
,
and for simplicity well assume that the potential energy is a func-
tion, V (x
1
, . . . , x
n
), of the position coordinates alone, i.e., it doesnt
depend on the momenta and is time-independent as well. Let
(2.7.18) H =
1
2

1
m
i
y
2
i
+V (x
1
, . . . , x
n
)
be the total energy of the system. Well show below that Newtons
second law of motion in classical mechanics reduces to the assertion:
the trajectories in phase space of the system above are just the integral
curves of the Hamiltonian vector eld, v
H
.
Proof. For the function (2.7.18) the equations (2.7.14) become
dx
i
dt
=
1
m
i
y
i
(2.7.19)
dy
i
dt
=
V
x
i
.
2.7 Symplectic geometry and classical mechanics 99
The rst set of equation are essentially just the denitions of mo-
menta, however, if we plug them into the second set of equations we
get
(2.7.20) m
i
d
2
x
i
dt
2
=
V
x
i
and interpreting the term on the right as the force exerted on the i
th
point-mass and the term on the left as mass times acceleration this
equation becomes Newtons second law.
In classical mechanics the equations (2.7.14) are known as the
HamiltonJacobi equations. For a more detailed account of their role
in classical mechanics we highly recommend Arnolds book, Mathe-
matical Methods of Classical Mechanics. Historically these equations
came up for the rst time, not in Newtonian mechanics, but in gemo-
metric optics and a brief description of their origins there and of their
relation to Maxwells equations can be found in the bookl we cited
above, Symplectic Techniques in Physics.
Well conclude this chapter by mentioning a few implications of
the Hamiltonian description (2.7.14) of Newtons equations (2.7.20).
1. Conservation of energy. By (2.7.15) the energy function (2.7.18)
is constant along the integral curves of v, hence the energy of the
system (2.7.14) doesnt change in time.
2. Noethers principle. Let
t
: R
2n
R
2n
be a one-parameter
group of dieomorphisms of phase space and w its innitesmal gen-
erator. The
t
s are called a symmetry of the system above if
(a) They preserve the function (2.7.18)
and
(b) the vector eld w is Hamiltonian.
The condition (b) means that
(2.7.21) (w) = dG
for some C

function, G, and what Noethers principle asserts is that


this function is an integral of motion of the system (2.7.14), i.e., sat-
ises L
v
G = 0. In other words stated more succinctly: symmetries
of the system (2.7.14) give rise to integrals of motion.
100 Chapter 2. Dierential forms
3. Poincare recurrence. An important theorem of Poincare asserts
that if the function H : R
2n
R dened by (2.7.18) is proper then
every trajectory of the system (2.7.14) returns arbitrarily close to
its initial position at some positive time, t
0
, and, in fact, does this
not just once but does so innitely often. Well sketch a proof of this
theorem, using (2.7.16), in the next chapter.
Exercises.
1. Let v
H
be the vector eld (2.7.13). Prove that div(v
H
) = 0.
2. Let U be an open subset of R
m
, f
t
: U U a one-parameter
group of dieomorphisms of U and v the innitesmal generator of
this group. Show that if is a k-form on U then f

t
= for all t if
and only if L
v
= 0 (i.e., generalize to arbitrary k-forms the result
we proved above for the Darboux form).
3. The harmonic oscillator. Let H be the function

n
i=1
m
i
(x
2
i
+
y
2
i
) where the m
i
s are positive constants.
(a) Compute the integral curves of v
H
.
(b) Poincare recurrence. Show that if (x(t), y(t)) is an integral curve
with initial point (x
0
, y
0
) = (x(0), y(0)) and U an arbitrarily small
neighborhood of (x
0
, y
0
), then for every c > 0 there exists a t > c
such that (x(t), y(t)) U.
4. Let U be an open subset of R
2n
and let H
i
, i = 1, 2, be in
C

(U)
i
. Show that
[v
H
1
, v
H
2
] = v
H
(2.7.22)
where
H =
n

i=1
H
1
x
i
H
2
y
i

H
2
x
i
H
1
y
i
. (2.7.23)
5. The expression (2.7.23) is known as the Poisson bracket of H
1
and H
2
and is denoted by {H
1
, H
2
}. Show that it is anti-symmetric
{H
1
, H
2
} = {H
2
, H
1
}
2.7 Symplectic geometry and classical mechanics 101
and satises Jacobis identity
0 = {H
1
, {H
2
, H
3
}} +{H
2
, {H
3
, H
1
}} +{H
3
, {H
1
, H
2
}} .
6. Show that
(2.7.24) {H
1
, H
2
} = L
v
H
1
H
2
= L
v
H
2
H
1
.
7. Prove that the following three properties are equivalent.
(a) {H
1
, H
2
} = 0.
(b) H
1
is an integral of motion of v
2
.
(c) H
2
is an integral of motion of v
1
.
8. Verify Noethers principle.
9. Conservation of linear momentum. Suppose the potential, V in
(2.7.18) is invariant under the one-parameter group of translations
T
t
(x
1
, . . . , x
n
) = (x
1
+t, . . . , x
n
+t) .
(a) Show that the function (2.7.18) is invariant under the group of
dieomorphisms

t
(x, y) = (T
t
x, y) .
(b) Show that the innitesmal generator of this group is the Hamil-
tonian vector eld v
G
where G =

n
i=1
y
i
.
(c) Conclude from Noethers principle that this function is an in-
tegral of the vector eld v
H
, i.e., that total linear moment is con-
served.
(d) Show that total linear momentum is conserved if V is the
Coulomb potential

i=j
m
i
|x
i
x
j
|
.
10. Let R
i
t
: R
2n
R
2n
be the rotation which xes the variables,
(x
k
, y
k
), k = i and rotates (x
i
, y
i
) by the angle, t:
R
i
t
(x
i
, y
i
) = (cos t x
i
+ sint y
i
, sin t x
i
+ cos t y
i
) .
102 Chapter 2. Dierential forms
(a) Show that R
i
t
, < t < , is a one-parameter group of
symplectomorphisms.
(b) Show that its generator is the Hamiltonian vector eld, v
H
i
,
where H
i
= (x
2
i
+y
2
i
)/2.
(c) Let H be the harmonic oscillator Hamiltonian in exercise 3.
Show that the R
j
t
s preserve H.
(d) What does Noethers principle tell one about the classical me-
chanical system with energy function H?
11. Show that if U is an open subset of R
2n
and v is a symplec-
tic vector eld on U then for every point, p
0
U, there exists a
neighborhood, U
0
, of p
0
on which v is Hamiltonian.
12. Deduce from exercises 4 and 11 that if v
1
and v
2
are symplectic
vector elds on an open subset, U, of R
2n
their Lie bracket, [v
1
, v
2
],
is a Hamiltonian vector eld.
13. Let be the one-form,

n
i=1
y
i
dx
i
.
(a) Show that = d.
(b) Show that if
1
is any one-form on R
2n
with the property,
= d
1
, then
=
1
+F
for some C

function F.
(c) Show that = (w) where w is the vector eld

y
i

y
i
.
14. Let U be an open subset of R
2n
and v a vector eld on U. Show
that v has the property, L
v
= 0, if and only if
(2.7.25) (v) = d(v).
In particular conclude that if L
v
= 0 then v is Hamiltonian. Hint: (2.7.2).
15. Let H be the function
(2.7.26) H(x, y) =

f
i
(x)y
i
,
where the f
i
s are C

functions on R
n
. Show that
(2.7.27) L
v
H
= 0 .
2.7 Symplectic geometry and classical mechanics 103
16. Conversely show that if H is any C

function on R
2n
satisfying
(2.7.27) it has to be a function of the form (2.7.26). Hints:
(a) Let v be a vector eld on R
2n
satisfying L
v
= 0. By the
previous exercise v = v
H
, where H = (v).
(b) Show that H has to satisfy the equation
n

i=1
y
i
H
y
i
= H .
(c) Conclude that if H
r
=
H
yr
then H
r
has to satisfy the equation
n

i=1
y
i

y
i
H
r
= 0 .
(d) Conclude that H
r
has to be constant along the rays (x, ty),
0 t < .
(e) Conclude nally that H
r
has to be a function of x alone, i.e., doesnt
depend on y.
17. Show that if v
R
n is a vector eld

f
i
(x)

x
i
on conguration space there is a unique lift of v
R
n to phase space
v =

f
i
(x)

x
i
+g
i
(x, y)

y
i
satisfying L
v
= 0.
This is page 104
Printer: Opaque this
This is page 105
Printer: Opaque this
CHAPTER 3
INTEGRATION OF FORMS
3.1 Introduction
The change of variables formula asserts that if U and V are open
subsets of R
n
and f : U V a C
1
dieomorphism then, for every
continuous function, : V R the integral
_
V
(y) dy
exists if and only if the integral
_
U
f(x)| det Df(x)| dx
exists, and if these integrals exist they are equal. Proofs of this can
be found in [?], [?] or [?]. This chapter contains an alternative proof
of this result. This proof is due to Peter Lax. Our version of his
proof in 3.5 below makes use of the theory of dierential forms;
but, as Lax shows in the article [?] (which we strongly recommend as
collateral reading for this course), references to dierential forms can
be avoided, and the proof described in3.5 can be couched entirely
in the language of elementary multivariable calculus.
The virtue of Laxs proof is that is allows one to prove a version
of the change of variables theorem for other mappings besides dif-
feomorphisms, and involves a topological invariant, the degree of a
mapping, which is itself quite interesting. Some properties of this in-
variant, and some topological applications of the change of variables
formula will be discussed in 3.6 of these notes.
Remark 3.1.1. The proof we are about to describe is somewhat
simpler and more transparent if we assume that f is a C

dieo-
morphism. Well henceforth make this assumption.
106 Chapter 3. Integration of forms
3.2 The Poincare lemma for compactly supported forms
on rectangles
Let be a k-form on R
n
. We dene the support of to be the closure
of the set
{x R
n
,
x
= 0}
and we say that is compactly supported if supp is compact. We
will denote by
k
c
(R
n
) the set of all C

k-forms which are compactly


supported, and if U is an open subset of R
n
, we will denote by

k
c
(U) the set of all compactly supported k-forms whose support is
contained in U .
Let = f dx
1
dx
n
be a compactly supported n-form with
f C

0
(R
n
). We will dene the integral of over R
n
:
_
R
n

to be the usual integral of f over R


n
_
R
n
f dx.
(Since f is C

and compactly supported this integral is well-dened.)


Now let Q be the rectangle
[a
1
, b
1
] [a
n
, b
n
] .
The Poincare lemma for rectangles asserts:
Theorem 3.2.1. Let be a compactly supported n-form, with supp
Int Q. Then the following assertions are equivalent:
a.
_
= 0.
b. There exists a compactly supported (n1)-form, , with supp
Int Q satisfying d = .
We will rst prove that (b)( a). Let
=
n

i=1
f
i
dx
1
. . .

dx
i
. . . dx
n
,
3.2 The Poincare lemma for compactly supported forms on rectangles 107
(the hat over the dx
i
meaning that dx
i
has to be omitted from the
wedge product). Then
d =
n

i=1
(1)
i1
f
i
x
i
dx
1
. . . dx
n
,
and to show that the integral of d is zero it suces to show that
each of the integrals
(2.1)
i
_
R
n
f
x
i
dx
is zero. By Fubini we can compute (2.1)
i
by rst integrating with
respect to the variable, x
i
, and then with respect to the remaining
variables. But
_
f
x
i
dx
i
= f(x)

x
i
=b
i
x
i
=a
i
= 0
since f
i
is supported on U.
We will prove that (a) (b) by proving a somewhat stronger
result. Let U be an open subset of R
m
. Well say that U has property
P if every form,
m
c
(U) whose integral is zero in d
m1
c
(U).
We will prove
Theorem 3.2.2. Let U be an open subset of R
n1
and A R an
open interval. Then if U has property P, U A does as well.
Remark 3.2.3. Its very easy to see that the open interval A itself
has property P. (See exercise 1 below.) Hence it follows by induction
from Theorem 3.2.2 that
Int Q = A
1
A
n
, A
i
= (a
i
, b
i
)
has property P, and this proves (a) (b).
To prove Theorem 3.2.2 let (x, t) = (x
1
, . . . , x
n1
, t) be product
coordinates on U A. Given
n
c
(U A) we can express
as a wedge product, dt with = f(x, t) dx
1
dx
n1
and
f C

0
(U A). Let
n1
c
(U) be the form
(3.2.1) =
__
A
f(x, t) dt
_
dx
1
dx
n1
.
Then
_
R
n1
=
_
R
n
f(x, t) dxdt =
_
R
n

108 Chapter 3. Integration of forms


so if the integral of is zero, the integral of is zero. Hence since U
has property P, = d for some
n1
c
(U). Let C

(R) be a
bump function which is supported on A and whose integral over A
is one. Setting
= (t) dt
we have
d = (t) dt d = (t) dt ,
and hence
d = dt ( (t)) = dt u(x, t) dx
1
dx
n1
where
u(x, t) = f(x, t) (t)
_
A
f(x, t) dt
by (3.2.1). Thus
(3.2.2)
_
u(x, t) dt = 0 .
Let a and b be the end points of A and let
(3.2.3) v(x, t) =
_
t
a
i(x, s) ds .
By (3.2.2) v(a, x) = v(b, x) = 0, so v is in C

0
(U A) and by (3.2.3),
v/t = u. Hence if we let be the form, v(x, t) dx
1
dx
n1
,
we have:
d = u(x, t) dx dx
n1
= d
and
= d( +) .
Since and are both in
n1
c
(U A) this proves that is in
d
n1
c
(U A) and hence that U A has property P.
Exercises for 3.2.
1. Let f : R R be a compactly supported function of class
C
r
with support on the interval, (a, b). Show that the following are
equivalent.
3.2 The Poincare lemma for compactly supported forms on rectangles 109
(a)
_
b
a
f(x) dx = 0.
(b) There exists a function, g : R R of class C
r+1
with support
on (a, b) with
dg
dx
= f.
Hint: Show that the function
g(x) =
_
x
a
f(s) ds
is compactly supported.
2. Let f = f(x, y) be a compactly supported function on R
k
R

with the property that the partial derivatives


f
x
i
(x, y) , i = 1, . . . , k ,
and are continuous as functions of x and y. Prove the following dif-
ferentiation under the integral sign theorem (which we implicitly
used in our proof of Theorem 3.2.2).
Theorem 3.2.4. The function
g(x) =
_
f(x, y) dy
is of class C
1
and
g
x
i
(x) =
_
f
x
i
(x, y) dy .
Hints: For y xed and h R
k
,
f
i
(x +h, y) f
i
(x, y) = D
x
f
i
(c)h
for some point, c, on the line segment joining x to x + c. Using the
fact that D
x
f is continuous as a function of x and y and compactly
supported, conclude:
Lemma 3.2.5. Given > 0 there exists a > 0 such that for |h|
|f(x +h, y) f(x, y) D
x
f(x, c)h| |h| .
110 Chapter 3. Integration of forms
Now let Q R

be a rectangle with suppf R


k
Q and show
that
|g(x +h) g(x)
__
D
x
f(x, y) dy
_
h| vol (Q)|h| .
Conclude that g is dierentiable at x and that its derivative is
_
D
x
f(x, y) dy .
3. Let f : R
k
R

R be a compactly supported continuous


function. Prove
Theorem 3.2.6. If all the partial derivatives of f(x, y) with respect
to x of order r exist and are continuous as functions of x and y
the function
g(x) =
_
f(x, y) dy
is of class C
r
.
4. Let U be an open subset of R
n1
, A R an open interval
and (x, t) product coordinates on U A. Recall (2.2) exercise 5)
that every form,
k
(U A), can be written uniquely as a sum,
= dt + where and are reduced, i.e., dont contain a factor
of dt.
(a) Show that if is compactly supported on U A then so are
and .
(b) Let =

I
f
I
(x, t) dx
I
. Show that the form
(3.2.4) =

I
__
A
f
I
(x, t) dt
_
dx
I
is in
k1
c
(U).
(c) Show that if d = 0, then d = 0. Hint: By (3.2.4)
d =

I,i
__
A
f
I
x
i
(x, t) dt
_
dx
i
dx
I
=
_
A
(d
U
) dt
and by (??) d
U
=
d
dt
.
3.2 The Poincare lemma for compactly supported forms on rectangles 111
5. In exercise 4 show that if is in d
k1
(U) then is in d
k
c
(U).
Hints:
(a) Let = d, with =
k2
c
(U) and let C

(R) be a bump
function which is supported on A and whose integral over A is
one. Setting k = (t) dt show that
d = dt ( (t)) +
= dt (

I
u
I
(x, t) dx
I
) +
where
u
I
(x, t) = f
I
(x, t) (t)
_
A
f
I
(x, t) dt .
(b) Let a and b be the end points of A and let
v
I
(x, t) =
_
t
a
u
I
(x, t) dt .
Show that the form

v
I
(x, t) dx
I
is in
k1
c
(U A) and that
d = d d
U
.
(c) Conclude that the form d( +) is reduced.
(d) Prove: If
k
c
(U A) is reduced and d = 0 then = 0.
Hint: Let =

g
I
(x, t) dx
I
. Show that d = 0

t
g
I
(x, t) = 0
and exploit the fact that for xed x, g
I
(x, t) is compactly sup-
ported in t.
6. Let U be an open subset of R
m
. Well say that U has property
P
k
, for k < n, if every closed k-form,
k
c
(U), is in d
k1
c
(U).
Prove that if the open set U R
n1
in exercise 3 has property P
k
then so does U A.
7. Show that if Q is the rectangle [a
1
, b
1
] [a
n
, b
n
] and U =
Int Q then u has property P
k
.
8. Let H
n
be the half-space
(3.2.5) {(x
1
, . . . , x
n
) ; x
1
0}
112 Chapter 3. Integration of forms
and let
n
c
(R) be the n-form, f dx
1
dx
n
with f C

0
(R
n
).
Dene:
(3.2.6)
_
H
n
=
_
H
n
f(x
1
, . . . , x
n
) dx
1
dx
n
where the right hand side is the usual Riemann integral of f over
H
n
. (This integral makes sense since f is compactly supported.) Show
that if = d for some
n1
c
(R
n
) then
(3.2.7)
_
H
n
=
_
R
n1

where : R
n1
R
n
is the inclusion map
(x
2
, . . . , x
n
) (0, x
2
, . . . , x
n
) .
Hint: Let =

i
f
i
dx
1


dx
i
dx
n
. Mimicking the (b)
(a) part of the proof of Theorem 3.2.1 show that the integral (3.2.6)
is the integral over R
n1
of the function
_
0

f
1
x
1
(x
1
, x
2
, . . . , x
n
) dx
1
.
3.3 The Poincare lemma for compactly supported forms
on open subsets of R
n
In this section we will generalize Theorem 3.2.1 to arbitrary con-
nected open subsets of R
n
.
Theorem 3.3.1. Let U be a connected open subset of R
n
and let
be a compactly supported n-form with supp U. The the following
assertions are equivalent,
a.
_
= 0.
b. There exists a compactly supported (n1)-form, , with supp
U and = d.
Proof that (b) (a). The support of is contained in a large
rectangle, so the integral of d is zero by Theorem 3.2.1.
3.3 The Poincare lemma for compactly supported forms on open subsets of R
n
113
Proof that (a) (b): Let
1
and
2
be compactly supported n-
forms with support in U. We will write

1

2
as shorthand notation for the statement: There exists a compactly
supported (n1)-form, , with support in U and with
1

2
= d.,
We will prove that (a) (b) by proving an equivalent statement:
Fix a rectangle, Q
0
U and an n-form,
0
, with supp
0
Q
0
and
integral equal to one.
Theorem 3.3.2. If is a compactly supported n-form with supp
U and c =
_
then c
0
.
Thus in particular if c = 0, Theorem 3.3.2 says that 0 proving
that (a) (b).
To prove Theorem 3.3.2 let Q
i
U, i = 1, 2, 3, . . ., be a collection
of rectangles with U = Int Q
i
and let
i
be a partition of unity
with supp
i
Int Q
i
. Replacing by the nite sum

m
i=1

i
, m
large, it suces to prove Theorem 3.3.2 for each of the summands

i
. In other words we can assume that supp is contained in one
of the open rectangles, Int Q
i
. Denote this rectangle by Q. We claim
that one can join Q
0
to Q by a sequence of rectangles as in the gure
below.
Q
0
Q
Lemma 3.3.3. There exists a sequence of rectangles, R
i
, i = 0, . . . ,
N + 1 such that R
0
= Q
0
, R
N+1
= Q and Int R
i
Int R
i+1
is non-
empty.
Proof. Denote by A the set of points, x U, for which there exists a
sequence of rectangles, R
i
, i = 0, . . . , N +1 with R
0
= Q
0
, with x
Int R
N+1
and with Int R
i
Int R
i+1
non-empty. It is clear that this
114 Chapter 3. Integration of forms
set is open and that its complement is open; so, by the connectivity
of U, U = A.
To prove Theorem 3.3.2 with supp Q, select, for each i, a
compactly supported n-form,
i
, with supp
i
Int R
i
Int R
i+1
and with
_

i
= 1. The dierence,
i

i+1
is supported in Int R
i+1
,
and its integral is zero; so by Theorem 3.2.1,
i

i+1
. Similarly,

0

1
and, if c =
_
, c
N
. Thus
c
0
c
0
c
N
=
proving the theorem.
3.4 The degree of a dierentiable mapping
Let U and V be open subsets of R
n
and R
k
. A continuous mapping,
f : U V , is proper if, for every compact subset, B, of V , f
1
(B) is
compact. Proper mappings have a number of nice properties which
will be investigated in the exercises below. One obvious property
is that if f is a C

mapping and is a compactly supported k-


form with support on V , f

is a compactly supported k-form with


support on U. Our goal in this section is to show that if U and V
are connected open subsets of R
n
and f : U V is a proper C

mapping then there exists a topological invariant of f, which we


will call its degree (and denote by deg(f)), such that the change of
variables formula:
(3.4.1)
_
U
f

= deg(f)
_
V

holds for all


n
c
(V ).
Before we prove this assertion lets see what this formula says in
coordinates. If
= (y) dy
1
dy
n
then at x U
f

= ( f)(x) det(Df(x)) dx
1
dx
n
;
so, in coordinates, (3.4.1) takes the form
(3.4.2)
_
V
(y) dy = deg(f)
_
U
f(x) det(Df(x)) dx.
3.4 The degree of a dierentiable mapping 115
Proof of 3.4.1. Let
0
be an n-form of compact support with supp
0
V and with
_

0
= 1. If we set deg f =
_
U
f

0
then (3.4.1) clearly
holds for
0
. We will prove that (3.4.1) holds for every compactly
supported n-form, , with supp V . Let c =
_
V
. Then by
Theorem 3.1 c
0
= d, where is a completely supported (n1)-
form with supp V . Hence
f

cf

0
= f

d = d f

,
and by part (a) of Theorem 3.1
_
U
f

= c
_
f

0
= deg(f)
_
V
.
We will show in 3.6 that the degree of f is always an integer
and explain why it is a topological invariant of f. For the moment,
however, well content ourselves with pointing out a simple but useful
property of this invariant. Let U, V and W be connected open subsets
of R
n
and f : U V and g : V W proper C

mappings. Then
(3.4.3) deg(g f) = deg(g) deg(f) .
Proof. Let be a compactly supported n-form with support on W.
Then
(g f)

= g

;
so
_
U
(g f)

=
_
U
g

(f

) = deg(g)
_
V
f

= deg(g) deg(f)
_
W
.
From this multiplicative property it is easy to deduce the following
result (which we will need in the next section).
Theorem 3.4.1. Let A be a non-singular n n matrix and f
A
:
R
n
R
n
the linear mapping associated with A. Then deg(f
A
) = +1
if det A is positive and 1 if det A is negative.
A proof of this result is outlined in exercises 59 below.
116 Chapter 3. Integration of forms
Exercises for 3.4.
1. Let U be an open subset of R
n
and
i
, i = 1, 2, 3, . . ., a partition
of unity on U. Show that the mapping, f : U R dened by
f =

k=1
k
k
is a proper C

mapping.
2. Let U and V be open subsets of R
n
and R
k
and let f : U V
be a proper continuous mapping. Prove:
Theorem 3.4.2. If B is a compact subset of V and A = f
1
(B)
then for every open subset, U
0
, with A U
0
U, there exists an
open subset, V
0
, with B V
0
V and f
1
(V
0
) U
0
.
Hint: Let C be a compact subset of V with B Int C. Then the
set, W = f
1
(C) U
0
is compact; so its image, f(W), is compact.
Show that f(W) and B are disjoint and let
V
0
= Int C f(W) .
3. Show that if f : U V is a proper continuous mapping and X
is a closed subset of U, f(X) is closed.
Hint: Let U
0
= U X. Show that if p is in V f(X), f
1
(p) is
contained in U
0
and conclude from the previous exercise that there
exists a neighborhood, V
0
, of p such that f
1
(V
0
) is contained in U
0
.
Conclude that V
0
and f(X) are disjoint.
4. Let f : R
n
R
n
be the translation, f(x) = x + a. Show that
deg(f) = 1.
Hint: Let : R R be a compactly supported C

function. For
a R, the identity
(3.4.4)
_
(t) dt =
_
(t a) dt
is easy to prove by elementary calculus, and this identity proves the
assertion above in dimension one. Now let
(3.4.5) (x) = (x
1
) . . . (x
n
)
and compute the right and left sides of (3.4.2) by Fubinis theorem.
3.4 The degree of a dierentiable mapping 117
5. Let be a permutation of the numbers, 1, . . . , n and let f

:
R
n
R
n
be the dieomorphism, f

(x
1
, . . . , x
n
) = (x
(1)
, . . . , x
(n)
).
Prove that deg f

= sgn().
Hint: Let be the function (3.4.5). Show that if is equal to
(x) dx
1
dx
n
, f

= (sgn ).
6. Let f : R
n
R
n
be the mapping
f(x
1
, . . . , x
n
) = (x
1
+x
2
, x
2
, . . . , x
n
).
Prove that deg(f) = 1.
Hint: Let = (x
1
, . . . , x
n
) dx
1
. . . dx
n
where : R
n
R is
compactly supported and of class C

. Show that
_
f

=
_
(x
1
+x
2
, x
2
, . . . , x
n
) dx
1
. . . dx
n
and evaluate the integral on the right by Fubinis theorem; i.e., by
rst integrating with respect to the x
1
variable and then with respect
to the remaining variables. Note that by (3.4.4)
_
f(x
1
+x
2
, x
2
, . . . , x
n
) dx
1
=
_
f(x
1
, x
2
, . . . , x
n
) dx
1
.
7. Let f : R
n
R
n
be the mapping
f(x
1
, . . . , x
n
) = (x
1
, x
2
, . . . , x
n
)
with = 0. Show that deg f = +1 if is positive and 1 if is
negative.
Hint: In dimension 1 this is easy to prove by elementary calculus
techniques. Prove it in d-dimensions by the same trick as in the
previous exercise.
8. (a) Let e
1
, . . . , e
n
be the standard basis vectors of R
n
and A,
B and C the linear mappings
Ae
1
= e, Ae
i
=

j
a
j,i
e
j
, i > 1
Be
i
= e
i
, i > 1 , Be
1
=
n

j=1
b
j
e
j
(3.4.6)
Ce
1
= e
1
, Ce
i
= e
i
+c
i
e
1
, i > 1 .
118 Chapter 3. Integration of forms
Show that
BACe
1
=

b
j
e
j
and
BACe
i
=
n

j
= (a
j,i
+c
i
b
j
)e
j
+c
i
b
1
e
1
for i > 1.
(b)
(3.4.7) Le
i
=
n

j=1

j,i
e
j
, i = 1, . . . , n.
Show that if
1,1
= 0 one can write L as a product, L = BAC, where
A, B and C are linear mappings of the form (3.4.6).
Hint: First solve the equations

j,1
= b
j
for j = 1, . . . , n, then the equations

1,i
= b
1
c
i
for i > 1, then the equations

j,i
= a
j,i
+c
i
b
j
for i, j > 1.
(c) Suppose L is invertible. Conclude that A, B and C are invertible
and verify that Theorem 3.4.1 holds for B and C using the previous
exercises in this section.
(d) Show by an inductive argument that Theorem 3.4.1 holds for
A and conclude from (3.4.3) that it holds for L.
9. To show that Theorem 3.4.1 holds for an arbitrary linear map-
ping, L, of the form (3.4.7) well need to eliminate the assumption:

1,1
= 0. Show that for some j,
j,1
is non-zero, and show how to
eliminate this assumption by considering f

L where is the trans-


position, 1 j.
3.5 The change of variables formula 119
10. Here is an alternative proof of Theorem 4.3.1 which is shorter
than the proof outlined in exercise 9 but uses some slightly more
sophisticated linear algebra.
(a) Prove Theorem 3.4.1 for linear mappings which are orthogonal,
i.e., satisfy L
t
L = I.
Hints:
i. Show that L

(x
2
1
+ +x
2
n
) = x
2
1
+ +x
2
n
.
ii. Show that L

(dx
1
dx
n
) is equal to dx
1
dx
n
or
dx
1
dx
n
depending on whether L is orientation preserving
or orinetation reversing. (See 1.2, exercise 10.)
iii. Let be as in exercise 4 and let be the form
= (x
2
1
+ +x
2
n
) dx
1
dx
n
.
Show that L

= if L is orientation preserving and L

= if
L is orientation reversing.
(b) Prove Theorem 3.4.1 for linear mappings which are self-adjoint
(satisfy L
t
= L). Hint: A self-adjoint linear mapping is diagonizable:
there exists an intervertible linear mapping, M : R
n
R
n
such that
(3.4.8) M
1
LMe
i
=
i
e
i
, i = 1, . . . , n.
(c) Prove that every invertible linear mapping, L, can be written
as a product, L = BC where B is orthogonal and C is self-adjoint.
Hints:
i. Show that the mapping, A = L
t
L, is self-adjoint and that its
eigenvalues, the
i
s in 3.4.8, are positive.
ii. Show that there exists an invertible self-adjoint linear mapping,
C, such that A = C
2
and AC = CA.
iii. Show that the mapping B = LC
1
is orthogonal.
3.5 The change of variables formula
Let U and V be connected open subsets of R
n
. If f : U V is a
dieomorphism, the determinant of Df(x) at x U is non-zero, and
hence, since it is a continuous function of x, its sign is the same at
every point. We will say that f is orientation preserving if this sign
is positive and orientation reversing if it is negative. We will prove
below:
120 Chapter 3. Integration of forms
Theorem 3.5.1. The degree of f is +1 if f is orientation preserving
and 1 if f is orientation reversing.
We will then use this result to prove the following change of vari-
ables formula for dieomorphisms.
Theorem 3.5.2. Let : V R be a compactly supported continu-
ous function. Then
(3.5.1)
_
U
f(x)| det(Df)(x)| =
_
V
(y) dy .
Proof of Theorem 3.5.1. Given a point, a
1
U, let a
2
= f(a
1
) and
for i = 1, 2, let g
i
: R
n
R
n
be the translation, g
i
(x) = x + a
i
. By
(3.4.1) and exercise 4 of 4 the composite dieomorphism
(3.5.2) g
2
f g
1
has the same degree as f, so it suces to prove the theorem for this
mapping. Notice however that this mapping maps the origin onto
the origin. Hence, replacing f by this mapping, we can, without loss
of generality, assume that 0 is in the domain of f and that f(0) = 0.
Next notice that if A : R
n
R
n
is a bijective linear mapping the
theorem is true for A (by exercise 9 of 3.4), and hence if we can
prove the theorem for A
1
f, (3.4.1) will tell us that the theorem
is true for f. In particular, letting A = Df(0), we have
D(A
1
f)(0) = A
1
Df(0) = I
where I is the identity mapping. Therefore, replacing f by A
1
f,
we can assume that the mapping, f, for which we are attempting to
prove Theorem 3.5.1 has the properties: f(0) = 0 and Df(0) = I.
Let g(x) = f(x) x. Then these properties imply that g(0) = 0 and
Dg(0) = 0.
Lemma 3.5.3. There exists a > 0 such that |g(x)|
1
2
|x| for
|x| .
Proof. Let g(x) = (g
1
(x), . . . , g
n
(x)). Then
g
i
x
j
(0) = 0 ;
3.5 The change of variables formula 121
so there exists a > 0 such that

g
i
x
j
(x)

1
2
for |x| . However, by the mean value theorem,
g
i
(x) =

g
i
x
j
(c)x
j
for c = t
0
x, 0 < t
0
< 1. Thus, for |x| < ,
|g
i
(x)|
1
2
sup|x
i
| =
1
2
|x| ,
so
|g(x)| = sup|g
i
(x)|
1
2
|x| .
Let be a compactly supported C

function with 0 1
and with (x) = 0 for |x| and (x) = 1 for |x|

2
and let

f : R
n
R
n
be the mapping
(3.5.3)

f(x) = x +(x)g(x) .
Its clear that
(3.5.4)

f(x) = x for |x|
and, since f(x) = x +g(x),
(3.5.5)

f(x) = f(x) for |x|

2
.
In addition, for all x R
n
:
(3.5.6) |

f(x)|
1
2
|x| .
Indeed, by (3.5.4), |

f(x)| |x| for |x| , and for |x|


|

f(x)| |x| (x)|g(x)|


|x| |g(x)| |x|
1
2
|x| =
1
2
|x|
122 Chapter 3. Integration of forms
by Lemma 3.5.3.
Now let Q
r
be the cube, {x R
n
, |x| r}, and let Q
c
r
= R
n
Q
r
.
From (3.5.6) we easily deduce that
(3.5.7)

f
1
(Q
r
) Q
2r
for all r, and hence that

f is proper. Also notice that for x Q

,
|

f(x)| |x| +|g(x)|


3
2
|x|
by Lemma 3.5.3 and hence
(3.5.8)

f
1
(Q
c
3
2

) Q
c

.
We will now prove Theorem 3.5.1. Since f is a dieomorphism
mapping 0 to 0, it maps a neighborhood, U
0
, of 0 in U dieomor-
phically onto a neighborhood, V
0
, of 0 in V , and by shrinking U
0
if
necessary we can assume that U
0
is contained in Q
/2
and V
0
con-
tained in Q
/4
. Let be an n-form with support in V
0
whose integral
over R
n
is equal to one. Then f

is supported in U
0
and hence in
Q
/2
. Also by (3.5.7)

f

is supported in Q
/2
. Thus both of these
forms are zero outside Q
/2
. However, on Q
/2
,

f = f by (3.5.5), so
these forms are equal everywhere, and hence
deg(f) =
_
f

=
_

f

= deg(

f) .
Next let be a compactly supported n-form with support in Q
c
3/2
and with integral equal to one. Then

f

is supported in Q
c

by
(3.5.8), and hence since f(x) = x on Q
c

= . Thus
deg(

f) =
_
f

=
_
= 1 .
Putting these two identities together we conclude that deg(f) = 1.
Q.E.D.
If the function, , in Theorem 3.5.2 is a C

function, the iden-


tity (3.5.1) is an immediate consequence of the result above and the
identity (3.4.2). If is not C

, but is just continuous, we will deduce


Theorem 3.5.2 from the following result.
3.5 The change of variables formula 123
Theorem 3.5.4. Let V be an open subset of R
n
. If : R
n
R is
a continuous function of compact support with supp V ; then for
every > 0 there exists a C

function of compact support, : R


n

R with supp V and


sup|(x) (x)| < .
Proof. Let A be the support of and let d be the distance in the
sup norm from A to the complement of V . Since is continuous and
compactly supported it is uniformly continuous; so for every > 0
there exists a > 0 with <
d
2
such that |(x) (y)| < when
|x y| . Now let Q be the cube: |x| < and let : R
n
R be a
non-negative C

function with supp Q and


(3.5.9)
_
(y) dy = 1 .
Set
(x) =
_
(y x)(y) dy .
By Theorem 3.2.5 is a C

function. Moreover, if A

is the set of
points in R
d
whose distance in the sup norm from A is then for
x / A

and y A, |x y| > and hence (y x) = 0. Thus for


x / A

_
(y x)(y) dy =
_
A
(y x)(y) dy = 0 ,
so is supported on the compact set A

. Moreover, since <


d
2
,
supp is contained in V . Finally note that by (3.5.9) and exercise 4
of 3.4:
(3.5.10)
_
(y x) dy =
_
(y) dy = 1
and hence
(x) =
_
(x)(y x) dy
so
(x) (x) =
_
((x) (y))(y x) dy
and
124 Chapter 3. Integration of forms
|(x) (x)|
_
|(x) (y)| (y x) dy .
But (yx) = 0 for |xy| ; and |(x)(y)| < for |xy| ,
so the integrand on the right is less than

_
(y x) dy ,
and hence by (3.5.10)
|(x) (x)| .
To prove the identity (3.5.1), let : R
n
R be a C

cut-o
function which is one on a neighborhood, V
1
, of the support of , is
non-negative, and is compactly supported with supp V , and let
c =
_
(y) dy .
By Theorem 3.5.4 there exists, for every > 0, a C

function ,
with support on V
1
satisfying
(3.5.11) | |

2c
.
Thus

_
V
( )(y) dy


_
V
| |(y) dy

_
V
| |(xy) dy


2c
_
(y) dy

2
so
(3.5.12)

_
V
(y) dy
_
V
(y) dy



2
.
Similarly, the expression

_
U
( ) f(x)| det Df(x)| dx

3.5 The change of variables formula 125


is less than or equal to the integral
_
U
f(x)|( ) f(x)| | det Df(x)| dx
and by (3.5.11), |( ) f(x)|

2c
, so this integral is less than or
equal to

2c
_
f(x)| det Df(x)| dx
and hence by (3.5.1) is less than or equal to

2
. Thus
(3.5.13)

_
U
f(x) | det Df(x)|dx
_
U
f(x)| det Df(x)| dx


2
.
Combining (3.5.12), (3.5.13) and the identity
_
V
(y) dy =
_
f(x)| det Df(x)| dx
we get, for all > 0,

_
V
(y) dy
_
U
f(x)| det Df(x)| dx


and hence
_
(y) dy =
_
f(x)| det Df(x)| dx.
Exercises for 3.5
1. Let h : V R be a non-negative continuous function. Show
that if the improper integral
_
V
h(y) dy
is well-dened, then the improper integral
_
U
h f(x)| det Df(x)| dx
is well-dened and these two integrals are equal.
126 Chapter 3. Integration of forms
Hint: If
i
, i = 1, 2, 3, . . . is a partition of unity on V then
i
=

i
f is a partition of unity on U and
_

i
hdy =
_

i
(h f(x))| det Df(x)| dx.
Now sum both sides of this identity over i.
2. Show that the result above is true without the assumption that
h is non-negative.
Hint: h = h
+
h

, where h
+
= max(h, 0) and h

= max(h, 0).
3. Show that, in the formula (3.4.2), one can allow the function,
, to be a continuous compactly supported function rather than a
C

compactly supported function.


4. Let H
n
be the half-space (??) and U and V open subsets of
R
n
. Suppose f : U V is an orientation preserving dieomorphism
mapping U H
n
onto V H
n
. Show that for
n
c
(V )
(3.5.14)
_
UH
n
f

=
_
V H
n
.
Hint: Interpret the left and right hand sides of this formula as im-
proper integrals over U Int H
n
and V Int H
n
.
5. The boundary of H
n
is the set
bH
n
= {(0, x
2
, . . . , x
n
) , (x
2
, . . . , x
n
) R
n
}
so the map
: R
n1
H
n
, (x
2
, . . . , x
n
) (0, x
2
, . . . , x
n
)
in exercise 9 in 3.2 maps R
n1
bijectively onto bH
n
.
(a) Show that the map f : U V in exercise 4 maps U bH
n
onto V bH
n
.
(b) Let U

=
1
(U) and V

=
1
(V ). Conclude from part (a)
that the restriction of f to U bH
n
gives one a dieomorphism
g : U

V

satisfying:
(3.5.15) g = f .
3.6 Techniques for computing the degree of a mapping 127
(c) Let be in
n1
c
(V ). Conclude from (3.2.7) and (3.5.14):
(3.5.16)
_
U

=
_
V

and in particular show that the dieomorphism, g : U

,
is orientation preserving.
3.6 Techniques for computing the degree of a mapping
Let U and V be open subsets of R
n
and f : U V a proper C

mapping. In this section we will show how to compute the degree


of f and, in particular, show that it is always an integer. From this
fact we will be able to conclude that the degree of f is a topological
invariant of f: if we deform f smoothly, its degree doesnt change.
Denition 3.6.1. A point, x U, is a critical point of f if the
derivative
Df(x) : R
n
R
n
fails to be bijective, i.e., if det(Df(x)) = 0.
We will denote the set of critical points of f by C
f
. Its clear from
the denition that this set is a closed subset of U and hence, by
exercise 3 in 3.4, f(C
f
) is a closed subset of V . We will call this
image the set of critical values of f and the complement of this image
the set of regular values of f. Notice that V f(U) is contained in
f f(C
f
), so if a point, g V is not in the image of f, its a
regular value of f by default, i.e., it contains no points of U in
the pre-image and hence, a fortiori, contains no critical points in its
pre-image. Notice also that C
f
can be quite large. For instance, if c is
a point in V and f : U V is the constant map which maps all of U
onto c, then C
f
= U. However, in this example, f(C
f
) = {c}, so the
set of regular values of f is V {c}, and hence (in this example) is
an open dense subset of V . We will show that this is true in general.
Theorem 3.6.2. (Sards theorem.)
If U and V are open subsets of R
n
and f : U V a proper C

map, the set of regular values of f is an open dense subset of V .


We will defer the proof of this to Section 3.7 and, in this section,
explore some of its implications. Picking a regular value, q, of f we
will prove:
128 Chapter 3. Integration of forms
Theorem 3.6.3. The set, f
1
(q) is a nite set. Moreover, if f
1
(q) =
{p
1
, . . . , p
n
} there exist connected open neighborhoods, U
i
, of p
i
in Y
and an open neighborhood, W, of q in V such that:
i. for i = j U
i
and U
j
are disjoint;
ii. f
1
(W) =

U
i
,
iii. f maps U
i
dieomorphically onto W.
Proof. If p f
1
(q) then, since q is a regular value, p / C
f
; so
Df(p) : R
n
R
n
is bijective. Hence by the inverse function theorem, f maps a neigh-
borhood, U
p
of p dieomorphically onto a neighborhood of q. The
open sets
{U
p
, p f
1
(q)}
are a covering of f
1
(q); and, since f is proper, f
1
(q) is compact;
so we can extract a nite subcovering
{U
p
i
, i = 1, . . . , N}
and since p
i
is the only point in U
p
i
which maps onto q, f
1
(q) =
{p
1
, . . . , p
N
}.
Without loss of generality we can assume that the U
p
i
s are disjoint
from each other; for, if not, we can replace them by smaller neighbor-
hoods of the p
i
s which have this property. By Theorem 3.4.2 there
exists a connected open neighborhood, W, of q in V for which
f
1
(W)
_
U
p
i
.
To conclude the proof let U
i
= f
1
(W) U
p
i
.
The main result of this section is a recipe for computing the de-
gree of f by counting the number of p
i
s above, keeping track of
orientation.
Theorem 3.6.4. For each p
i
f
1
(q) let
p
i
= +1 if f : U
i
W is
orientation preserving and 1 if f : U
i
W is orientation reversing.
Then
(3.6.1) deg(f) =
N

i=1

p
i
.
3.6 Techniques for computing the degree of a mapping 129
Proof. Let be a compactly supported n-form on W whose integral
is one. Then
deg(f) =
_
U
f

=
N

i=1
_
U
i
f

.
Since f : U
i
W is a dieomorphism
_
U
i
f

=
_
W
= +1 or 1
depending on whether f : U
i
W is orientation preserving or not.
Thus deg(f) is equal to the sum (3.6.1).
As we pointed out above, a point, q V can qualify as a regular
value of f by default, i.e., by not being in the image of f. In this
case the recipe (3.6.1) for computing the degree gives by default
the answer zero. Lets corroborate this directly.
Theorem 3.6.5. If f : U V isnt onto, deg(f) = 0.
Proof. By exercise 3 of 3.4, V f(U) is open; so if it is non-empty,
there exists a compactly supported n-form, , with support in V
f(U) and with integral equal to one. Since = 0 on the image of f,
f

= 0; so
0 =
_
U
f

= deg(f)
_
V
= deg(f) .
Remark: In applications the contrapositive of this theorem is much
more useful than the theorem itself.
Theorem 3.6.6. If deg(f) = 0 f maps U onto V .
In other words if deg(f) = 0 the equation
(3.6.2) f(x) = y
has a solution, x U for every y V .
We will now show that the degree of f is a topological invariant of
f: if we deform f by a homotopy we dont change its degree. To
make this assertion precise, lets recall what we mean by a homotopy
130 Chapter 3. Integration of forms
between a pair of C

maps. Let U be an open subset of R


m
, V an
open subset of R
n
, A an open subinterval of R containing 0 and 1, and
f
i
: U V , i = 0, 1, C

maps. Then a C

map F : U A V is a
homotopy between f
0
and f
1
if F(x, 0) = f
0
(x) and F(x, 1) = f
1
(x).
(See Denition ??.) Suppose now that f
0
and f
1
are proper.
Denition 3.6.7. F is a proper homotopy between f
0
and f
1
if the
map
(3.6.3) F

: U A V A
mapping (x, t) to (F(x, t), t) is proper.
Note that if F is a proper homotopy between f
0
and f
1
, then for
every t between 0 and 1, the map
f
t
: U V , f
t
(x) = F
t
(x)
is proper.
Now let U and V be open subsets of R
n
.
Theorem 3.6.8. If f
0
and f
1
are properly homotopic, their degrees
are the same.
Proof. Let
= (y) d y
1
d y
n
be a compactly supported n-form on X whose integral over V is 1.
The the degree of f
t
is equal to
(3.6.4)
_
U
(F
1
(x, t), . . . , F
n
(x, t)) det D
x
F(x, t) dx.
The integrand in (3.6.4) is continuous and for 0 t 1 is supported
on a compact subset of U [0, 1], hence (3.6.4) is continuous as a
function of t. However, as weve just proved, deg(f
t
) is integer valued
so this function is a constant.
(For an alternative proof of this result see exercise 9 below.) Well
conclude this account of degree theory by describing a couple appli-
cations.
Application 1. The Brouwer xed point theorem
3.6 Techniques for computing the degree of a mapping 131
Let B
n
be the closed unit ball in R
n
:
{x R
n
, x 1} .
Theorem 3.6.9. If f : B
n
B
n
is a continuous mapping then f
has a xed point, i.e., maps some point, x
0
B
n
onto itself.
The idea of the proof will be to assume that there isnt a xed
point and show that this leads to a contradiction. Suppose that for
every point, x B
n
f(x) = x. Consider the ray through f(x) in the
direction of x:
f(x) +s(x f(x)) , 0 s < .
This intersects the boundary, S
n1
, of B
n
in a unique point, (x),
(see gure 1 below); and one of the exercises at the end of this section
will be to show that the mapping : B
n
S
n1
, x (x), is a
continuous mapping. Also it is clear from gure 1 that (x) = x if
x S
n1
, so we can extend to a continuous mapping of R
n
into
R
n
by letting be the identity for x 1. Note that this extended
mapping has the property
(3.6.5) (x) 1
for all x R
n
and
(3.6.6) (x) = x
for all x 1. To get a contradiction well show that can be
approximated by a C

map which has similar properties. For this


we will need the following corollary of Theorem 3.5.4.
Lemma 3.6.10. Let U be an open subset of R
n
, C a compact subset
of U and : U R a continuous function which is C

on the
complement of C. Then for every > 0, there exists a C

function,
: U R, such that has compact support and | | < .
Proof. Let be a bump function which is in C

0
(U) and is equal to
1 on a neighborhood of C. By Theorem 3.5.4 there exists a function,

0
C

0
(U) such that |
0
| < . Let = (1 ) +
0
, and
note that
= (1 ) + (1 )
0
=
0
.
132 Chapter 3. Integration of forms
By applying this lemma to each of the coordinates of the map, ,
one obtains a C

map, g : R
n
R
n
such that
(3.6.7) g < < 1
and such that g = on the complement of a compact set. How-
ever, by (3.6.6), this means that g is equal to the identity on the
complement of a compact set and hence (see exercise 9) that g is
proper and has degree one. On the other hand by (3.6.8) and (3.6.6)
g(x) > 1 for all x R
n
, so 0 / Img and hence by Theorem 3.6.4,
deg(g) = 0. Contradiction.
x
f(x)
(x)
Figure 3.6.1.
Application 2. The fundamental theorem of algebra
Let p(z) = z
n
+a
n1
z
n1
+ +a
1
z+a
0
be a polynomial of degree
n with complex coecients. If we identify the complex plane
C = {z = x +iy ; x, y R}
with R
2
via the map, (x, y) R
2
z = x +iy, we can think of p as
dening a mapping
p : R
2
R
2
, z p(z) .
3.6 Techniques for computing the degree of a mapping 133
We will prove
Theorem 3.6.11. The mapping, p, is proper and deg(p) = n.
Proof. For t R
p
t
(z) = (1 t)z
n
+tp(z)
= z
n
+t
n1

i=0
a
i
z
i
.
We will show that the mapping
g : R R
2
R
2
, z p
t
(z)
is a proper homotopy. Let
C = sup{|a
i
| , i = 0, . . . , n 1} .
Then for |z| 1
|a
0
+ +a
n1
z
n1
| |a
0
| +|a
1
||z| + +|a
n1
| |z|
n1
C|z|
n1
,
and hence, for |t| a and |z| 2aC,
|p
t
(z)| |z|
n
aC|z|
n1
aC|z|
n1
.
If A is a compact subset of C then for some R > 0, A is contained
in the disk, |w| R and hence the set
{z C, (p
t
(z), t) A[a, a]}
is contained in the compact set
{z C, aC|z|
n1
R} ,
and this shows that g is a proper homotopy. Thus each of the map-
pings,
p
t
: C C,
is proper and deg p
t
= deg p
1
= deg p = deg p
0
. However, p
0
: C C
is just the mapping, z z
n
and an elementary computation (see
exercises 5 and 6 below) shows that the degree of this mapping is n.
134 Chapter 3. Integration of forms
In particular for n > 0 the degree of p is non-zero; so by Theo-
rem 3.6.4 we conclude that p : C C is surjective and hence has
zero in its image.
Theorem 3.6.12. (fundamental theorem of algebra)
Every polynomial,
p(z) = z
n
+a
n1
z
n1
+ +a
0
,
with complex coecients has a complex root, p(z
0
) = 0, for some
z
0
C.
Exercises for 3.6
1. Let W be a subset of R
n
and let a(x), b(x) and c(x) be real-
valued functions on W of class C
r
. Suppose that for every x W
the quadratic polynomial
(*) a(x)s
2
+b(x)s +c(x)
has two distinct real roots, s
+
(x) and s

(x), with s
+
(x) > s

(x).
Prove that s
+
and s

are functions of class C


r
.
Hint: What are the roots of the quadratic polynomial: as
2
+bs+c?
2. Show that the function, (x), dened in gure 1 is a continuous
mapping of B
n
onto S
2n1
. Hint: (x) lies on the ray,
f(x) +s(x f(x)) , 0 s <
and satises (x) = 1; so (x) is equal to
f(x) +s
0
(x f(x))
where s
0
is a non-negative root of the quadratic polynomial
f(x) +s(x f(x))
2
1 .
Argue from gure 1 that this polynomial has to have two distinct
real roots.
3.6 Techniques for computing the degree of a mapping 135
3. Show that the Brouwer xed point theorem isnt true if one
replaces the closed unit ball by the open unit ball. Hint: Let U be
the open unit ball (i.e., the interior of B
n
). Show that the map
h : U R
n
, h(x) =
x
1 x
2
is a dieomorphism of U onto R
n
, and show that there are lots of
mappings of R
n
onto R
n
which dont have xed points.
4. Show that the xed point in the Brouwer theorem doesnt have
to be an interior point of B
n
, i.e., show that it can lie on the bound-
ary.
5. If we identify C with R
2
via the mapping: (x, y) z = x +iy,
we can think of a C-linear mapping of C into itself, i.e., a mapping
of the form
z cz , c C
as being an R-linear mapping of R
2
into itself. Show that the deter-
minant of this mapping is |c|
2
.
6. (a) Let f : C C be the mapping, f(z) = z
n
. Show that
Df(z) = nz
n1
.
Hint: Argue from rst principles. Show that for h C = R
2
(z +h)
n
z
n
nz
n1
h
|h|
tends to zero as |h| 0.
(b) Conclude from the previous exercise that
det Df(z) = n
2
|z|
2n2
.
(c) Show that at every point z C0, f is orientation preserving.
(d) Show that every point, w C 0 is a regular value of f and
that
f
1
(w) = {z
1
, . . . , z
n
}
with
z
i
= +1.
(e) Conclude that the degree of f is n.
136 Chapter 3. Integration of forms
7. Prove that the map, f, in exercise 6 has degree n by deducing
this directly from the denition of degree. Some hints:
(a) Show that in polar coordinates, f is the map, (r, ) (r
n
, n).
(b) Let be the two-form, g(x
2
+y
2
) dx dy, where g(t) is a com-
pactly supported C

function of t. Show that in polar coordinates,


= g(r
2
)r dr d, and compute the degree of f by computing the
integrals of and f

, in polar coordinates and comparing them.


8. Let U be an open subset of R
n
, V an open subset of R
m
, A an
open subinterval of R containing 0 and 1, f
i
: U V i = 0, 1, a pair
of C

mappings and F : U A V a homotopy between f


0
and f
1
.
(a) In 2.3, exercise 4 you proved that if is in
k
(V ) and d = 0,
then
(3.6.8) f

0
f

1
= d
where is the (k 1)-form, Q, in formula (??). Show (by careful
inspection of the denition of Q) that if F is a proper homotopy
and
k
c
(V ) then
k1
c
(U).
(b) Suppose in particular that U and V are open subsets of R
n
and is in
n
c
(V ). Deduce from (3.6.8) that
_
f

0
=
_
f

and deduce directly from the denition of degree that degree is a


proper homotopy invariant.
9. Let U be an open connected subset of R
n
and f : U U
a proper C

map. Prove that if f is equal to the identity on the


complement of a compact set, C, then f is proper and its degree is
equal to 1. Hints:
(a) Show that for every subset, A, of U, f
1
(A) A C, and
conclude from this that f is proper.
(b) Let C

= f(C). Use the recipe (1.6.1) to compute deg(f) with


q U C

.
10. Let [a
i,j
] be an n n matrix and A : R
n
R
n
the linear
mapping associated with this matrix. Frobenius theorem asserts: If
the a
i,j
s are non-negative then A has a non-negative eigenvalue. In
3.7 Appendix: Sards theorem 137
other words there exists a v R
n
and a R, 0, such that
Av = v. Deduce this linear algebra result from the Brouwer xed
point theorem. Hints:
(a) We can assume that A is bijective, otherwise 0 is an eigenvalue.
Let S
n1
be the (n 1)-sphere, |x| = 1, and f : S
n1
S
n1
the
map,
f(x) =
Ax
Ax
.
Show that f maps the set
Q = {(x
1
, . . . , x
n
) S
n1
; x
i
0}
into itself.
(b) Its easy to prove that Q is homeomorphic to the unit ball
B
n1
, i.e., that there exists a continuous map, g : Q B
n1
which is
invertible and has a continuous inverse. Without bothering to prove
this fact deduce from it Frobenius theorem.
3.7 Appendix: Sards theorem
The version of Sards theorem stated in 3.5 is a corollary of the
following more general result.
Theorem 3.7.1. Let U be an open subset of R
n
and f : U R
n
a
C

map. Then R
n
f(C
f
) is dense in R
n
.
Before undertaking to prove this we will make a few general com-
ments about this result.
Remark 3.7.2. If O
n
, n = 1, 2, are open dense subsets of R
n
, the
intersection

n
O
n
is dense in R
n
. (See [?], pg. 200 or exercise 4 below.)
Remark 3.7.3. If A
n
, n = 1, 2, . . . are a covering of U by compact
sets, O
n
= R
n
f(C
f
A
n
) is open, so if we can prove that its dense
then by Remark 3.7.2 we will have proved Sards theorem. Hence
since we can always cover U by a countable collection of closed cubes,
it suces to prove: for every closed cube, A U, R
n
f(C
f
A) is
dense in R
n
.
138 Chapter 3. Integration of forms
Remark 3.7.4. Let g : W U be a dieomorphism and let h =
f g. Then
(3.7.1) f(C
f
) = h(C
h
)
so Sards theorem for g implies Sards theorem for f.
We will rst prove Sards theorem for the set of super-critical
points of f, the set:
(3.7.2) C

f
= {p U , Df(p) = 0} .
Proposition 3.7.5. Let A U be a closed cube. Then the open set
R
n
f(A C

f
) is a dense subset of R
n
.
Well deduce this from the lemma below.
Lemma 3.7.6. Given > 0 one can cover f(A C

f
) by a nite
number of cubes of total volume less than .
Proof. Let the length of each of the sides of A be . Given > 0 one
can subdivide A into N
n
cubes, each of volume,
_

N
_
n
, such that
if x and y are points of any one of these subcubes
(3.7.3)

f
i
x
j
(x)
f
i
x
j
(y)

< .
Let A
1
, . . . , A
m
be the cubes in this collection which intersect C

f
.
Then for z
0
A
i
C

f
,
f
i
x
j
(z
0
) = 0, so for z A
i
(3.7.4)

f
i
x
j
(z)

<
by (3.7.3). If x and y are points of A
i
then by the mean value theorem
there exists a point z on the line segment joining x to y such that
f
i
(x) f
i
(y) =

f
i
x
j
(z)(x
j
y
j
)
and hence by (3.7.4)
(3.7.5) |f
i
(x) f
i
(y)|

|x
i
y
i
| n

N
.
3.7 Appendix: Sards theorem 139
Thus f(C
f
A
i
) is contained in a cube, B
i
, of volume
_
n

N
_
n
, and
f(C
f
A) is contained in a union of cubes, B
i
, of total volume less
that
N
n
n
n

n
N
n
= n
n

n
so if w choose
n

n
< , were done.
Proof. To prove Proposition 3.7.5 we have to show that for every
point p R
n
and neighborhood, W, of p, W f(C

f
A) is non-
empty. Suppose
(3.7.6) W f(C

f
A) .
Without loss of generality we can assume W is a cube of volume ,
but the lemma tells us that f(C

f
A) can be covered by a nite
number of cubes whose total volume is less than , and hence by
(3.7.6) W can be covered by a nite number of cubes of total volume
less than , so its volume is less than . This contradiction proves
that the inclusion (3.7.6) cant hold.
To prove Theorem 3.7.1 let U
i,j
be the subset of U where
f
i
x
j
= 0.
Then
U =
_
U
i,j
C

f
,
so to prove the theorem it suces to show that R
n
f(U
i,j
C
f
) is
dense in R
n
, i.e., it suces to prove the theorem with U replaced by
U
i,j
. Let
i
: R
n
R
n
be the involution which interchanges x
1
and
x
i
and leaves the remaining x
k
s xed. Letting f
new
=
i
f
old

j
and
U
new
=
j
U
old
, we have, for f = f
new
and U = U
new
(3.7.7)
f
1
x
1
(p) = 0 for all p U}
so were reduced to proving Theorem 3.7.1 for maps f : U R
n
having the property (3.7.6). Let g : U R
n
be dened by
(3.7.8) g(x
1
, . . . , x
n
) = (f
1
(x), x
2
, . . . , x
n
) .
140 Chapter 3. Integration of forms
Then
g

x
1
= f

x
1
= f
1
(x
1
, . . . , x
n
) (3.7.9)
and
det(Dg) =
f
1
x
1
= 0 . (3.7.10)
Thus, by the inverse function theorem, g is locally a dieomorphism
at every point, p U. This means that if A is a compact subset of
U we can cover A by a nite number of open subsets, U
i
U such
that g maps U
i
dieomorphically onto an open subset W
i
in R
n
. To
conclude the proof of the theorem well show that R
n
f(C
f
U
i
A)
is a dense subset of R
n
. Let h : W
i
R
n
be the map h = f g
1
.
To prove this assertion it suces by Remark 3.7.4 to prove that the
set
R
n
h(C
h
)
is dense in R
n
. This we will do by induction on n. First note that for
n = 1, C
f
= C

f
, so weve already proved Theorem 3.7.1 in dimension
one. Now note that by (3.7.8), h

x
1
= x
1
, i.e., h is a mapping of the
form
(3.7.11) h(x
1
, . . . , x
n
) = (x
1
, h
2
(x), . . . , h
n
(x)) .
Thus if we let W
c
be the set
(3.7.12) {(x
2
, . . . , x
n
) R
n1
; (c, x
2
, . . . , x
n
) W
i
}
and let h
c
: W
c
R
n1
be the map
(3.7.13) h
c
(x
2
, . . . , x
n
) = (h
2
(c, x
2
, . . . , x
n
), . . . , h
n
(c, x
2
, . . . , x
n
)) .
Then
(3.7.14) det(Dh
c
)(x
2
, . . . , x
n
) = det(Dh)(c, x
2
, . . . , x
n
)
and hence
(3.7.15) (c, x) W
i
C
h
x C
hc
.
Now let p
0
= (c, x
0
) be a point in R
n
. We have to show that every
neighborhood, V , of p
0
contains a point p R
n
h(C
h
). Let V
c

R
n1
be the set of points, x, for which (c, x) V . By induction V
c
contains a point, x R
n1
h
c
(C
hc
) and hence p = (c, x) is in V by
denition and in R
n
h(C
n
) by (3.7.15).
Q.E.D.
3.7 Appendix: Sards theorem 141
Exercises for 3.7
1. (a) Let f : R R be the map f(x) = (x
2
1)
2
. What is the
set of critical points of f? What is its image?
(b) Same questions for the map f(x) = sin x +x.
(c) Same questions for the map
f(x) =
_
0, x 0
e

1
x
, x > 0
.
2. Let f : R
n
R
n
be an ane map, i.e., a map of the form
f(x) = A(x) +x
0
where A : R
n
R
n
is a linear map. Prove Sards theorem for f.
3. Let : R R be a C

function which is supported in the


interval
_

1
2
,
1
2
_
and has a maximum at the origin. Let r
1
, r
2
, . . . ,
be an enumeration of the rational numbers, and let f : R R be
the map
f(x) =

i=1
r
i
(x i) .
Show that f is a C

map and show that the image of C


f
is dense in
R. (The moral of this example: Sards theorem says that the com-
plement of C
f
is dense in R, but C
f
can be dense as well.)
4. Prove the assertion made in Remark 3.7.2. Hint: You need to
show that for every point p R
n
and every neighborhood, V , of p,

O
n
V is non-empty. Construct, by induction, a family of closed
balls, B
k
, such that
(a) B
k
V
(b) B
k+1
B
k
(c) B
k

nk
O
n
(d) radius B
k
<
1
k
and show that the intersection of the B
k
s is non-empty.
5. Verify (3.7.1).
This is page 142
Printer: Opaque this
This is page 143
Printer: Opaque this
CHAPTER 4
FORMS ON MANIFOLDS
4.1 Manifolds
Our agenda in this chapter is to extend to manifolds the results of
Chapters 2 and 3 and to formulate and prove manifold versions of two
of the fundamental theorems of integral calculus: Stokes theorem
and the divergence theorem. In this section well dene what we
mean by the term manifold, however, before we do so, a word of
encouragement. Having had a course in multivariable calculus, you
are already familiar with manifolds, at least in their one and two
dimensional emanations, as curves and surfaces in R
3
, i.e., a manifold
is basically just an n-dimensional surface in some high dimensional
Euclidean space. To make this denition precise let X be a subset of
R
N
, Y a subset of R
n
and f : X Y a continuous map. We recall
Denition 4.1.1. f is a C

map if for every p X, there exists a


neighborhood, U
p
, of p in R
N
and a C

map, g
p
: U
p
R
n
, which
coincides with f on U
p
X.
We also recall:
Theorem 4.1.2. If f : X Y is a C

map, there exists a neigh-


borhood, U, of X in R
N
and a C

map, g : U R
n
such that g
coincides with f on X.
(A proof of this can be found in Appendix A.)
We will say that f is a dieomorphism if it is oneone and onto
and f and f
1
are both C

maps. In particular if Y is an open subset


of R
n
, X is an example of an object which we will call a manifold.
More generally,
Denition 4.1.3. A subset, X, of R
N
is an n-dimensional manifold
if, for every p X, there exists a neighborhood, V , of p in R
m
, an
open subset, U, in R
n
, and a dieomorphism : U X V .
Thus X is an n-dimensional manifold if, locally near every point p,
X looks like an open subset of R
n
.
Some examples:
144 Chapter 4. Forms on Manifolds
1. Graphs of functions. Let U be an open subset of R
n
and
f : U R a C

function. Its graph

f
= {(x, t) R
n+1
; x U , t = f(x)}
is an n-dimensional manifold in R
n+1
. In fact the map
: U R
n+1
, x (x, f(x))
is a dieomorphism of U onto
f
. (Its clear that is a C

map,
and it is a dieomorphism since its inverse is the map, :
f
U,
(x, t) = x, which is also clearly C

.)
2. Graphs of mappings. More generally if f : U R
k
is a C

map, its graph

f
= {(x, y) R
n
R
n
, x U , y = f(x)}
is an n-dimensional manifold in R
n+k
.
3. Vector spaces. Let V be an n- dimensional vector subspace of
R
N
, and (e
1
, . . . , e
n
) a basis of V . Then the linear map
(4.1.1) : R
n
V , (x
1
, . . . , x
n
)

x
i
e
i
is a dieomorphism of R
n
onto V . Hence every n-dimensional vector
subspace of R
N
is automatically an n-dimensional submanifold of
R
N
. Note, by the way, that if V is any n-dimensional vector space,
not necessarily a subspace of R
N
, the map (4.1.1) gives us an iden-
tication of V with R
n
. This means that we can speak of subsets
of V as being k-dimensional submanifolds if, via this identication,
they get mapped onto k-dimensional submanifolds of R
n
. (This is
a trivial, but useful, observation since a lot of interesting manifolds
occur in nature as subsets of some abstract vector space rather
than explicitly as subsets of some R
n
. An example is the manifold,
O(n), of orthogonal n n matrices. (See example 10 below.) This
manifold occurs in nature as a submanifold of the vector space of n
by n matrices.)
4. Ane subspaces of R
n
. These are manifolds of the form p+V ,
where V is a vector subspace of R
N
, and p is some specied point in
4.1 Manifolds 145
R
N
. In other words, they are dieomorphic copies of the manifolds
in example 3 with respect to the dieomorphism

p
: R
N
R
N
, x x +p .
If X is an arbitrary submanifold of R
N
its tangent space a point,
p X, is an example of a manifold of this type. (Well have more to
say about tangent spaces in 4.2.)
5. Product manifolds. Let X
i
, i = 1, 2 be an n
i
-dimensional sub-
manifold of R
N
i
. Then the Cartesian product of X
1
and X
2
X
1
X
2
= {(x
1
, x
2
) ; x
i
X
i
}
is an n-dimensional submanifold of R
N
where n = n
1
+ n
2
and
R
N
= R
N
1
R
N
2
.
We will leave for you to verify this fact as an exercise. Hint: For
p
i
X
i
, i = 1, 2, there exists a neighborhood, V
i
, of p
i
in R
N
i
, an
open set, U
i
in R
n
i
, and a dieomorphism : U
i
X
i
V
i
. Let
U = U
1
U
2
, V = V
1
V
2
and X = X
1
X
2
, and let : U XV
be the product dieomorphism, ((q
1
),
2
(q
2
)).
6. The unit n-sphere. This is the set of unit vectors in R
n+1
:
S
n
= {x R
n+1
, x
2
1
+ +x
2
n+1
= 1} .
To show that S
n
is an n-dimensional manifold, let V be the open
subset of R
n+1
on which x
n+1
is positive. If U is the open unit ball
in R
n
and f : U R is the function, f(x) = (1(x
2
1
+ +x
2
n
))
1/2
,
then S
n
V is just the graph,
f
, of f as in example 1. So, just as
in example 1, one has a dieomorphism
: U S
n
V .
More generally, if p = (x
1
, . . . , x
n+1
) is any point on the unit sphere,
then x
i
is non-zero for some i. If x
i
is positive, then letting be the
transposition, i n + 1 and f

: R
n+1
R
n+1
, the map
f

(x
1
, . . . , x
n
) = (x
(1)
, . . . , x
(n)
)
one gets a dieomorphism, f

, of U onto a neighborhood of p in
S
n
and if x
i
is negative one gets such a dieomorphism by replacing
f

by f

. In either case weve shown that for every point, p, in S


n
,
there is a neighborhood of p in S
n
which is dieomorphic to U.
146 Chapter 4. Forms on Manifolds
7. The 2-torus. In calculus books this is usually described as the
surface of rotation in R
3
obtained by taking the unit circle centered
at the point, (2, 0), in the (x
1
, x
3
) plane and rotating it about the
x
3
-axis. However, a slightly nicer description of it is as the product
manifold S
1
S
1
in R
4
. (Exercise: Reconcile these two descriptions.)
Well now turn to an alternative way of looking at manifolds: as
solutions of systems of equations. Let U be an open subset of R
N
and f : U R
k
a C

map.
Denition 4.1.4. A point, a R
k
, is a regular value of f if for
every point, p f
1
(a), f is a submersion at p.
Note that for f to be a submersion at p, Df(p) : R
N
R
k
has to
be onto, and hence k has to be less than or equal to N. Therefore
this notion of regular value is interesting only if N k.
Theorem 4.1.5. Let N k = n. If a is a regular value of f, the
set, X = f
1
(a), is an n-dimensional manifold.
Proof. Replacing f by
a
f we can assume without loss of gener-
ality that a = 0. Let p f
1
(0). Since f is a submersion at p, the
canonical submersion theorem (see Appendix B, Theorem 2) tells us
that there exists a neighborhood, O, of 0 in R
N
, a neighborhood, U
0
,
of p in U and a dieomorphism, g : O U
0
such that
(4.1.2) f g =
where is the projection map
R
N
= R
k
R
n
R
k
, (x, y) x.
Hence
1
(0) = {0} R
n
= R
n
and by (4.1.1), g maps O
1
(0)
dieomorphically onto U
0
f
1
(0). However, O
1
(0) is a neigh-
borhood, V , of 0 in R
n
and U
0
f
1
(0) is a neighborhood of p in X,
and, as remarked, these two neighborhoods are dieomorphic.
Some examples:
8. The n-sphere. Let
f : R
n+1
R
4.1 Manifolds 147
be the map,
(x
1
, . . . , x
n+1
) x
2
1
+ +x
2
n+1
1 .
Then
Df(x) = 2(x
1
, . . . , x
n+1
)
so, if x = 0 f is a submersion at x. In particular f is a submersion
at all points, x, on the n-sphere
S
n
= f
1
(0)
so the n-sphere is an n-dimensional submanifold of R
n+1
.
9. Graphs. Let g : R
n
R
k
be a C

map and as in example 2 let

f
= {(x, y) R
n
R
k
, y = g(x)} .
We claim that
f
is an n-dimensional submanifold of R
n+k
= R
n

R
k
.
Proof. Let
f : R
n
R
k
R
k
be the map, f(x, y) = y g(x). Then
Df(x, y) = [Dg(x) , I
k
]
where I
k
is the identity map of R
k
onto itself. This map is always
of rank k. Hence
f
= f
1
(0) is an n-dimensional submanifold of
R
n+k
.
10. Let M
n
be the set of all n n matrices and let S
n
be the set
of all symmetric n n matrices, i.e., the set
S
n
= {A M
n
, A = A
t
} .
The map
[a
i,j
] (a
11
, a
12
, . . . , a
1n
, a
2,1
, . . . , a
2n
, . . .)
gives us an identication
M
n

= R
n
2
148 Chapter 4. Forms on Manifolds
and the map
[a
i,j
] (a
11
, . . . a
1n
, a
22
, . . . a
2n
, a
33
, . . . a
3n
, . . .)
gives us an identication
S
n

= R
n(n+1)
2
.
(Note that if A is a symmetric matrix,
a
12
= a
21
, a
13
= a
31
, a
32
= a
23
, etc.
so this map avoids redundancies.) Let
O(n) = {A M
n
, A
t
A = I} .
This is the set of orthogonal nn matrices, and we will leave for you
as an exercise to show that its an n(n1)/2-dimensional manifold.
Hint: Let f : M
n
S
n
be the map f(A) = A
t
AI. Then
O(n) = f
1
(0) .
These examples show that lots of interesting manifolds arise as
zero sets of submersions, f : U R
k
. This is, in fact, not just an
accident. We will show that locally every manifold arises this way.
More explicitly let X R
N
be an n-dimensional manifold, p a point
of X, U a neighborhood of 0 in R
n
, V a neighborhood of p in R
N
and
: (U, 0) (V X, p) a dieomorphism. We will for the moment
think of as a C

map : U R
N
whose image happens to lie in
X.
Lemma 4.1.6. The linear map
D(0) : R
n
R
N
is injective.
Proof.
1
: V X U is a dieomorphism, so, shrinking V if
necessary, we can assume that there exists a C

map : V U
which coincides with
1
on V X Since maps U onto V X,
=
1
is the identity map on U. Therefore,
D( )(0) = (D)(p)D(0) = I
by the chain rule, and hence if D(0)v = 0, it follows from this
identity that v = 0.
4.1 Manifolds 149
Lemma 4.1.6 says that is an immersion at 0, so by the canon-
ical immersion theorem (see Appendix B,Theorem 4) there exists a
neighborhood, U
0
, of 0 in U, a neighborhood, V
p
, of p in V , and a
dieomorphism
(4.1.3) g : (V
p
, p) (U
0
R
Nn
, 0)
such that
(4.1.4) g = ,
being, as in Appendix B, the canonical immersion
(4.1.5) : U
0
U
0
R
Nn
, x (x, 0) .
By (4.1.3) g maps (U
0
) dieomorphically onto (U
0
). However, by
(4.1.2) and (4.1.3) (U
0
) is dened by the equations, x
i
= 0, i =
n + 1, . . . , N. Hence if g = (g
1
, . . . , g
N
) the set, (U
0
) = V
p
X is
dened by the equations
(4.1.6) g
i
= 0 , i = n + 1, . . . , N .
Let = N n, let
: R
N
= R
n
R

be the canonical submersion,


(x
1
, . . . , x
N
) = (x
n+1
, . . . x
N
)
and let f = g. Since g is a dieomorphism, f is a submersion and
(4.1.5) can be interpreted as saying that
(4.1.7) V
p
X = f
1
(0) .
Thus to summarize weve proved
Theorem 4.1.7. Let X be an n-dimensional submanifold of R
N
and
let = N n. Then for every p X there exists a neighborhood, V
p
,
of p in R
N
and a submersion
f : (V
p
, p) (R

, 0)
such that X V
p
is dened by the equation (4.1.6).
150 Chapter 4. Forms on Manifolds
A nice way of thinking about Theorem 4.1.2 is in terms of the
coordinates of the mapping, f. More specically if f = (f
1
, . . . , f
k
)
we can think of f
1
(a) as being the set of solutions of the system of
equations
(4.1.8) f
i
(x) = a
i
, i = 1, . . . , k
and the condition that a be a regular value of f can be interpreted
as saying that for every solution, p, of this system of equations the
vectors
(4.1.9) (df
i
)
p
=

f
i
x
j
(0) dx
j
in T

p
R
n
are linearly independent, i.e., the system (4.1.7) is an in-
dependent system of dening equations for X.
Exercises.
1. Show that the set of solutions of the system of equations
x
2
1
+ +x
2
n
= 1
and
x
1
+ +x
n
= 0
is an n 2-dimensional submanifold of R
n
.
2. Let S
n1
be the n-sphere in R
n
and let
X
a
= {x S
n1
, x
1
+ +x
n
= a} .
For what values of a is X
a
an (n 2)-dimensional submanifold of
S
n1
?
3. Show that if X
i
, i = 1, 2, is an n
i
-dimensional submanifold of
R
N
i
then
X
1
X
2
R
N
1
R
N
2
is an (n
1
+n
2
)-dimensional submanifold of R
N
1
R
N
2
.
4.1 Manifolds 151
4. Show that the set
X = {(x, v) S
n1
R
n
, x v = 0}
is a 2n 2-dimensional submanifold of R
n
R
n
. (Here x v is the
dot product,

x
i
v
i
.)
5. Let g : R
n
R
k
be a C

map and let X = graph g. Prove


directly that X is an n-dimensional manifold by proving that the
map
: R
n
X , x (x, g(x))
is a dieomorphism.
6. Prove that O(n) is an n(n1)/2-dimensional manifold. Hints:
(a) Let f : M
n
S
n
be the map
f(A) = A
t
A = I .
Show that O(n) = f
1
(0).
(b) Show that
f(A+B) = A
t
A+(A
t
B +B
t
A) +
2
B
t
B.
(c) Conclude that the derivative of f at A is the map
(*) B M
n
A
t
B +B
t
A.
(d) Let A be in O(n). Show that if C is in S
n
and B = AC/2 then
the map, (*), maps B onto C.
(e) Conclude that the derivative of f is surjective at A.
(f) Conclude that 0 is a regular value of the mapping, f.
7. The next ve exercises, which are somewhat more demanding
than the exercises above, are an introduction to Grassmannian
geometry.
(a) Let e
1
, . . . , e
n
be the standard basis of R
n
and let W = span{e
k+1
, . . . , e
n
}.
Prove that if V is a k-dimensional subspace of R
n
and
(1.1) V W = {0} ,
152 Chapter 4. Forms on Manifolds
then one can nd a unique basis of V of the form
(1.2) v
i
= e
i
+

j=1
b
i,j
e
k+j
, i = 1, . . . , k ,
where = n k.
(b) Let G
k
be the set of k-dimensional subspaces of R
n
having the
property (1.1) and let M
k,
be the vector space of k matrices.
Show that one gets from the identities (1.2) a bijective map:
(1.3) : M
k,
G
k
.
8. Let S
n
be the vector space of linear mappings of R
n
into itself
which are self-adjoint, i.e., have the property A = A
t
.
(a) Given a k-dimensional subspace, V of R
n
let
V
: R
n
R
n
be
the orthogonal projection of R
n
onto V . Show that
V
is in S
n
and
is of rank k, and show that (
V
)
2
=
V
.
(b) Conversely suppose A is an element of S
n
which is of rank k
and has the property, A
2
= A. Show that if V is the image of A in
R
n
, then A =
V
.
Notation. We will call an A S
n
of the form, A =
V
above a
rank k projection operator.
9. Composing the map
(1.4) : G
k
S
n
, V
V
with the map (1.3) we get a map
(1.5) : M
k,
S
n
, = .
Prove that is C

.
Hints:
(a) By GramSchmidt one can convert (1.2) into an orthonormal
basis
(1.6) e
1,B
, . . . , e
n,B
of V . Show that the e
i,B
s are C

functions of the matrix, B = [b


i,j
].
4.1 Manifolds 153
(b) Show that
V
is the linear mapping
v V
k

i=1
(v e
i,B
)e
i,B
.
10. Let V
0
= span {e
1
, . . . , e
n
} and let

G
k
= (G
k
). Show that
maps a neighborhood of 0 in M
k,
dieomorphically onto a neigh-
borhood of
V
0
in

G
k
.
Hints:
V
is in

G
k
if and only if V satises (1.1). For 1 i k let
(1.7) w
i
=
V
(e
i
) =
k

j=1
a
i,j
e
j
+

r=1
c
i,r
e
k+r
.
(a) Show that if the matrix A = [a
i,j
] is invertible,
V
is in

G
k
.
(b) Let O

G
k
be the set of all
V
s for which A is invertible.
Show that
1
: O M
k,
is the map

1
(
V
) = B = A
1
C
where C = [c
i,j
].
11. Let G(k, n) S
n
be the set of rank k projection operators.
Prove that G(k, n) is a k-dimensional submanifold of the Euclidean
space, S
n
= R
n(n+1)
2
.
Hints:
(a) Show that if V is any k-dimensional subspace of R
n
there exists
a linear mapping, A O(n) mapping V
0
to V .
(b) Show that
V
= A
V
0
A
1
.
(c) Let K
A
: S
n
S
n
be the linear mapping,
K
A
(B) = ABA
1
.
Show that
K
A
: M
k,
S
n
maps a neighborhood of 0 in M
k,
dieomorphically onto a neigh-
borhood of
V
in G(k, n).
154 Chapter 4. Forms on Manifolds
Remark 4.1.8. Let Gr(k, n) be the set of all k-dimensional sub-
spaces of R
n
. The identication of Gr(k, n) with G(k, n) given by
V
V
allows us to restate the result above in the form.
The Grassmannnian Theorem: The set (Gr(k, n)) (a.k.a. the Grass-
mannian of k-dimensional subspaces of R
n
) is a k-dimensional sub-
manifold of S
n
= R
n(n+1)
2
.
12. Show that Gr(k, n) is a compact submanifold of S
n
. Hint: Show
that its closed and bounded.
4.2 Tangent spaces
We recall that a subset, X, of R
N
is an n-dimensional manifold, if,
for every p X, there exists an open set, U R
n
, a neighborhood,
V , of p in R
N
and a C

-dieomorphism, : U X X.
Denition 4.2.1. We will call a parametrization of X at p.
Our goal in this section is to dene the notion of the tangent space,
T
p
X, to X at p and describe some of its properties. Before giving
our ocial denition well discuss some simple examples.
Example 1.
Let f : R R be a C

function and let X = graphf.


4.2 Tangent spaces 155
p
0
X = graph f
x
l
Then in this gure above the tangent line, , to X at p
0
= (x
0
, y
0
)
is dened by the equation
y y
0
= a(x x
0
)
where a = f

(x
0
) In other words if p is a point on then p = p
0
+v
0
where v
0
= (1, a) and R. We would, however, like the tangent
space to X at p
0
to be a subspace of the tangent space to R
2
at p
0
,
i.e., to be the subspace of the space: T
p
0
R
2
= {p
0
} R
2
, and this
well achieve by dening
T
p
0
X = {(p
0
, v
0
) , R} .
Example 2.
Let S
2
be the unit 2-sphere in R
3
. The tangent plane to S
2
at p
0
is usually dened to be the plane
{p
0
+ v ; v R
3
, v p
0
} .
However, this tangent plane is easily converted into a subspace of
T
p
R
3
via the map, p
0
+ v (p
0
, v) and the image of this map
{(p
0
, v) ; v R
3
, v p
0
}
will be our denition of T
p
0
S
2
.
156 Chapter 4. Forms on Manifolds
Lets now turn to the general denition. As above let X be an
n-dimensional submanifold of R
N
, p a point of X, V a neighborhood
of p in R
N
, U an open set in R
n
and
: (U, q) (X V, p)
a parameterization of X. We can think of as a C

map
: (U, q) (V, p)
whose image happens to lie in X V and we proved in 4.1 that its
derivative at q
(4.2.1) (d)
q
: T
q
R
n
T
p
R
N
is injective.
Denition 4.2.2. The tangent space, T
p
X, to X at p is the image
of the linear map (4.2.1). In other words, w T
p
R
N
is in T
p
X if
and only if w = d
q
(v) for some v T
q
R
n
. More succinctly,
(4.2.2) T
p
X = (d
q
)(T
q
R
n
) .
(Since d
q
is injective this space is an n-dimensional vector subspace
of T
p
R
N
.)
One problem with this denition is that it appears to depend on
the choice of . To get around this problem, well give an alternative
denition of T
p
X. In 4.1 we showed that there exists a neighbor-
hood, V , of p in R
N
(which we can without loss of generality take
to be the same as V above) and a C

map
(4.2.3) f : (V, p) (R
k
, 0) , k = N n,
such that X V = f
1
(0) and such that f is a submersion at all
points of X V , and in particular at p. Thus
df
p
: T
p
R
N
T
0
R
k
is surjective, and hence the kernel of df
p
has dimension n. Our alter-
native denition of T
p
X is
(4.2.4) T
p
X = kernel df
p
.
4.2 Tangent spaces 157
The spaces (4.2.2) and (4.2.4) are both n-dimensional subspaces
of T
p
R
N
, and we claim that these spaces are the same. (Notice that
the denition (4.2.4) of T
p
X doesnt depend on , so if we can show
that these spaces are the same, the denitions (4.2.2) and (4.2.4) will
depend neither on nor on f.)
Proof. Since (U) is contained in X V and X V is contained in
f
1
(0), f = 0, so by the chain rule
(4.2.5) df
p
d
q
= d(f )
q
= 0 .
Hence if v T
p
R
n
and w = d
q
(v), df
p
(w) = 0. This shows that the
space (4.2.2) is contained in the space (4.2.4). However, these two
spaces are n-dimensional so they coincide.
From the proof above one can extract a slightly stronger result:
Theorem 4.2.3. Let W be an open subset of R

and h : (W, q)
(R
N
, p) a C

map. Suppose h(W) is contained in X. Then the image


of the map
dh
q
: T
q
R

T
p
R
N
is contained in T
p
X.
Proof. Let f be the map (4.2.3). We can assume without loss of
generality that h(W) is contained in V , and so, by assumption,
h(W) XV . Therefore, as above, f h = 0, and hence dh
q
(T
q
R

)
is contained in the kernel of df
p
.
This result will enable us to dene the derivative of a mapping
between manifolds. Explicitly: Let X be a submanifold of R
N
, Y a
submanifold of R
m
and g : (X, p) (Y, y
0
) a C

map. By De-
nition 4.1.1 there exists a neighborhood, O, of X in R
N
and a C

map, g : O R
m
extending to g. We will dene
(4.2.6) (dg
p
) : T
p
X T
y
0
Y
to be the restriction of the map
(4.2.7) (d g)
p
: T
p
R
N
T
y
0
R
m
to T
p
X. There are two obvious problems with this denition:
158 Chapter 4. Forms on Manifolds
1. Is the space
(d g
p
)(T
p
X)
contained in T
y
0
Y ?
2. Does the denition depend on g?
To show that the answer to 1. is yes and the answer to 2. is no, let
: (U, x
0
) (X V, p)
be a parametrization of X, and let h = g . Since (U) X,
h(U) Y and hence by Theorem 4.2.4
dh
x
0
(T
x
0
R
n
) T
y
0
Y .
But by the chain rule
(4.2.8) dh
x
0
= d g
p
d
x
0
,
so by (4.2.2)
(d g
p
)(T
p
X) T
p
Y (4.2.9)
and
(d g
p
)(T
p
X) = (dh)
x
0
(T
x
0
R
n
) (4.2.10)
Thus the answer to 1. is yes, and since h = g = g , the answer
to 2. is no.
From (4.2.5) and (4.2.6) one easily deduces
Theorem 4.2.4 (Chain rule for mappings between manifolds). Let
Z be a submanifold of R

and : (Y, y
0
) (Z, z
0
) a C

map. Then
d
y
0
dg
p
= d( g)
p
.
We will next prove manifold versions of the inverse function theo-
rem and the canonical immersion and submersion theorems.
Theorem 4.2.5 (Inverse function theorem for manifolds). Let X
and Y be n-dimensional manifolds and f : X Y a C

map.
Suppose that at p X the map
df
p
: T
p
X T
q
Y , q = f(p) ,
is bijective. Then f maps a neighborhood, U, of p in X dieomor-
phically onto a neighborhood, V , of q in Y .
4.2 Tangent spaces 159
Proof. Let U and V be open neighborhoods of p in X and q in Y
and let

0
: (U
0
, p
0
) (U, p)
and

0
: (V
0
, q
0
) (V, q)
be parametrizations of these neighborhoods. Shrinking U
0
and U we
can assume that f(U) V . Let
g : (U
0
, p
0
) (V
0
, q
0
)
be the map
1
0
f
0
. Then
0
g = f , so by the chain rule
(d
0
)
q
0
(dg)
p
0
= (df)
p
(d
0
)
p
0
.
Since (d
0
)
q
0
and (d
0
)
p
0
are bijective its clear from this identity
that if df
p
is bijective the same is true for (dg)
p
0
. Hence by the inverse
function theorem for open subsets of R
n
, g maps a neighborhood of
p
0
in U
0
dieomorphically onto a neighborhood of q
0
in V
0
. Shrinking
U
0
and V
0
we assume that these neighborhoods are U
0
and V
0
and
hence that g is a dieomorphism. Thus since f : U V is the map

0
g
1
0
, it is a dieomorphism as well.
Theorem 4.2.6 (The canonical submersion theorem for manifolds).
Let X and Y be manifolds of dimension n and m, m < n, and let
f : X Y be a C

map. Suppose that at p X the map


df
p
: T
p
X T
q
Y , q = f(p) ,
is surjective. Then there exists an open neighborhood, U, of p in X,
and open neighborhood, V of f(U) in Y and parametrizations

0
: (U
0
, 0) (U, p)
and

0
: (V
0
, 0) (V, q)
160 Chapter 4. Forms on Manifolds
such that in the diagram below
U
f
V

0
U
0
V
0
the bottom arrow,
1
0
f
0
, is the canonical submersion, .
Proof. Let U and V be open neighborhoods of p and q and

0
: (U
0
, p
0
) (U, p)
and

0
: (V
0
, q
0
) (V, q)
be parametrizations of these neighborhoods. Composing
0
and
0
with the translations we can assume that p
0
is the origin in R
n
and
q
0
the origin in R
m
, and shrinking U we can assume f(U) V . As
above let g : (U
0
, 0) (V
0
, 0) be the map,
1
0
f
0
. By the chain
rule
(d
0
)
0
(dg)
0
= df
p
(d
0
)
0
,
therefore, since (d
0
)
0
and (d
0
)
0
are bijective it follows that (dg)
0
is
surjective. Hence, by Theorem ??, we can nd an open neighborhood,
U, of the origin in R
n
and a dieomorphism,
1
: (U
1
, 0) (U
0
, 0)
such that g
1
is the canonical submersion. Now replace U
0
by U
1
and
0
by
0

1
.
Theorem 4.2.7 (The canonical immersion theorem for manifolds).
Let X and Y be manifolds of dimension n and m, n < m, and
f : X Y a C

map. Suppose that at p X the map


df
p
: T
p
X T
q
Y , q = f(p)
is injective. Then there exists an open neighborhood, U, of p in X,
an open neighborhood, V , of f(U) in Y and parametrizations

0
: (U
0
, 0) (U, p)
and
4.2 Tangent spaces 161

0
: (V
0
, 0) (V, q)
such that in the diagram below
U
f
V

0
U
0
V
0
the bottom arrow,
0
f
0
, is the canonical immersion, .
Proof. The proof is identical with the proof of Theorem 4.2.6 except
for the last step. In the last step one converts g into the canonical
immersion via a map
1
: (V
1
, 0) (V
0
, 0) with the property g
1
=
and then replaces
0
by
0

1
.
Exercises.
1. What is the tangent space to the quadric, x
n
= x
2
1
+ +x
2
n1
,
at the point, (1, 0, . . . , 0, 1)?
2. Show that the tangent space to the (n 1)-sphere, S
n1
, at p,
is the space of vectors, (p, v) T
p
R
n
satisfying p v = 0.
3. Let f : R
n
R
k
be a C

map and let X = graphf. What is


the tangent space to X at (a, f(a))?
4. Let : S
n1
S
n1
be the antipodal map, (x) = x. What
is the derivative of at p S
n1
?
5. Let X
i
R
N
i
, i = 1, 2, be an n
i
-dimensional manifold and let
p
i
X
i
. Dene X to be the Cartesian product
X
1
X
2
R
N
1
R
N
2
and let p = (p
1
, p
2
). Show that T
p
X is the vector space sum of the
vector spaces,T
p
1
X
1
and T
p
2
X
2
.
162 Chapter 4. Forms on Manifolds
6. Let X R
N
be an n-dimensional manifold and
i
: U
i

X V
i
, i = 1, 2, two parametrizations. From these parametrizations
one gets an overlap diagram
(4.2.11) W
1
W
2
X V


d
1
J
J
J ]
d
2

-
where V = V
1
V
2
, W
i
=
1
i
(X V ) and =
1
2

1
.
(a) Let p X V and let q
i
=
1
i
(p). Derive from the overlap
diagram (4.2.11) an overlap diagram of linear maps
(4.2.12) T
q
1
R
n
T
q
2
R
n
T
p
R
N


(d
1
)
q
1
J
J
J ^
(d
2
)
q
2
(d)
q
1
-
(b) Use overlap diagrams to give another proof that T
p
X is intrin-
sically dened.
4.3 Vector elds and dierential forms on manifolds
A vector eld on an open subset, U, of R
n
is a function, v, which
assigns to each p U an element, v(p), of T
p
U, and a k-form is a
function, , which assigns to each p U an element, (p), of
k
(T

p
).
These denitions have obvious generalizations to manifolds:
Denition 4.3.1. Let X be a manifold. A vector eld on X is a
function, v, which assigns to each p X an element, v(p), of T
p
X,
and a k-form is a function, , which assigns to each p X an
element, (p), of
k
(T

p
X).
Well begin our study of vector elds and k-forms on manifolds
by showing that, like their counterparts on open subsets of R
n
, they
have nice pull-back and push-forward properties with respect to map-
pings. Let X and Y be manifolds and f : X Y a C

mapping.
Denition 4.3.2. Given a vector eld, v, on X and a vector eld,
w, on Y , well say that v and w are f-related if, for all p X and
q = f(p)
(4.3.1) (df)
p
v(p) = w(q) .
4.3 Vector elds and dierential forms on manifolds 163
In particular, if f is a dieomorphism, and were given a vector
eld, v, on X we can dene a vector eld, w, on Y by requiring
that for every point, q Y , the identity (??) holds at the point,
p = f
1
(q). In this case well call w the push-forward of v by f
and denote it by f

v. Similarly, given a vector eld, w, on Y we can


dene a vector eld, v, on X by applying the same construction to
the inverse dieomorphism, f
1
: Y X. We will call the vector
eld (f
1
)

w the pull-back of w by f (and also denote it by f

w).
For dierential forms the situation is even nicer. Just as in 2.5
we can dene the pull-back operation on forms for any C

map
f : X Y . Specically: Let be a k-form on Y . For every p X,
and q = f(p) the linear map
df
p
: T
p
X T
q
Y
induces by (1.8.2) a pull-back map
(df
p
)

:
k
(T

q
)
k
(T

p
)
and, as in 2.5, well dene the pull-back, f

, of to X by dening
it at p by the identity
(4.3.2) (f

)(p) = (df
p
)

(q) .
The following results about these operations are proved in exactly
the same way as in 2.5.
Proposition 4.3.3. Let X, Y and Z be manifolds and f : X Y
and g : Y Z C

maps. Then if is a k-form on Z


(4.3.3) f

(g

) = (g f)

,
and if v is a vector eld on X and f and g are dieomorphisms
(4.3.4) (g f)

v = g

(f

v) .
Our rst application of these identities will be to dene what one
means by a C

vector eld and a C

k-form.
Let X be an n-dimensional manifold and U an open subset of X.
Denition 4.3.4. The set U is a parametrizable open set if there
exists an open set, U
0
, in R
n
and a dieomorphism,
0
: U
0
U.
164 Chapter 4. Forms on Manifolds
In other words, U is parametrizable if there exists a parametriza-
tion having U as its image. (Note that X being a manifold means
that every point is contained in a parametrizable open set.)
Now let U X be a parametrizable open set and : U
0
U a
parametrization of U.
Denition 4.3.5. A k-form on U is C

if

0
is C

.
This denition appears to depend on the choice of the parametriza-
tion, , but we claim it doesnt. To see this let
1
: U
1
U be
another parametrization of U and let
: U
0
U
1
be the composite map,
1
0

0
. Then
0
=
1
and hence by
Proposition 4.3.3

0
=

1
,
so by (2.5.11)

0
is C

if

1
is C

. The same argument applied


to
1
shows that

1
is C

if

0
is C

. Q.E.D
The notion of C

for vector elds is dened similarly:


Denition 4.3.6. A vector eld, v, on U is C

if

0
v is C

.
By Proposition 4.3.3

0
v =

1
v, so, as above, this denition is
independent of the choice of parametrization.
We now globalize these denitions.
Denition 4.3.7. A k-form, , on X is C

if, for every point


p X, is C

on a neighborhood of p. Similarly, a vector eld, v,


on X is C

if, for every point, p X, v is C

on a neighborhood
of p.
We will also use the identities (4.3.4) and (4.3.5) to prove the
following two results.
Proposition 4.3.8. Let X and Y be manifolds and f : X Y a
C

map. Then if is a C

k-form on Y , f

is a C

k-form on X.
Proof. For p X and q = f(p) let
0
: U
0
U and
0
: V
0
V
be parametrizations with p U and q V . Shrinking U if necessary
we can assume that f(U) V . Let g : U
0
V
0
be the map, g =

1
0
f
0
. Then
0
g = f
0
, so g

0
=

0
f

. Since is
C

0
is C

, so by (2.5.11) g

0
is C

, and hence,

0
f

is C

.
Thus by denition f

is C

on U.
4.3 Vector elds and dierential forms on manifolds 165
By exactly the same argument one proves:
Proposition 4.3.9. If w is a C

vector eld on Y and f is a dif-


feomorphism, f

w is a C

vector eld on X.
Some notation:
1. Well denote the space of C

k-forms on X by
k
(X).
2. For
k
(X) well dene the support of to be the
closure of the set
{p X , (p) = 0}
and well denote by
k
c
(X) the space of completely supported
k-forms.
3. For a vector eld, v, on X well dene the support of v to
be the closure of the set
{p X , v(p) = 0} .
We will now review some of the results about vector elds and the
dierential forms that we proved in Chapter 2 and show that they
have analogues for manifolds.
1. Integral curves
Let I R be an open interval and : I X a C

curve. For t
0
I
we will call u = (t
0
, 1) T
t
0
R the unit vector in T
t
0
R and if p = (t
0
)
we will call the vector
d
t
0
(u) T
p
X
the tangent vector to at p. If v is a vector eld on X we will say
that is an integral curve of v if for all t
0
I
v((t
0
)) = d
t
0
(u) .
Proposition 4.3.10. Let X and Y be manifolds and f : X Y a
C

map. If v and w are vector elds on X and Y which are f-related,


then integral curves of v get mapped by f onto integral curves of w.
166 Chapter 4. Forms on Manifolds
Proof. If the curve, : I X is an integral curve of v we have to
show that f : I Y is an integral curve of w. If (t) = p and
q = f(p) then by the chain rule
w(q) = df
p
(v(p)) = df
p
(d
t
(u))
= d(f )
t
(u) .
From this result it follows that the local existence, uniqueness
and smooth dependence on initial data results about vector elds
that we described in 2.1 are true for vector elds on manifolds.
More explicitly, let U be a parametrizable open subset of X and
: U
0
U a parametrization. Since U
0
is an open subset of R
n
these results are true for the vector eld, w =

0
v and hence since
w and v are
0
-related they are true for v. In particular
Proposition 4.3.11 (local existence). For every p U there exists
an integral curve, (t) , < t < , of v with (0) = p.
Proposition 4.3.12 (local uniqueness). Let
i
: I
i
U i = 1, 2 be
integral curves of v and let I = I
1
I
2
. Suppose
2
(t) =
1
(t) for
some t I. Then there exists a unique integral curve, : I I
2
U
with =
1
on I
1
and =
2
on I
2
.
Proposition 4.3.13 (smooth dependence on initial data). For every
p U there exists a neighborhood, O of p in U, an interval (, )
and a C

map, h : O (, ) U such that for every p O the


curve

p
(t) = h(p, t) , < t < ,
is an integral curve of v with
p
(0) = p.
As in Chapter 2 we will say that v is complete if, for every p X
there exists an integral curve, (t), < t < , with (0) = p. In
Chapter 2 we showed that one simple criterium for a vector eld to
be complete is that it be compactly supported. We will prove that
the same is true for manifolds.
Theorem 4.3.14. If X is compact or, more generally, if v is com-
pactly supported, v is complete.
Proof. Its not hard to prove this by the same argument that we
used to prove this theorem for vector elds on R
n
, but well give a
4.3 Vector elds and dierential forms on manifolds 167
simpler proof that derives this directly from the R
n
result. Suppose
X is a submanifold of R
N
. Then for p X,
T
p
X T
p
R
N
= {(p, v) , v R
N
} ,
so v(p) can be regarded as a pair, (p, v(p)) where v(p) is in R
N
. Let
(4.3.5) f
v
: X R
N
be the map, f
v
(p) = v(p). It is easy to check that v is C

if and
only if f
v
is C

. (See exercise 11.) Hence (see Appendix B) there


exists a neighborhood, O of X and a map g : O R
N
extending
f
v
. Thus the vector eld w on O dened by w(q) = (q, g(q)) extends
the vector eld v to O. In other words if : X O is the inclusion
map, v and w are -related. Thus by Proposition 4.3.10 the integral
curves of v are just integral curves of w that are contained in X.
Suppose now that v is compactly supported. Then there exists a
function C

o
(O) which is 1 on the support of v, so, replacing
w by w, we can assume that w is compactly supported. Thus w is
complete. Let (t), < t < be an integral curve of w. We will
prove that if (0) X, then this curve is an integral curve of v. We
rst observe:
Lemma 4.3.15. The set of points, t R, for which (t) X is both
open and closed.
Proof. If p / suppv then w(p) = 0 so if (t) = p, (t) is the constant
curve, = p, and theres nothing to prove. Thus we are reduced to
showing that the set
(4.3.6) {t R, (t) suppv}
is both open and closed. Since suppv is compact this set is clearly
closed. To show that its open suppose (t
0
) suppv. By local exis-
tence there exist an interval ( +t
0
, +t
0
) and an integral curve,

1
(t), of v dened on this interval and taking the value
1
(t
0
) = (t
0
)
at p. However since v and w are -related
1
is also an integral curve
of w and so it has to coincide with on the interval ( +t
0
, +t
0
).
In particular, for t on this interval, (t) suppv, so the set (4.3.6)
is open.
168 Chapter 4. Forms on Manifolds
To conclude the proof of Theorem 4.3.14 we note that since R is
connected it follows that if (t
0
) X for some t
0
R then (t) X
for all t R, and hence is an integral curve of v. Thus in particular
every integral curve of v exists for all time, so v is complete.
Since w is complete it generates a one-parameter group of dieo-
morphisms, g
t
: O O, < t < having the property that the
curve
g
t
(p) =
p
(t) , < t <
is the unique integral curve of w with initial point,
p
(0) = p. But if
p X this curve is an integral curve of v, so the restriction
f
t
= g
t

X
is a one-parameter group of dieomorphisms of X with the property
that for p X the curve
f
t
(p) =
p
(t) , < t <
is the unique integral curve of v with initial point
p
(0) = p.
2. The exterior dierentiation operation
Let be a C

k-form on X and U X a parametrizable open


set. Given a parametrization,
0
: U
0
U we dene the exterior
derivative, d, of on X by the formula
(4.3.7) d = (
1
0
)

0
.
(Notice that since U
0
is an open subset of R
n
and

0
a k-form on
U
0
, the d on the right is well-dened.) We claim that this denition
doesnt depend on the choice of parametrization. To see this let
1
:
U
1
U be another parametrization of U and let : U
0
U
1
be
the dieomorphism,
1
1

0
. Then
0
=
1
and hence
d

0
= d

1
=

0
(
1
1
)

hence
(
1
0
)

0
= (
1
1
)

4.3 Vector elds and dierential forms on manifolds 169


as claimed. We can therefore, dene the exterior derivative, d, glob-
ally by dening it to be equal to (4.3.7) on every parametrizable open
set.
Its easy to see from the denition (4.3.7) that this exterior dier-
entiation operation inherits from the exterior dierentiation opera-
tion on open subsets of R
n
the properties (2.3.2) and (2.3.3) and that
for zero forms, i.e., C

functions, f : X R, df is the intrinsic df


dened in Section 2.1, i.e., for p X df
p
is the derivative of f
df
p
: T
p
X R
viewed as an element of
1
(T

p
X). Lets check that it also has the
property (2.5.12).
Theorem 4.3.16. Let X and Y be manifolds and f : X Y a C

map. Then for


k
(Y )
(4.3.8) f

d = df

.
Proof. For every p X well check that this equality holds in a
neighborhood of p. Let q = f(p) and let U and V be parametrizable
neighborhoods of p and q. Shrinking U if necessary we can assume
f(U) V . Given parametrizations
: U
0
U
and
: V
0
V
we get by composition a map
g : U
0
V
0
, g =
1
f
with the property g = f . Thus

d(f

) = d

(by denition of d)
= d(f )

= d( g)

= dg

)
= g

by (2.5.12)
= g

d (by denition of d)
=

d .
Hence df

= f

d.
170 Chapter 4. Forms on Manifolds
3. The interior product and Lie derivative operation
Given a k-form,
k
(X) and a C

vector eld, w, we will dene


the interior product
(4.3.9) (v)
k1
(X) ,
as in 2.4, by setting
((v))
p
= (v
p
)
p
and the Lie derivative
L
v
=
k
(X) (4.3.10)
by setting
L
v
= (v) d +d(v) . (4.3.11)
Its easily checked that these operations satisfy the identities (2.4.2)
(2.4.8) and (2.4.12)(2.4.13) (since, just as in 2.4, these identities
are deduced from the denitions (4.3.9) and (4.3.1) by purely formal
manipulations). Moreover, if v is complete and
f
t
: X X , < t <
is the one-parameter group of dieomorphisms of X generated by v
the Lie derivative operation can be dened by the alternative recipe
(4.3.12) L
v
=
_
d
dt
f

t

_
(t = 0)
as in (2.5.22). (Just as in 2.5 one proves this by showing that the
operation (4.3.12) has the properties (2.12) and (2.13) and hence
that it agrees with the operation (4.3.11) provided the two operations
agree on zero-forms.)
Exercises.
1. Let X R
3
be the paraboloid, x
3
= x
2
1
+x
2
2
and let w be
the vector eld
w = x
1

x
1
+x
2

x
2
+ 2x
3

x
3
.
4.3 Vector elds and dierential forms on manifolds 171
(a) Show that w is tangent to X and hence denes by
restriction a vector eld, v, on X.
(b) What are the integral curves of v?
2. Let S
2
be the unit 2-sphere, x
2
1
+ x
2
2
+ x
2
3
= 1, in R
3
and
let w be the vector eld
w = x
1

x
2
x
2

x
1
.
(a) Show that w is tangent to S
2
, and hence by restriction
denes a vector eld, v, on S
2
.
(b) What are the integral curves of v?
3. As in problem 2 let S
2
be the unit 2-sphere in R
3
and let
w be the vector eld
w =

x
3
x
3
_
x
1

x
1
+x
2

x
2
+x
3

x
3
_
(a) Show that w is tangent to S
2
and hence by restriction
denes a vector eld, v, on S
2
.
(b) What do its integral curves look like?
4. Let S
1
be the unit circle, x
2
1
+ x
2
2
= 1, in R
2
and let
X = S
1
S
1
in R
4
with dening equations
f
1
= x
2
1
+x
2
2
1 = 0
f
2
= x
2
3
+x
2
4
1 = 0 .
(a) Show that the vector eld
w = x
1

x
2
x
2

x
1
+
_
x
4

x
3
x
3

x
4
_
,
R, is tangent to X and hence denes by restriction a
vector eld, v, on X.
(b) What are the integral curves of v?
(c) Show that L
w
f
i
= 0.
172 Chapter 4. Forms on Manifolds
5. For the vector eld, v, in problem 4, describe the one-
parameter group of dieomorphisms it generates.
6. Let X and v be as in problem 1 and let f : R
2
X be the
map, f(x
1
, x
2
) = (x
1
, x
2
, x
2
1
+x
2
2
). Show that if u is the vector
eld,
u = x
1

x
1
+x
2

x
2
,
then f

u = v.
7. Let X be a submanifold of X in R
N
and let v and w be
the vector elds on X and U. Denoting by the inclusion map
of X into U, show that v and w are -related if and only if w
is tangent to X and its restriction to X is v.
8. Let X be a submanifold of R
N
and U an open subset of
R
N
containing X, and let v and w be the vector elds on X
and U. Denoting by the inclusion map of X into U, show that
v and w are -related if and only if w is tangent to X and its
restriction to X is v.
9. An elementary result in number theory asserts
Theorem 4.3.17. A number, R, is irrational if and only
if the set
{m+n, m and n intgers}
is a dense subset of R.
Let v be the vector eld in problem 4. Using the theorem above
prove that if /2 is irrational then for every integral curve,
(t), < t < , of v the set of points on this curve is a
dense subset of X.
10. Let X be an n-dimensional submanifold of R
N
. Prove that
a vector eld, v, on X is C

if and only if the map, (4.3.5) is


C

.
Hint: Let U be a parametrizable open subset of X and : U
0

U a parametrization of U. Composing with the inclusion map
: X R
N
one gets a map, : U R
N
. Show that if

v =

v
i

x
j
then
4.4 Orientations 173

f
i
=

i
x
j
v
j
where f
1
, . . . , f
N
are the coordinates of the map, f
v
, and
1
, . . . ,
N
the coordinates of .
11. Let v be a vector eld on X and : X R, a C

function.
Show that if the function
(4.3.13) L
v
= (v) d
is zero is constant along integral curves of v.
12. Suppose that : X R is proper. Show that if L
v
= 0,
v is complete.
Hint: For p X let a = (p). By assumption,
1
(a) is com-
pact. Let C

0
(X) be a bump function which is one on

1
(a) and let w be the vector eld, v. By Theorem 4.3.14,
w is complete and since
L
w
= (v) d = (v) d = 0
is constant along integral curves of w. Let (t), < t < ,
be the integral curve of w with initial point, (0) = p. Show
that is an integral curve of v.
4.4 Orientations
The last part of Chapter 4 will be devoted to the integral calculus
of forms on manifolds. In particular we will prove manifold versions
of two basic theorems of integral calculus on R
n
, Stokes theorem and
the divergence theorem, and also develop a manifold version of degree
theory. However, to extend the integral calculus to manifolds with-
out getting involved in horrendously technical orientation issues
we will conne ourselves to a special class of manifolds: orientable
manifolds. The goal of this section will be to explain what this term
means.
174 Chapter 4. Forms on Manifolds
Denition 4.4.1. Let X be an n-dimensional manifold. An orien-
tation of X is a rule for assigning to each p X an orientation of
T
p
X.
Thus by denition 1.9.1 one can think of an orientation as a label-
ing rule which, for every p X, labels one of the two components
of the set,
n
(T

p
X){0}, by
n
(T

p
X)
+
, which well henceforth call
the plus part of
n
(T

p
X), and the other component by
n
(T

p
X)

,
which well henceforth call the minus part of
n
(T

p
X).
Denition 4.4.2. An orientation of X is smooth if, for every p
X, there exists a neighborhood, U, of p and a non-vanishing n-form,

n
(U) with the property
(4.4.1)
q
=
n
(T

q
X)
+
for every q U.
Remark 4.4.3. If were given an orientation of X we can dene
another orientation by assigning to each p X the opposite orien-
tation to the orientation we already assigned, i.e., by switching the
labels on
n
(T

p
)
+
and
n
(T

p
)

. We will call this the reversed ori-


entation of X. We will leave for you to check as an exercise that
if X is connected and equipped with a smooth orientation, the only
smooth orientations of X are the given orientation and its reversed
orientation.
Hint: Given any smooth orientation of X the set of points where
it agrees with the given orientation is open, and the set of points
where it doesnt is also open. Therefore one of these two sets has to
be empty.
Note that if
n
(X) is a non-vanishing n-form one gets from
a smooth orientation of X by requiring that the labeling rule
above satisfy
(4.4.2)
p

n
(T

p
X)
+
for every p X. If has this property we will call a volume form.
Its clear from this denition that if
1
and
2
are volume forms on
X then
2
= f
2,1

1
where f
2,1
is an everywhere positive C

function.
4.4 Orientations 175
Example 1.
Open subsets, U of R
n
. We will usually assign to U its standard
orientation, by which we will mean the orientation dened by the
n-form, dx
1
dx
n
.
Example 2.
Let f : R
N
R
k
be a C

map. If zero is a regular value of f, the


set X = f
1
(0) is a submanifold of R
N
of dimension, n = N k, by
Theorem ??. Moreover, for p X, T
p
X is the kernel of the surjective
map
df
p
: T
p
R
N
T
o
R
k
so we get from df
p
a bijective linear map
(4.4.3) T
p
R
N
/T
p
X T
o
R
k
.
As explained in example 1, T
p
R
N
and T
o
R
k
have standard orien-
tations, hence if we require that the map (4.4.3) be orientation pre-
serving, this gives T
p
R
N
/T
p
X an orientation and, by Theorem 1.9.4,
gives T
p
X an orientation. Its intuitively clear that since df
p
varies
smoothly with respect to p this orientation does as well; however,
this fact requires a proof, and well supply a sketch of such a proof
in the exercises.
Example 3.
A special case of example 2 is the n-sphere
S
n
= {(x
1
, . . . , x
n+1
) R
n+1
, x
2
1
+ +x
2
n+1
= 1} ,
which acquires an orientation from its dening map, f : R
n+1
R,
f(x) = x
2
1
+ +x
2
n+1
1.
Example 4.
Let X be an oriented submanifold of R
N
. For every p X, T
p
X
sits inside T
p
R
N
as a vector subspace, hence, via the identication,
T
p
R
N
R
N
one can think of T
p
X as a vector subspace of R
N
. In
particular from the standard Euclidean inner product on R
N
one
gets, by restricting this inner product to vectors in T
p
X, an inner
product,
B
p
: T
p
X T
p
X R
176 Chapter 4. Forms on Manifolds
on T
p
X. Let
p
be the volume element in
n
(T

p
X) associated with
B
p
(see 1.9, exercise 10) and let =
X
be the non-vanishing n-form
on X dened by the assignment
p X
p
.
In the exercises at the end of this section well sketch a proof of the
following.
Theorem 4.4.4. The form,
X
, is C

and hence, in particular, is a


volume form. (We will call this form the Riemannian volume form.)
Example 5. The M obius strip. The M obius strip is a surface in
R
3
which is not orientable. It is obtained from the rectangle
R = {(x, y) ; 0 x 1 , 1 < y < 1}
by gluing the ends together in the wrong way, i.e., by gluing (1, y)
to (0, y) . It is easy to see that the M obius strip cant be oriented
by taking the standard orientation at p = (1, 0) and moving it along
the line, (t, 0), 0 t 1 to the point, (0, 0) (which is also the point,
p, after weve glued the ends of the rectangle together).
Well next investigate the compatibility question for dieomor-
phisms between oriented manifolds. Let X and Y be n-dimensional
manifolds and f : X Y a dieomorphism. Suppose both of these
manifolds are equipped with orientations. We will say that f is ori-
entation preserving if, for all p X and q = f(p) the linear map
df
p
: T
p
X T
q
Y
is orientation preserving. Its clear that if is a volume form on Y
then f is orientation preserving if and only if f

is a volume form
on X, and from (1.9.5) and the chain rule one easily deduces
Theorem 4.4.5. If Z is an oriented n-dimensional manifold and
g : Y Z a dieomorphism, then if both f and g are orientation
preserving, so is g f.
If f : X Y is a dieomorphism then the set of points, p X,
at which the linear map,
df
p
: T
p
X T
q
Y , q = f(p) ,
4.4 Orientations 177
is orientation preserving is open, and the set of points at which its
orientation reversing is open as well. Hence if X is connected, df
p
has
to be orientation preserving at all points or orientation reversing at
all points. In the latter case well say that f is orientation reversing.
If U is a parametrizable open subset of X and : U
0
U a
parametrization of U well say that this parametrization is an ori-
ented parametrization if is orientation preserving with respect to
the standard orientation of U
0
and the given orientation on U. No-
tice that if this parametrization isnt oriented we can convert it into
one that is by replacing every connected component, V
0
, of U
0
on
which isnt orientation preserving by the open set
(4.4.4) V

0
= {(x
1
, . . . , x
n
) R
n
, (x
1
, . . . , x
n1
, x
n
) V
0
}
and replacing by the map
(4.4.5) (x
1
, . . . , x
n
) = (x
1
, . . . , x
n1
, x
n
) .
If
i
: U
i
U, i = 0, 1, are oriented parametrizations of U and
: U
0
U
1
is the dieomorphism,
1
1

0
, then by the theorem
above is orientation preserving or in other words
(4.4.6) det
_

i
x
j
_
> 0
at every point on U
0
.
Well conclude this section by discussing some orientation issues
which will come up when we discuss Stokes theorem and the diver-
gence theorem in 4.6. First a denition.
Denition 4.4.6. An open subset, D, of X is a smooth domain if
(a) its boundary is an (n 1)-dimensional submanifold of X
and
(b) the boundary of D coincides with the boundary of the clo-
sure of D.
Examples.
1. The n-ball, x
2
1
+ + x
2
n
< 1, whose boundary is the sphere,
x
2
1
+ +x
2
n
= 1.
178 Chapter 4. Forms on Manifolds
2. The n-dimensional annulus,
1 < x
2
1
+ +x
2
n
< 2
whose boundary consists of the spheres,
x
2
1
+ +x
2
n
= 1 and x
2
1
+ +x
2
n
= 2 .
3. Let S
n1
be the unit sphere, x
2
1
+ + x
2
2
= 1 and let D =
R
n
S
n1
. Then the boundary of D is S
n1
but D is not a smooth
domain since the boundary of its closure is empty.
4. The simplest example of a smooth domain is the half-space
(4.4.7) H
n
= {(x
1
, . . . , x
n
) R
n
, x
1
< 0}
whose boundary
(4.4.8) {(x
1
, . . . , x
n
) R
n
, x
1
= 0}
we can identify with R
n1
via the map,
(x
2
, . . . , x
n
) R
n1
(0, x
2
, . . . , x
n
) .
We will show that every bounded domain looks locally like this
example.
Theorem 4.4.7. Let D be a smooth domain and p a boundary point
of D. Then there exists a neighborhood, U, of p in X, an open set,
U
0
, in R
n
and a dieomorphism, : U
0
U such that maps
U
0
H
n
onto U D.
Proof. Let Z be the boundary of D. First we will prove:
Lemma 4.4.8. For every p Z there exists an open set, U, in X
containing p and a parametrization
(4.4.9) : U
0
U
of U with the property
(4.4.10) (U
0
BdH
n
) = U Z .
4.4 Orientations 179
Proof. X is locally dieomorphic at p to an open subset of R
n
so it
suces to prove this assertion for X equal to R
n
. However, if Z is an
(n 1)-dimensional submanifold of R
n
then by ?? there exists, for
every p Z a neighborhood, U, of p in R
n
and a function, C

(U)
with the properties
x U Z (x) = 0 (4.4.11)
and
d
p
= 0 . (4.4.12)
Without loss of generality we can assume by (4.4.12) that
(4.4.13)

x
1
(p) = 0 .
Hence if : U R
n
is the map
(4.4.14) (x
1
, . . . , x
n
) = ((x), x
2
, . . . , x
n
)
(d)
p
is bijective, and hence is locally a dieomorphism at p.
Shrinking U we can assume that is a dieomorphism of U onto an
open set, U
0
. By (4.4.11) and (4.4.14) maps U Z onto U
0
BdH
n
hence if we take to be
1
, it will have the property (4.4.10).
We will now prove Theorem 4.4.4. Without loss of generality we
can assume that the open set, U
0
, in Lemma 4.4.8 is an open ball
with center at q BdH
n
and that the dieomorphism, maps q to
p. Thus for
1
(U D) there are three possibilities.
i.
1
(U D) = (R
n
BdH
n
) U
0
.
ii.
1
(U D) = (R
n
H
n
) U
0
.
or
iii.
1
(U D) = H
n
U
0
.
However, i. is excluded by the second hypothesis in Denition 4.4.6
and if ii. occurs we can rectify the situation by composing with
the map, (x
1
, . . . , x
n
) (x
1
, x
2
, . . . , x
n
).
180 Chapter 4. Forms on Manifolds
Denition 4.4.9. We will call an open set, U, with the properties
above a D-adapted parametrizable open set.
We will now show that if X is oriented and D X is a smooth
domain then the boundary, Z, of D acquires from X a natural ori-
entation. To see this we rst observe
Lemma 4.4.10. The dieomorphism, : U
0
U in Theorem 4.4.7
can be chosen to be orientation preserving.
Proof. If it is not, then by replacing with the dieomorphism,

(x
1
, . . . , x
n
) = (x
1
, . . . , x
n1
, x
n
), we get a D-adapted parametriza-
tion of U which is orientation preserving. (See (4.4.4)(4.4.5).)
Let V
0
= U
0
R
n1
be the boundary of U
0
H
n
. The restriction
of to V
0
is a dieomorphism of V
0
onto U Z, and we will orient
U Z by requiring that this map be an oriented parametrization.
To show that this is an intrinsic denition, i.e., doesnt depend on
the choice of , well prove
Theorem 4.4.11. If
i
: U
i
U, i = 0, 1, are oriented parametriza-
tions of U with the property

i
: U
i
H
n
U D
the restrictions of
i
to U
i
R
n1
induce compatible orientations on
U X.
Proof. To prove this we have to prove that the map,
1
1

0
, re-
stricted to U BdH
n
is an orientation preserving dieomorphism of
U
0
R
n1
onto U
1
R
n1
. Thus we have to prove the following:
Proposition 4.4.12. Let U
0
and U
1
be open subsets of R
n
and f :
U
0
U
1
an orientation preserving dieomorphism which maps U
0

H
n
onto U
1
H
n
. Then the restriction, g, of f to the boundary,
U
0
R
n1
, of U
0
H
n
is an orientation preserving dieomorphism,
g : U
0
R
n1
U
1
R
n1
.
Let f(x) = (f
1
(x), . . . , f
n
(x)). By assumption f
1
(x
1
, . . . , x
n
) is less
than zero if x
1
is less than zero and equal to zero if x
1
is equal to
zero, hence
f
1
x
1
(0, x
2
, . . . , x
n
) 0 (4.4.15)
and
4.4 Orientations 181
f
1
x
i
(0, x
2
, . . . , x
n
) = 0 , i > 1 (4.4.16)
Moreover, since g is the restriction of f to the set x
1
= 0
(4.4.17)
f
i
x
j
(0, x
2
, . . . , x
n
) =
g
i
x
j
(x
2
, . . . , x
1
)
for i, j 2. Thus on the set, x
1
= 0
(4.4.18) det
_
f
i
x
j
_
=
f
1
x
1
det
_
g
i
x
j
_
.
Since f is orientation preserving the left hand side of (4.4.18) is
positive at all points (0, x
2
, . . . , x
n
) U
0
R
n1
hence by (4.4.15) the
same is true for
f
1
x
1
and det
_
g
i
x
j
_
. Thus g is orientation preserving.
Remark 4.4.13. For an alternative proof of this result see exercise 8
in 3.2 and exercises 4 and 5 in 3.6.
We will now orient the boundary of D by requiring that for every
D-adapted parametrizable open set, U, the orientation of Z coin-
cides with the orientation of U Z that we described above. We will
conclude this discussion of orientations by proving a global version
of Proposition 4.4.12.
Proposition 4.4.14. Let X
i
, i = 1, 2, be an oriented manifold,
D
i
X
i
a smooth domain and Z
i
its boundary. Then if f is an
orientation preserving dieomorphism of (X
1
, D
1
) onto (X
2
, D
2
) the
restriction, g, of f to Z
1
is an orientation preserving dieomorphism
of Z
1
onto Z
2
.
Let U be an open subset of X
1
and : U
0
U an oriented
D
1
-compatible parametrization of U. Then if V = f(U) the map
f : U V is an oriented D
2
-compatible parametrization of V
and hence g : U Z
1
V Z
2
is orientation preserving.
Exercises.
182 Chapter 4. Forms on Manifolds
1. Let V be an oriented n-dimensional vector space, B an inner
product on V and e
i
V , i = 1, . . . , n an oriented orthonormal basis.
Given vectors, v
i
V , i = 1, . . . , n show that if
b
i,j
= B(v
i
, v
j
) (4.4.19)
and
v
i
=

a
j,i
e
j
, (4.4.20)
the matrices A = [a
i,j
] and B = [b
i,j
] satisfy the identity:
(4.4.21) B = A
t
A
and conclude that det B = (det A)
2
. (In particular conclude that
det B > 0.)
2. Let V and W be oriented n-dimensional vector spaces. Suppose
that each of these spaces is equipped with an inner product, and let
e
i
V , i = 1, . . . , n and f
i
W, i = 1, . . . , n be oriented orthonormal
bases. Show that if A : W V is an orientation preserving linear
mapping and Af
i
= v
i
then
(4.4.22) A

vol
V
= (det[b
i,j
])
1
2
vol
W
where vol
V
= e

1
e

n
, vol
W
= f

1
f

n
and [b
i,j
] is the
matrix (4.4.19).
3. Let X be an oriented n-dimensional submanifold of R
n
, U an
open subset of X, U
0
an open subset of R
n
and : U
0
U an
oriented parametrization. Let
i
, i = 1, . . . , N, be the coordinates of
the map
U
0
U R
N
.
the second map being the inclusion map. Show that if is the Rie-
mannian volume form on X then

= (det[
i,j
])
1
2
dx
1
dx
n
(4.4.23)
where

i,j
=
N

k=1

k
x
i

k
x
j
1 i, j n. (4.4.24)
4.4 Orientations 183
(Hint: For p U
0
and q = (p) apply exercise 2 with V = T
q
X,
W = T
p
R
n
, A = (d)
p
and v
i
= (d)
p
_

x
i
_
p
.) Conclude that is a
C

innity n-form and hence that it is a volume form.


4. Given a C

function f : R R, its graph


X = {(x, f(x)) , x R}
is a submanifold of R
2
and
: R X , x (x, f(x))
is a dieomorphism. Orient X by requiring that be orientation
preserving and show that if is the Riemannian volume form on X
then
(4.4.25)

=
_
1 +
_
df
dx
_
2
_1
2
dx.
Hint: Exercise 3.
5. Given a C

function f : R
n
R its graph
X = {(x, f(x)) , x R
n
}
is a submanifold of R
n+1
and
(4.4.26) : R
n
X , x (x, f(x))
is a dieomorphism. Orient X by requiring that is orientation
preserving and show that if is the Riemannian volume form on X
then
(4.4.27)

=
_
1 +
n

i=1
_
f
x
i
_
2
_1
2
dx
1
dx
n
.
Hints:
(a) Let v = (c
1
, . . . , c
n
) R
n
. Show that if C : R
n
R is the
linear mapping dened by the matrix [c
i
c
j
] then Cv = (

c
2
i
)v
and Cw = 0 if w v = 0 .
184 Chapter 4. Forms on Manifolds
(b) Conclude that the eigenvalues of C are
1
=

c
2
i
and

2
= =
n
= 0.
(c) Show that the determinant of I +C is 1 +

c
2
i
.
(d) Use (a)(c) to compute the determinant of the matrix
(4.4.24) where is the mapping (4.4.26).
6. Let V be an oriented N-dimensional vector space and
i
V

,
i = 1, . . . , k, k linearly independent vectors in V

. Dene
L : V R
k
to be the map v (
1
(v), . . . ,
k
(v)).
(a) Show that L is surjective and that the kernel, W, of L is
of dimension n = N k.
(b) Show that one gets from this mapping a bijective linear
mapping
(4.4.28) V/W R
K
and hence from the standard orientation on R
k
an induced ori-
entation on V/W and on W. Hint: 1.2, exercise 8 and Theo-
rem 1.9.4.
(c) Let be an element of
N
(V

). Show that there exists a



n
(V

) with the property
(4.4.29)
1

k
= .
Hint: Choose an oriented basis, e
1
, . . . , e
N
of V such that =
e

1
e

N
and
i
= e

i
for i = 1, . . . , k, and let = e

i+1

N
.
(d) Show that if is an element of
n
(V

) with the property

1

k
= 0
then there exist elements,
i
, of
n1
(V

) such that
(4.4.30) =

i

i
.
Hint: Same hint as in part (c).
4.4 Orientations 185
(e) Show that if =
i
, i = 1, 2, are elements of
n
(V

) with
the property (4.4.29) and : W V is the inclusion map then

1
=

2
. Hint: Let =
1

2
. Conclude from part (d)
that

= 0.
(f) Conclude that if is an element of
n
(V

) satisfying
(4.4.29) the element, =

, of
n
(W

) is intrinsically de-
ned independent of the choice of .
(g) Show that lies in
n
(V

)
+
.
7. Let U be an open subset of R
N
and f : U R
k
a C

map.
If zero is a regular value of f, the set, X = f
1
(0) is a manifold of
dimension n = N k. Show that this manifold has a natural smooth
orientation. Some suggestions:
(a) Let f = (f
1
, . . . , f
k
) and let
df
1
df
k
=

f
I
dx
I
summed over multi-indices which are strictly increasing. Show
that for every p X f
I
(p) = 0 for some multi-index, I =
(i
1
, . . . , i
k
), 1 i
1
< < i
k
N.
(b) Let J = (j
1
, . . . , j
n
), 1 j
1
< < j
n
N be the com-
plementary multi-index to I, i.e., j
r
= i
s
for all r and s. Show
that
df
1
df
k
dx
J
= f
I
dx
1
dx
N
and conclude that the n-form
=
1
f
I
dx
J
is a C

n-form on a neighborhood of p in U and has the prop-


erty:
(4.4.31) df
1
df
k
= dx
1
dx
N
.
(c) Let : X U be the inclusion map. Show that the assign-
ment
p X (

)
p
denes an intrinsic nowhere vanishing n-form

n
(X)
on X. Hint: Exercise 6.
186 Chapter 4. Forms on Manifolds
(d) Show that the orientation of X dened by coincides
with the orientation that we described earlier in this section.
Hint: Same hint as above.
8. Let S
n
be the n-sphere and : S
n
R
n+1
the inclusion map.
Show that if
n
(R
n+1
) is the n-form, =

(1)
i1
x
i
dx
1

dx
i
. . . dx
n+1
, the n-form


n
(S
n
) is the Riemannian volume
form.
9. Let S
n+1
be the (n + 1)-sphere and let
S
n+1
+
= {(x
1
, . . . , x
n+2
) S
n+1
, x
1
< 0}
be the lower hemi-sphere in S
n+1
.
(a) Prove that S
n+1
+
is a smooth domain.
(b) Show that the boundary of S
n+1
+
is S
n
.
(c) Show that the boundary orientation of S
n
agrees with the
orientation of S
n
in exercise 8.
4.5 Integration of forms over manifolds
In this section we will show how to integrate dierential forms over
manifolds. In what follows X will be an oriented n-dimensional man-
ifold and W an open subset of X, and our goal will be to make sense
of the integral
(4.5.1)
_
W

where is a compactly supported n-form. Well begin by showing


how to dene this integral when the support of is contained in
a parametrizable open set, U. Let U
0
be an open subset of R
n
and

0
: U
0
U a parametrization. As we noted in 4.4 we can assume
without loss of generality that this parametrization is oriented. Mak-
ing this assumption, well dene
(4.5.2)
_
W
=
_
W
0

4.5 Integration of forms over manifolds 187


where W
0
=
1
0
(U W). Notice that if

= fdx
1
dx
n
,
then, by assumption, f is in C

0
(U
0
). Hence since
_
W
0

0
=
_
W
0
fdx
1
. . . dx
n
and since f is a bounded continuous function and is compactly sup-
ported the Riemann integral on the right is well-dened. (See Ap-
pendix B.) Moreover, if
1
: U
1
U is another oriented parametriza-
tion of U and : U
0
U
1
is the map, =
1
1

0
then
0
=
1
,
so by Proposition 4.3.3

0
=

1
.
Moreover, by (4.3.5) is orientation preserving. Therefore since
W
1
= (W
0
) =
1
1
(U W)
Theorem 3.5.2 tells us that
(4.5.3)
_
W
1

1
=
_
W
0

0
.
Thus the denition (4.5.2) is a legitimate denition. It doesnt de-
pend on the parametrization that we use to dene the integral on
the right. From the usual additivity properties of the Riemann in-
tegral one gets analogous properties for the integral (4.5.2). Namely
for
i

n
c
(U), i = 1, 2
(4.5.4)
_
W

1
+
2
=
_
W

1
+
_
W

2
and for
n
c
(U) and c R
(4.5.5)
_
W
c = c
_
W
.
We will next show how to dene the integral (4.5.1) for any com-
pactly supported n-form. This we will do in more or less the same
way that we dened improper Riemann integrals in Appendix B: by
using partitions of unity. Well begin by deriving from the partition
of unity theorem in Appendix B a manifold version of this theorem.
188 Chapter 4. Forms on Manifolds
Theorem 4.5.1. Let
(4.5.6) U = {U

, I}
be a covering of X be open subsets. Then there exists a family of
functions,
i
C

0
(X), i = 1, 2, 3, . . . , with the properties
(a)
i
0.
(b) For every compact set, C X there exists a positive inte-
ger N such that if i > N, supp
i
C = .
(c)

i
= 1.
(d) For every i there exists an I such that supp
i
U

.
Remark 4.5.2. Conditions (a)(c) say that the
i
s are a partition
of unity and (d) says that this partition of unity is subordinate to the
covering (4.5.6).
Proof. To simplify the proof a bit well assume that X is a closed
subset of R
N
. For each U

choose an open subset, O

in R
N
with
(4.5.7) U

= O

X
and let O be the union of the O

s. By the theorem in Appendix B


that we cited above there exists a partition of unity,
i
C

0
(O),
i = 1, 2, . . . , subordinate to the covering of O by the O

s. Let
i
be
the restriction of
i
to X. Since the support of
i
is compact and X
is closed, the support of
i
is compact, so
i
C

0
(X) and its clear
that the
i
s inherit from the
i
s the properties (a)(d).
Now let the covering (4.5.6) be any covering of X by parametriz-
able open sets and let
i
C

0
(X), i = 1, 2, . . . , be a partition of
unity subordinate to this covering. Given
n
c
(X) we will dene
the integral of over W by the sum
(4.5.8)

i=1
_
W

i
.
Note that since each
i
is supported in some U

the individual sum-


mands in this sum are well-dened and since the support of is com-
pact all but nitely many of these summands are zero by part (b)
4.5 Integration of forms over manifolds 189
of Theorem 4.5.1. Hence the sum itself is well-dened. Lets show
that this sum doesnt depend on the choice of U and the
i
s. Let
U

be another covering of X by parametrizable open sets and

j
,
j = 1, 2, . . . , a partition of unity subordinate to U

. Then

j
_
W

j
=

j
_
W

i
(4.5.9)
=

j
_

i
_
W

_
by (4.5.4). Interchanging the orders of summation and resuming with
respect to the js this sum becomes

i
_
W

or

i
_
W

i
.
Hence

i
_
W

j
=

i
_
W

i
,
so the two sums are the same.
From (4.5.8) and (4.5.4) one easily deduces
Proposition 4.5.3. For
i

n
c
(X), i = 1, 2
(4.5.10)
_
W

1
+
2
=
_
W

1
+
_
W

2
and for
n
c
(X) and c R
(4.5.11)
_
W
c = c
_
W
.
The denition of the integral (4.5.1) depends on the choice of an
orientation of X, but its easy to see how it depends on this choice.
We pointed out in Section 4.4 that if X is connected, there is just one
way to orient it smoothly other than by its given orientation, namely
by reversing the orientation of T
p
at each point, p, and its clear from
190 Chapter 4. Forms on Manifolds
the denitions (4.5.2) and (4.5.8) that the eect of doing this is to
change the sign of the integral, i.e., to change
_
X
to
_
X
.
In the denition of the integral (4.5.1) weve allowed W to be an ar-
bitrary open subset of X but required to be compactly supported.
This integral is also well-dened if we allow to be an arbitrary
element of
n
(X) but require the closure of W in X to be compact.
To see this, note that under this assumption the sum (4.5.7) is still
a nite sum, so the denition of the integral still makes sense, and
the double sum on the right side of (4.5.9) is still a nite sum so its
still true that the denition of the integral doesnt depend on the
choice of partitions of unity. In particular if the closure of W in X
is compact we will dene the volume of W to be the integral,
(4.5.12) vol(W) =
_
W

vol
,
where
vol
is the Riemannian volume form and if X itself is compact
well dene its volume to be the integral
(4.5.13) vol(X) =
_
X

vol
.
Well next prove a manifold version of the change of variables
formula (3.5.1).
Theorem 4.5.4. Let X

and X be oriented n-dimensional manifolds


and f : X

X an orientation preserving dieomorphism. If W is


an open subset of X and W

= f
1
(W)
(4.5.14)
_
W

=
_
W

for all
n
c
(X).
Proof. By (4.5.8) the integrand of the integral above is a nite sum
of C

forms, each of which is supported on a parametrizable open


subset, so we can assume that itself as this property. Let V be
a parametrizable open set containing the support of and let
0
:
U V be an oriented parameterization of V . Since f is a dieomor-
phism its inverse exists and is a dieomorphism of X onto X
1
. Let
V

= f
1
(V ) and

0
= f
1

0
. Then

0
: U V

is an oriented
parameterization of V

. Moreover, f

0
=
0
so if W
0
=
1
0
(W)
we have
W
0
= (

0
)
1
(f
1
(W)) = (

0
)
1
(W

)
4.5 Integration of forms over manifolds 191
and by the chain rule we have

0
= (f

0
)

= (

0
)

hence
_
W
=
_
W
0

0
=
_
W
0
(

0
)

(f

) =
_
W

.
Exercise.
Show that if f : X

X is orientation reversing
(4.5.15)
_
W

=
_
W
.
Well conclude this discussion of integral calculus on manifolds
by proving a preliminary version of Stokes theorem.
Theorem 4.5.5. If is in
n1
c
(X) then
(4.5.16)
_
X
d = 0 .
Proof. Let
i
, i = 1, 2, . . . be a partition of unity with the property
that each
i
is supported in a parametrizable open set U
i
= U.
Replacing by
i
it suces to prove the theorem for
n1
c
(U).
Let : U
0
U be an oriented parametrization of U. Then
_
U
d =
_
U
0

d =
_
U
0
d

= 0
by Theorem 3.3.1.
Exercises.
1. Let f : R
n
R be a C

function and let


X = {(x, x
n+1
) R
n+1
, x
n+1
= f(x)}
be the graph of f. Lets orient X by requiring that the dieomor-
phism
: R
n
X , x (x, f(x))
192 Chapter 4. Forms on Manifolds
be orientation preserving. Given a bounded open set U in R
n
com-
pute the Riemannian volume of the image
X
U
= (U)
of U in X as an integral over U. Hint: 4.5, exercise 5.
2. Evaluate this integral for the open subset, X
U
, of the paraboloid,
x
3
= x
2
1
+x
2
2
, U being the disk x
2
1
+x
2
2
< 2.
3. In exercise 1 let : X R
n+1
be the inclusion map of X onto
R
n+1
.
(a) If
n
(R
n+1
) is the n-form, x
n+1
dx
1
dx
n
, what is the
integral of

over the set X


U
? Express this integral as an integral
over U.
(b) Same question for = x
2
n+1
dx
1
dx
n
.
(c) Same question for = dx
1
dx
n
.
4. Let f : R
n
(0, +) be a positive C

function, U a bounded
open subset of R
n
, and W the open set of R
n+1
dened by the
inequalities
0 < x
n+1
< f(x
1
, . . . , x
n
)
and the condition (x
1
, . . . , x
n
) U.
(a) Express the integral of the (n + 1)-form = x
n+1
dx
1

dx
n+1
over W as an integral over U.
(b) Same question for = x
2
n+1
dx
1
dx
n+1
.
(c) Same question for = dx
1
dx
n
5. Integrate the Riemannian area form
x
1
dx
2
dx
3
+x
2
dx
3
dx
1
+x
3
dx
1
dx
2
over the unit 2-sphere S
2
. (See 4.5, exercise 8.)
Hint: An easier problem: Using polar coordinates integrate =
x
3
dx
1
dx
2
over the hemisphere, x
3
=
_
1 x
2
1
x
2
2
, x
2
1
+x
2
2
< 1.
6. Let be the one-form

n
i=1
y
i
dx
i
in formula (2.7.2) and let
(t), 0 t 1, be a trajectory of the Hamiltonian vector eld (2.7.3).
What is the integral of over (t)?
4.6 Stokes theorem and the divergence theorem 193
4.6 Stokes theorem and the divergence theorem
Let X be an oriented n-dimensional manifold and D X a smooth
domain. We showed in 4.4 that if Z is the boundary of D it acquires
from D a natural orientation. Hence if : Z X is the inclusion
map and is in
n1
c
(X), the integral
_
Z

is well-dened. We will prove:


Theorem 4.6.1 (Stokes theorem). For
k1
c
(X)
(4.6.1)
_
Z

=
_
D
d.
Proof. Let
i
, i = 1, 2, . . . , be a partition of unity such that for each
i, the support of
i
is contained in a parametrizable open set, U
i
= U,
of one of the following three types:
(a) U Int D.
(b) U Ext D.
(c) There exists an open subset, U
0
, of R
n
and an oriented
D-adapted parametrization
(4.6.2) : U
0
U .
Replacing by the nite sum

i
it suces to prove (4.6.1) for
each
i
separately. In other words we can assume that the support
of itself is contained in a parametrizable open set, U, of type (a),
(b) or (c). But if U is of type (a)
_
D
d =
_
U
d =
_
X
d
and

= 0. Hence the left hand side of (4.6.1) is zero and, by


Theorem 4.5.5, the right hand side is as well. If U is of type (b) the
situation is even simpler:

is zero and the restriction of to D


is zero, so both sides of (4.6.1) are automatically zero. Thus one is
reduced to proving (4.6.1) when U is an open subset of type (c).
194 Chapter 4. Forms on Manifolds
In this case the restriction of the map (4.6.1) to U
0
BdH
n
is an
orientation preserving dieomorphism
: U
0
BdH
n
U Z (4.6.3)
and

Z
=
R
n1 (4.6.4)
where the maps =
Z
and

R
n1 : R
n1
R
n
are the inclusion maps of Z into X and BdH
n
into R
n
. (Here were
identifying BdH
n
with R
n1
.) Thus
_
D
d =
_
H
n

d =
_
H
n
d

and by (4.6.4)
_
Z

Z
=
_
R
n1

=
_
R
n1

R
n1

=
_
BdH
n

R
n1

.
Thus it suces to prove Stokes theorem with replaced by

, or,
in other words, to prove Stokes theorem for H
n
; and this we will now
do.
Stokes theorem for H
n
: Let
=

(1)
i1
f
i
dx
1

dx
i
dx
n
.
Then
d =

f
i
x
i
dx
1
dx
n
and
_
H
n
d =

i
_
H
n
f
i
x
i
dx
1
dx
n
.
4.6 Stokes theorem and the divergence theorem 195
We will compute each of these summands as an iterated integral
doing the integration with respect to dx
i
rst. For i > 1 the dx
i
integration ranges over the interval, < x
i
< and hence since
f
i
is compactly supported
_

f
i
x
i
dx
i
= f
i
(x
1
, . . . , x
i
, . . . , x
n
)

x
i
=+
x
i
=
= 0 .
On the other hand the dx
1
integration ranges over the integral,
< x
1
< 0 and
_
0

f
1
x
1
dx
1
= f(0, x
2
, . . . , x
n
) .
Thus integrating with respect to the remaining variables we get
(4.6.5)
_
H
n
d =
_
R
n1
f(0, x
2
, . . . , x
n
) dx
2
. . . dx
n
.
On the other hand, since

R
n1
x
1
= 0 and

R
n1
x
i
= x
i
for i > 1,

R
n1
= f
1
(0, x
2
, . . . , x
n
) dx
2
dx
n
so
_

R
n1
=
_
f(0, x
2
, . . . , x
n
) dx
2
. . . dx
n
. (4.6.6)
Hence the two sides, (4.6.5) and (4.6.6), of Stokes theorem are equal.
One important variant of Stokes theorem is the divergence theo-
rem: Let be in
n
c
(X) and let v be a vector eld on X. Then
L
v
= (v) d + d(v) = d(v) ,
hence, denoting by
Z
the inclusion map of Z into X we get from
Stokes theorem, with = (v):
Theorem 4.6.2 (The manifold version of the divergence theorem).
(4.6.7)
_
D
L
v
=
_
Z

Z
((v)) .
196 Chapter 4. Forms on Manifolds
If D is an open domain in R
n
this reduces to the usual divergence
theorem of multi-variable calculus. Namely if = dx
1
dx
n
and v =

v
i

x
i
then by (2.4.14)
L
v
dx
1
dx
n
= div(v) dx
1
dx
n
where
div(v) =

v
i
x
i
. (4.6.8)
Thus if Z is the boundary of D and
Z
the inclusion map of Z into
R
n
(4.6.9)
_
D
div(v) dx =
_
Z

Z
(
v
dx
1
dx
n
) .
The right hand side of this identity can be interpreted as the ux
of the vector eld, v, through the boundary of D. To see this let
f : R
n
R be a C

dening function for D, i.e., a function with


the properties
p D f(p) < 0 (4.6.10)
and
df
p
= 0 if p BdD. (4.6.11)
This second condition says that zero is a regular value of f and hence
that Z = BdD is dened by the non-degenerate equation:
p Z f(p) = 0 .
Let w be the vector eld
_

_
f
x
i
_
2
_
1

f
i
x
i

x
i
.
In view of (4.6.11) this vector eld is well-dened on a neighborhood,
U, of Z and satises
(4.6.12) (w) df = 1 .
4.6 Stokes theorem and the divergence theorem 197
Now note that since df dx
1
dx
n
= 0
0 = (w)(df dx
1
dx
n
)
= ((w) df) dx
1
dx
n
df (w) dx
1
dx
n
= dx
1
dx
n
df (w) dx
1
dx
n
,
hence letting be the (n 1)-form (w) dx
1
dx
n
we get the
identity
(4.6.13) dx
1
dx
n
= df
and by applying the operation, (v), to both sides of (4.6.13) the
identity
(4.6.14) (v) dx
1
dx
n
= (L
v
f) df (v) .
Let
Z
=

Z
be the restriction of to Z. Since

Z
= 0,

Z
df = 0
and hence by (4.6.14)

Z
((v) dx
1
dx
n
) =

Z
(L
v
f)
Z
,
and the formula (4.6.9) now takes the form
(4.6.15)
_
D
div(v) dx =
_
Z
L
v
f
Z
where the term on the right is by denition the ux of v through Z.
In calculus books this is written in a slightly dierent form. Letting

Z
=
_

_
f
x
i
_
2
_1
2

Z
and letting
n =
_

_
f
x
i
_
2
_

1
2
_
f
x
1
, ,
f
x
n
_
and
v = (v
1
, . . . , v
n
)
we have
198 Chapter 4. Forms on Manifolds
L
v

Z
= (n v)
Z
and hence
(4.6.16)
_
D
div(v) dx =
_
Z
(n v)
Z
.
In three dimensions
Z
is just the standard innitesimal element of
area on the surface Z and n
p
the unit outward normal to Z at p,
so this version of the divergence theorem is the version one nds in
most calculus books.
As an application of Stokes theorem, well give a very short alter-
native proof of the Brouwer xed point theorem. As we explained in
3.6 the proof of this theorem basically comes down to proving
Theorem 4.6.3. Let B
n
be the closed unit ball in R
n
and S
n1
its
boundary. Then the identity map
id
S
n1 : S
n1
S
n1
cant be extended to a C

map
f : B
n
S
n1
.
Proof. Suppose that f is such a map. Then for every n 1-form,

n1
(S
n1
),
(4.6.17)
_
B
n
df

=
_
S
n1
(
S
n1)

.
But df

= f

d = 0 since is an (n1)-form and S


n1
is an (n
1)-dimensional manifold, and since f is the identity map on S
n1
,
(
S
n1
)

= (f
S
n1)

= . Thus for every


n1
(S
n1
),
(4.6.17) says that the integral of over S
n1
is zero. Since there are
lots of (n 1)-forms for which this is not true, this shows that a
mapping, f, with the property above cant exist.
Exercises.
1. Let B
n
be the open unit ball in R
n
and S
n1
the unit (n
1)-sphere, Show that volume (S
n1
) = nvolume (B
n
). Hint: Apply
4.6 Stokes theorem and the divergence theorem 199
Stokes theorem to the (n1)-form =

(1)
i1
x
i
dx
1


dx
i

dx
n
and note (4.4, exercise 9) that is the Riemannian volume
form of S
n1
.
2. Let D R
n
be a smooth domain with boundary Z. Show
that there exists a neighborhood, U, of Z in R
n
and a C

dening
function, g : U R for D with the properties
(I) p U D g(p) < 0.
and
(II) dg
p
= 0 if p Z
Hint: Deduce from Theorem ?? that a local version of this result
is true. Show that you can cover Z by a family
U = {U

, I}
of open subsets of R
n
such that for each there exists a function,
g

: U

R, with properties (I) and (II). Now let


i
, i = 1, 2, . . . ,
be a partition of unity and let g =

i
g

i
where supp
i
U

i
.
3. In exercise 2 suppose Z is compact. Show that there exists a
global dening function, f : R
n
R for D with properties (I) and
(II). Hint: Let C

0
(U), 0 1, be a function which is one on
a neighborhood of Z, and replace g by the function
f =
_

_
g + (1 ) on ext D
g on Z
g(1 ) on int D.
4. Show that the form L
v
f
Z
in formula (4.6.15) doesnt depend
on what choice we make of a dening function, f, for D. Hints:
(a) Show that if g is another dening function then, at p Z,
df
p
= dg
p
, where is a positive constant.
(b) Show that if one replaces df
p
by (dg)
p
the rst term in the
product, (L
v
f)(p)(
Z
)
p
changes by a factor, , and the second
term by a factor 1/.
5. Show that the form,
Z
, is intrinsically dened in the sense
that if is any (n 1)-form satisfying (4.6.13),
Z
is equal to

Z
.
Hint: 4.5, exercise 7.
200 Chapter 4. Forms on Manifolds
6. Show that the form,
Z
, in the formula (4.6.16) is the Rieman-
nian volume form on Z.
7. Show that the (n 1)-form
= (x
2
1
+ +x
2
n
)
n

(1)
r1
x
r
dx
1


dx
r
dx
n
is closed and prove directly that Stokes theorem holds for the annulus
a < x
2
1
+ + x
2
n
< b by showing that the integral of over the
sphere, x
2
1
+ + x
2
n
= a, is equal to the integral over the sphere,
x
2
1
+ +x
2
n
= b.
8. Let f : R
n1
R be an everywhere positive C

function and
let U be a bounded open subset of R
n1
. Verify directly that Stokes
theorem is true if D is the domain
0 < x
n
< f(x
1
, . . . , x
n1
) , (x
1
, . . . , x
n1
) U
and an (n 1)-form of the form
(x
1
, . . . , x
n
) dx
1
dx
n1
where is in C

0
(R
n
).
9. Let X be an oriented n-dimensional manifold and v a vector
eld on X which is complete. Verify that for
n
c
(X)
_
X
L
v
= 0 ,
(a) directly by using the divergence theorem,
(b) indirectly by showing that
_
X
f

t
=
_
X

where f
t
: X X, < t < , is the one-parameter group
of dieomorphisms of X generated by v.
10. Let X be an oriented n-dimensional manifold and D X a
smooth domain whose closure is compact. Show that if Z is the
boundary of D and g : Z Z a dieomorphism, g cant be extended
to a smooth map, f : D Z.
4.7 Degree theory on manifolds 201
4.7 Degree theory on manifolds
In this section well show how to generalize to manifolds the results
about the degree of a proper mapping that we discussed in Chap-
ter 3. Well begin by proving the manifold analogue of Theorem 3.3.1.
Theorem 4.7.1. Let X be an oriented connected n-dimensional
manifold and
n
c
(X) a compactly supported n-form. Then the
following are equivalent
(a)
_
X
= 0.
(b) = d for some
n1
c
(X).
Weve already veried the assertion (b) (a) (see Theorem ??),
so what is left to prove is the converse assertion. The proof of this
is more or less identical with the proof of the (a) (b) part of
Theorem 3.2.1:
Step 1. Let U be a connected parametrizable open subset of X.
If
n
c
(U) has property (a), then = d for some
n1
c
(U).
Proof. Let : U
0
U be an oriented parametrization of U. Then
_
U
0

=
_
X
= 0
and since U
0
is a connected open subset of R
n
,

= d for some

n1
c
(U
0
) by Theorem 3.3.1. Let = (
1
)

. Then d =
(
1
)

d = .
Step 2. Fix a base point, p
0
X and let p be any point of X. Then
there exists a collection of connected parametrizable open sets, W
i
,
i = 1, . . . , N with p
0
W
1
and p W
N
such that, for 1 i N 1,
the intersection of W
i
and W
i+1
is non-empty.
Proof. The set of points, p X, for which this assertion is true is
open and the set for which it is not true is open. Moreover, this
assertion is true for p = p
0
.
202 Chapter 4. Forms on Manifolds
Step 3. We deduce Theorem 4.7.1 from a slightly stronger result.
Introduce an equivalence relation on
n
c
(X) by declaring that two
n-forms,
1
and
2
, in
n
c
(X) are equivalent if
1

2
d
n1
x
(X).
Denote this equivalence relation by a wiggly arrow:
1

2
. We will
prove
Theorem 4.7.2. For
1
and
2

n
c
(X) the following are equiva-
lent
(a)
_
X

1
=
_
X

2
(b)
1

2
.
Applying this result to a form,
n
c
(X), whose integral is zero,
we conclude that 0, which means that = d for some

n1
c
(X). Hence Theorem 4.7.2 implies Theorem 4.7.1. Conversely,
if
_
X

1
=
_
X

2
. Then
_
X
(
1

2
) = 0, so
1

2
= d for some

n
c
(X). Hence Theorem 4.7.1 implies Theorem 4.7.2.
Step 4. By a partition of unity argument it suces to prove The-
orem 4.7.2 for
1

n
c
(U
1
) and
2

n
c
(U
2
) where U
1
and U
2
are
connected parametrizable open sets. Moreover, if the integrals of
1
and
2
are zero then
i
= d
i
for some
i

n
c
(U
i
) by step 1, so in
this case, the theorem is true. Suppose on the other hand that
_
X

1
=
_
X

2
= c = 0 .
Then dividing by c, we can assume that the integrals of
1
and
2
are both equal to 1.
Step 5. Let W
i
, i = 1, . . . , N be, as in step 2, a sequence of
connected parametrizable open sets with the property that the in-
tersections, W
1
U
1
, W
N
U
2
and W
i
W
i+1
, i = 1, . . . , N 1, are
all non-empty. Select n-forms,
0

n
c
(U
1
W
1
),
N

n
c
(W
N
U
2
)
and
i

n
c
(W
i
W
i+1
), i = 1, . . . , N 1 such that the integral of
each
i
over X is equal to 1. By step 1 Theorem 4.7.1 is true for
U
1
, U
2
and the W
i
s, hence Theorem 4.7.2 is true for U
1
, U
2
and the
W
i
s, so

1

0

1

N

2
and thus
1

2
.
4.7 Degree theory on manifolds 203
Just as in (3.4.1) we get as a corollary of the theorem above the
following denitiontheorem of the degree of a dierentiable map-
ping:
Theorem 4.7.3. Let X and Y be compact oriented n-dimensional
manifolds and let Y be connected. Given a proper C

mapping, f :
X Y , there exists a topological invariant, deg(f), with the dening
property:
(4.7.1)
_
X
f

= deg f
_
Y
.
Proof. As in the proof of Theorem 3.4.1 pick an n-form,
0

n
c
(Y ),
whose integral over Y is one and dene the degree of f to be the
integral over X of f

0
, i.e., set
(4.7.2) deg(f) =
_
X
f

0
.
Now let be any n-form in
n
c
(Y ) and let
(4.7.3)
_
Y
= c .
Then the integral of c
0
over Y is zero so there exists an (n1)-
form, , in
n1
c
(Y ) for which c
0
= d. Hence f

= cf

0
+
df

, so
_
X
f

= c
_
X
f

0
= deg(f)
_
Y

by (4.7.2) and (4.7.3).


Its clear from the formula (4.7.1) that the degree of f is indepen-
dent of the choice of
0
. (Just apply this formula to any
n
c
(Y )
having integral over Y equal to one.) Its also clear from (4.7.1) that
degree behaves well with respect to composition of mappings:
Theorem 4.7.4. Let Z be an oriented, connected n-dimensional
manifold and g : Y Z a proper C

map. Then
(4.7.4) deg g f = (deg f)(deg g) .
204 Chapter 4. Forms on Manifolds
Proof. Let be an element of
n
c
(Z) whose integral over Z is one.
Then
deg g f =
_
X
(g f)

=
_
X
f

= deff
_
Y
g

= (deg f)(deg g) .
We will next show how to compute the degree of f by generalizing
to manifolds the formula for deg(f) that we derived in 3.6.
Denition 4.7.5. A point, p X is a critical point of f if the map
(4.7.5) df
p
: T
p
X T
f(p)
Y
is not bijective.
Well denote by C
f
the set of all critical points of f, and well call
a point q Y a critical value of f if it is in the image, f(C
f
), of
C
f
and a regular value if its not. (Thus the set of regular values
is the set, Y f(C
f
).) If q is a regular value, then as we observed
in 3.6, the map (4.7.5) is bijective for every p f
1
(q) and hence
by Theorem 4.2.5, f maps a neighborhood U
p
of p dieomorphically
onto a neighborhood, V
p
, of q. In particular, U
p
f
1
(q) = p. Since f
is proper the set f
1
(q) is compact, and since the sets, U
p
, are a cov-
ering of f
1
(q), this covering must be a nite covering. In particular
the set f
1
(q) itself has to be a nite set. As in 2.6 we can shrink
the U
p
s so as to insure that they have the following properties:
(i) Each U
p
is a parametrizable open set.
(ii) U
p
U
p
is empty for p = p

.
(iii) f(U
p
) = f(U
p
) = V for all p and p

.
(iv) V is a parametrizable open set.
(v) f
1
(V ) =

U
p
, p f
1
(q).
To exploit these properties let be an n-form in
n
c
(V ) with
integral equal to 1. Then by (v):
deg(f) =
_
X
f

p
_
Up
f

.
4.7 Degree theory on manifolds 205
But f : U
p
V is a dieomorphism, hence by (4.5.14) and (4.5.15)
_
Up
f

=
_
V

if f : U
p
V is orientation preserving and
_
Up
f

=
_
V

if f : U
p
V is orientation reversing. Thus weve proved
Theorem 4.7.6. The degree of f is equal to the sum
(4.7.6)

pf
1
(q)

p
where
p
= +1 if the map (4.7.5) is orientation preserving and
p
=
1 if it is orientation reversing.
We will next show that Sards Theorem is true for maps between
manifolds and hence that there exist lots of regular values. We rst
observe that if U is a parametrizable open subset of X and V a
parametrizable open neighborhood of f(U) in Y , then Sards Theo-
rem is true for the map, f : U V since, up to dieomorphism, U
and V are just open subsets of R
n
. Now let q be any point in Y , let B
be a compact neighborhood of q, and let V be a parametrizable open
set containing B. Then if A = f
1
(B) it follows from Theorem 3.4.2
that A can be covered by a nite collection of parametrizable open
sets, U
1
, . . . , U
N
such that f(U
i
) V . Hence since Sards Theorem
is true for each of the maps f : U
i
V and f
1
(B) is contained in
the union of the U
i
s we conclude that the set of regular values of f
intersects the interior of B in an open dense set. Thus, since q is an
arbitrary point of Y , weve proved
Theorem 4.7.7. If X and Y are n-dimensional manifolds and f :
X Y is a proper C

map the set of regular values of f is an open


dense subset of Y .
Since there exist lots of regular values the formula (4.7.6) gives us
an eective way of computing the degree of f. Well next justify our
assertion that deg(f) is a topological invariant of f. To do so, lets
generalize to manifolds the Denition 2.5.1, of a homotopy between
C

maps.
206 Chapter 4. Forms on Manifolds
Denition 4.7.8. Let X and Y be manifolds and f
i
: X Y ,
i = 0, 1, a C

map. A C

map
(4.7.7) F : X [0, 1] Y
is a homotopy between f
0
and f
1
if, for all x X, F(x, 0) = f
0
(x)
and F(x, 1) = f
1
(x). Moreover, if f
0
and f
1
are proper maps, the
homotopy, F, is a proper homotopy if it is proper as a C

map,
i.e., for every compact set, C, of Y , F
1
(C) is compact.
Lets now prove the manifold analogue of Theorem 3.6.8.
Theorem 4.7.9. Let X and Y be oriented n-dimensional manifolds
and let Y be connected. Then if f
i
: X Y , i = 0, 1,, is a proper
map and the map (4.7.4) is a property homotopy, the degrees of these
maps are the same.
Proof. Let be an n-form in
n
c
(Y ) whose integral over Y is equal
to 1, and let C be the support of . Then if F is a proper homotopy
between f
0
and f
1
, the set, F
1
(C), is compact and its projection
on X
(4.7.8) {x X ; (x, t) F
1
(C) for some t [0, 1]}
is compact. Let
f
t
: X Y
be the map: f
t
(x) = F(x, t). By our assumptions on F, f
t
is a proper
C

map. Moreover, for all t the n-form, f

t
is a C

function of t
and is supported on the xed compact set (4.7.8). Hence its clear
from the Denition 4.6.8 that the integral
_
X
f

t

is a C

function of t. On the other hand this integral is by denition


the degree of f
t
and hence by Theorem 4.7.3 is an integer, so it
doesnt depend on t. In particular, deg(f
0
) = deg(f
1
).
Exercises.
1. Let f : R R be the map, x x
n
. Show that deg(f) = 0 if n
is even and 1 if n is odd.
4.7 Degree theory on manifolds 207
2. Let f : R R be the polynomial function,
f(x) = x
n
+a
1
x
n1
+ +a
n1
x +a
n
,
where the a
i
s are in R. Show that if n is even, deg(f) = 0 and if n
is odd, deg(f) = 1.
3. Let S
1
be the unit circle
{e
i
, 0 < 2}
in the complex plane and let f : S
1
S
1
be the map, e
i
e
iN
,
N being a positive integer. Whats the degree of f?
4. Let S
n1
be the unit sphere in R
n
and : S
n1
S
n1
the
antipodal map, x x. Whats the degree of ?
5. Let A be an element of the group, O(n) of orthogonal n n
matrices and let
f
A
: S
n1
S
n1
be the map, x Ax. Whats the degree of f
A
?
6. A manifold, Y , is contractable if for some point, p
0
Y , the
identity map of Y onto itself is homotopic to the constant map,
f
p
0
: Y Y , f
p
0
(y) = p
0
. Show that if Y is an oriented contractable
n-dimensional manifold and X an oriented connected n-dimensional
manifold then for every proper mapping f : X Y deg(f) = 0. In
particular show that if n is greater than zero and Y is compact then
Y cant be contractable. Hint: Let f be the identity map of Y onto
itself.
7. Let X and Y be oriented connected n-dimensional manifolds
and f : X Y a proper C

map. Show that if deg(f) = 0 f is


surjective.
8. Using Sards Theorem prove that if X and Y are manifolds of
dimension k and , with k < and f : X Y is a proper C

map,
then the complement of the image of X in Y is open and dense.
Hint: Let r = k and apply Sards Theorem to the map
g : X S
r
Y , g(x, a) = f(x) .
9. Prove that the sphere, S
2
, and the torus, S
1
S
2
, are not
dieomorphic.
208 Chapter 4. Forms on Manifolds
4.8 Applications of degree theory
The purpose of this section will be to describe a few typical ap-
plications of degree theory to problems in analysis, geometry and
topology. The rst of these applications will be yet another variant
of the Brouwer xed point theorem.
Application 1. Let X be an oriented (n+1)-dimensional manifold,
D X a smooth domain and Z the boundary of D. Assume that
the closure,

D = Z D, of D is compact (and in particular that X
is compact).
Theorem 4.8.1. Let Y be an oriented connected n-dimensional
manifold and f : Z Y a C

map. Suppose there exists a C

map, F :

D Y whose restriction to Z is f. Then the degree of f
is zero.
Proof. Let be an element of
n
c
(Y ). Then d = 0, so dF

=
F

d = 0. On the other hand if : Z X is the inclusion map,


_
D
dF

=
_
Z

=
_
Z
f

= deg(f)
_
Y

by Stokes theorem since F = f. Hence deg(f) has to be zero.


Application 2. (a non-linear eigenvalue problem)
This application is a non-linear generalization of a standard theo-
rem in linear algebra. Let A : R
n
R
n
be a linear map. If n is even,
A may not have real eigenvalues. (For instance for the map
A : R
2
R
2
, (x, y) (y, x)
the eigenvalues of A are

1.) However, if n is odd it is a standard


linear algebra fact that there exists a vector, v R
n
{0}, and a
R such that Av = v. Moreover replacing v by
v
|v|
one can
assume that |v| = 1. This result turns out to be a special case of a
much more general result. Let S
n1
be the unit (n1)-sphere in R
n
and let f : S
n1
R
n
be a C

map.
Theorem 4.8.2. There exists a vector, v S
n1
and a number
R such that f(v) = v.
4.8 Applications of degree theory 209
Proof. The proof will be by contradiction. If the theorem isnt true
the vectors, v and f(v), are linearly independent and hence the vector
(4.8.1) g(v) = f(v) (f(v) v)v
is non-zero. Let
(4.8.2) h(v) =
g(v)
|g(v)|
.
By (4.8.1)(4.8.2), |v| = |h(v)| = 1 and v h(v) = 0, i.e., v and h(v)
are both unit vectors and are perpendicular to each other. Let
(4.8.3)
t
: S
n1
S
n1
, 0 t 1
be the map
(4.8.4)
t
(v) = (cos t)v + (sin t)h(v) .
For t = 0 this map is the identity map and for t = 1, it is the
antipodal map, (v) = v, hence (4.8.3) asserts that the identity map
and the antipodal map are homotopic and therefore that the degree
of the antipodal map is one. On the other hand the antipodal map
is the restriction to S
n1
of the map, (x
1
, . . . , x
n
) (x
1
, . . . , x
n
)
and the volume form, , on S
n1
is the restriction to S
n1
of the
(n 1)-form
(4.8.5)

(1)
i1
x
i
dx
i

dx
i
dx
n
.
If we replace x
i
by x
i
in (4.8.5) the sign of this form changes by
(1)
n
hence

= (1)
n
. Thus if n is odd, is an orientation
reversing dieomorphism of S
n1
onto S
n1
, so its degree is 1,
and this contradicts what we just deduced from the existence of the
homotopy (4.8.4).
From this argument we can deduce another interesting fact about
the sphere, S
n1
, when n1 is even. For v S
n1
the tangent space
to S
n1
at v is just the space,
{(v, w) ; w R
n
, v w = 0} ,
210 Chapter 4. Forms on Manifolds
so a vector eld on S
n1
can be viewed as a function, g : S
n1
R
n
with the property
(4.8.6) g(v) v = 0
for all v S
n1
. If this function is non-zero at all points, then,
letting h be the function, (4.8.2), and arguing as above, were led to
a contradiction. Hence we conclude:
Theorem 4.8.3. If n1 is even and v is a vector eld on the sphere,
S
n1
, then there exists a point p S
n1
at which v(p) = 0.
Note that if n 1 is odd this statement is not true. The vector
eld
(4.8.7) x
1

x
2
x
2

x
1
+ +x
2n1

x
2n
x
2n

x
2n1
is a counterexample. It is nowhere vanishing and at p S
n1
is
tangent to S
n1
.
Application 3. (The JordanBrouwer separation theorem.) Let X
be a compact oriented (n 1)-dimensional submanifold of R
n
. In
this subsection of 4.8 well outline a proof of the following theorem
(leaving the details as a string of exercises).
Theorem 4.8.4. If X is connected, the complement of X : R
n
X
has exactly two connected components.
This theorem is known as the Jordan-Brouwer separation theorem
(and in two dimensions as the Jordan curve theorem). For simple,
easy to visualize, submanifolds of R
n
like the (n1)-sphere this result
is obvious, and for this reason its easy to be misled into thinking of
it as being a trivial (and not very interesting) result. However, for
submanifolds of R
n
like the curve in R
2
depicted in the gure below
its much less obvious. (In ten seconds or less, is the point, p, in this
gure inside this curve or outside?)
Figure . Guillemin-Pollack, p. 86 g. 2-19
To determine whether a point, p R
n
X is inside X or outside
X, one needs a topological invariant to detect the dierence, and
such an invariant is provided by the winding number.
4.8 Applications of degree theory 211
Denition 4.8.5. For p R
n
X let
(4.8.8)
p
: X S
n1
be the map
(4.8.9)
p
(x) =
x p
|x p|
.
The winding number of X about p is the degree of this map.
Denoting this number by W(X, p) we will show below that W(X, p) =
0 if p is outside X and W(X, p) = 1 (depending on the orientation
of X) if p is inside X, and hence that the winding number tells us
which of the two components of R
n
X, p is contained in.
Exercise 1.
Let U be a connected component of R
n
X. Show that if p
0
and
p
1
are in U, W(X, p
0
) = W(X, p
1
).
Hints:
(a) First suppose that the line segment,
p
t
= (1 t)p
0
+tp
1
, 0 t 1
lies in U. Conclude from the homotopy invariance of degree
that W(X, p
0
) = W(X, p
t
) = W(X, p
1
).
(b) Show that there exists a sequence of points
q
i
, i = 1, . . . , N , q
i
U ,
with q
1
= p
0
and q
N
= p
1
, such that the line segment joining
q
i
to q
i+1
is in U.
Exercise 2.
Show that R
n
X has at most two connected components.
Hints:
(a) Show that if q is in X there exists a small -ball, B

(q),
centered at q such that B

(q) X has two components. (See


Theorem ??.
212 Chapter 4. Forms on Manifolds
(b) Show that if p is in R
n
X, there exists a sequence
q
i
, i = 1, . . . , N , q
i
R
n
X ,
such that q
1
= p, q
N
B

(q) and the line segments joining q


i
to q
i+1
are in R
n
X.
Exercise 3.
For v S
n1
, show that x X is in
1
p
(v) if and only if x lies
on the ray
(4.8.10) p +tv , 0 < t < .
Exercise 4.
Let x X be a point on this ray. Show that
(4.8.11) (d
p
)
x
: T
p
X T
v
S
n1
is bijective if and only if v / T
p
X, i.e., if and only if the ray (4.8.10)
is not tangent to X at x. Hint:
p
: X S
n1
is the composition
of the maps

p
: X R
n
{0} , x x p , (4.8.12)
and
: R
n
{0} S
n1
, y
y
|y|
. (4.8.13)
Show that if (y) = v, then the kernel of (d)
g
, is the one-dimensional
subspace of R
n
spanned by v. Conclude that if y = xp and v = y/|y|
the composite map
(d
p
)
x
= (d)
y
(d
p
)
x
is bijective if and only if v / T
x
X.
Exercise 5.
From exercises 3 and 4 conclude that v is a regular value of
p
if
and only if the ray (4.8.10) intersects X in a nite number of points
and at each point of intersection is not tangent to X at that point.
4.8 Applications of degree theory 213
Exercise 6.
In exercise 5 show that the map (4.8.11) is orientation preserving
if the orientations of T
x
X and v are compatible with the standard
orientation of T
p
R
n
. (See 1.9, exercise 5.)
Exercise 7.
Conclude that deg(
p
) counts (with orientations) the number of
points where the ray (4.8.10) intersects X.
Exercise 8.
Let p
1
R
n
X be a point on the ray (4.8.10). Show that if
v S
n1
is a regular value of
p
, it is a regular value of
p
1
and
show that the number
deg(
p
) deg(
p
1
) = W(X, p) W(X, p
1
)
counts (with orientations) the number of points on the ray lying
between p and p
1
. Hint: Exercises 5 and 7.
Exercise 8.
Let x X be a point on the ray (4.8.10). Suppose x = p + tv.
Show that if is a small positive number and
p

= p + (t )v
then
W(X, p
+
) = W(X, p

) 1 ,
and from exercise 1 conclude that p
+
and p

lie in dierent compo-


nents of R
n
X. In particular conclude that R
n
X has exactly two
components.
Exercise 9.
Finally show that if p is very large the dierence

p
(x)
p
|p|
, x X ,
is very small, i.e.,
p
is not surjective and hence the degree of
p
is
zero. Conclude that for p R
n
X, p is in the unbounded compo-
nent of R
n
X if W(X, p) = 0 and in the bounded component if
W(X, p) = 1 (the depending on the orientation of X).
214 Chapter 4. Forms on Manifolds
Notice, by the way, that the proof of JordanBrouwer sketched
above gives us an eective way of deciding whether the point, p, in
Figure 4.8, is inside X or outside X. Draw a non-tangential ray from
p as in Figure 4.9.2. If it intersects X in an even number of points,
p is outside X and if it intersects X is an odd number of points p
inside.


Figure 4.9.2.
Application 3. (The GaussBonnet theorem.) Let X R
n
be
a compact, connected, oriented (n 1)-dimensional submanifold.
By the JordanBrouwer theorem X is the boundary of a bounded
smooth domain, so for each x X there exists a unique outward
pointing unit normal vector, n
x
. The Gauss map
: X S
n1
4.8 Applications of degree theory 215
is the map, x n
x
. Let be the Riemannian volume form of S
n1
,
or, in other words, the restriction to S
n1
of the form,

(1)
i1
x
i
dx
1

dx
i
dx
n
,
and let
X
be the Riemannian volume form of X. Then for each
p X
(4.8.14) (

)
p
= K(p)(
X
)
q
where K(p) is the scalar curvature of X at p. This number measures
the extent to which X is curved at p. For instance, if X
a
is the
circle, |x| = a in R
2
, the Gauss map is the map, p p/a, so for all
p, K
a
(p) = 1/a, reecting the fact that, for a < b, X
a
is more curved
than X
b
.
The scalar curvature can also be negative. For instance for sur-
faces, X in R
3
, K(p) is positive at p if X is convex at p and negative
if X is convexconcave at p. (See Figure 4.9.3 below. The surface in
part (a) is convex at p, and the surface in part (b) is convexconcave.)

Figure 4.9.3.
216 Chapter 4. Forms on Manifolds
Let vol (S
n1
) be the Riemannian volume of the (n 1)-sphere,
i.e., let
vol (S
n1
) =
2
n/2
(n/2)
(where is the gamma function). Then by (4.8.14) the quotient
(4.8.15)
_
K
X
vol (S
n1
)
is the degree of the Gauss map, and hence is a topological invariant
of the surface of X. For n = 3 the GaussBonnet theorem asserts
that this topological invariant is just 1 g where g is the genus
of X or, in other words, the number of holes. Figure 4.9.4 gives a
pictorial proof of this result. (Notice that at the points, p
1
, . . . , p
g
the
surface,X is convexconcave so the scalar curvature at these points
is negative, i.e., the Gauss map is orientation reversing. On the other
hand, at the point, p
0
, the surface is convex, so the Gauss map at
this point us orientation preserving.)

suifaco ofgonus
Figure 4.9.4.
This is page 217
Printer: Opaque this
CHAPTER 5
COHOMOLOGY VIA FORMS
5.1 The DeRham cohomology groups of a manifold
In the last four chapters weve frequently encountered the question:
When is a closed k-form on an open subset of R
N
(or, more generally
on a submanifold of R
N
) exact? To investigate this question more
systematically than weve done heretofore, let X be an n-dimensional
manifold and let
Z
k
(X) = {
k
(X) ; d = 0} (5.1.1)
and
B
k
(X) = {
k
(X) ; in d
k1
(X)} (5.1.2)
be the vector spaces of closed and exact k-forms. Since (5.1.2) is a
vector subspace of (5.1.1) we can form the quotient space
(5.1.3) H
k
(X) = Z
k
(X)/B
k
(X) ,
and the dimension of this space is a measure of the extent to which
closed forms fail to be exact. We will call this space the k
th
DeRham
cohomology group of the manifold, X. Since the vector spaces (5.1.1)
and (5.1.2) are both innite dimensional there is no guarantee that
this quotient space is nite dimensional, however, well show later in
this chapter that it is in lots of interesting cases.
The spaces (5.1.3) also have compactly supported counterparts.
Namely let
Z
k
c
(X) = {
k
c
(X) ; d = 0} (5.1.4)
and
B
k
c
(X) = {
k
c
(X) , in d
k1
c
(X)} . (5.1.5)
Then as above B
k
c
(X) is a vector subspace of Z
k
c
(X) and the vector
space quotient
(5.1.6) H
k
c
(X) = Z
k
c
(X)/B
k
c
(X)
218 Chapter 5. Cohomology via forms
is the k
th
compactly supported DeRham cohomology group of X.
Given a closed k-form, Z
k
(X), we will denote by [] the image
of in the quotient space (5.1.3) and call [] the cohomology class
of . We will also use the same notation for compactly supported
cohomology. If is in Z
k
c
(X) well denote by [] the cohomology
class of in the quotient space (5.1.6).
Some cohomology groups of manifolds weve already computed in
the previous chapters (although we didnt explicitly describe these
computations as computing cohomology). Well make a list below
of some of the things weve already learned about DeRham cohomol-
ogy:
1. If X is connected, H
0
(X) = R. Proof: A closed zero form is
a function, f C

(X) having the property, df = 0, and if X is


connected the only such functions are constants.
2. If X is connected and non-compact H
0
c
(X) = {0}. Proof: If f
is in C

0
(X) and X is non-compact, f has to be zero at some point,
and hence if df = 0 it has to be identically zero.
3. If X is n-dimensional,

k
(X) =
k
c
(X) = {0}
for k less than zero or k greater than n, hence
H
k
(X) = H
k
c
(X) = {0}
for k less than zero or k greater than n.
4. If X is an oriented, connected n-dimensional manifold, the in-
tegration operation is a linear map
(5.1.7)
_
:
n
c
(X) R
and, by Theorem 4.8.1, the kernel of this map is B
n
c
(X). Moreover, in
degree n, Z
n
c
(X) =
n
c
(X) and hence by (5.1.6), we get from (5.1.7)
a bijective map
(5.1.8) I
X
: H
n
c
(X) R.
In other words
(5.1.9) H
n
c
(X) = R.
5.1 The DeRham cohomology groups of a manifold 219
5. Let U be a star-shaped open subset of R
n
. In 2.5, exercises 4
7, we sketched a proof of the assertion: For k > 0 every closed form,
Z
k
(U) is exact, i.e., translating this assertion into cohomology
language, we showed that
(5.1.10) H
k
(U) = {0} for k > 0 .
6. Let U R
n
be an open rectangle. In 3.2, exercises 47, we
sketched a proof of the assertion: If
k
c
(U) is closed and k is less
than n, then = d for some (k 1)-form,
k1
c
(U). Hence we
showed
(5.1.11) H
k
c
(U) = 0 for k < n.
7. Poincares lemma for manifolds: Let X be an n-dimensional
manifold and Z
k
(X), k > 0 a closed k-form. Then for every
point, p X, there exists a neighborhood, U of p and a (k 1)-form

k1
(U) such that = d on U. Proof: For open subsets of R
n
we proved this result in 2.3 and since X is locally dieomorphic at
p to an open subset of R
n
this result is true for manifolds as well.
8. Let X be the unit sphere, S
n
, in R
n+1
. Since S
n
is compact,
connected and oriented
(5.1.12) H
0
(S
n
) = H
n
(S
n
) = R.
We will show that for k =, 0, n
(5.1.13) H
k
(S
n
) = {0} .
To see this let
k
(S
n
) be a closed k-form and let p = (0, . . . , 0, 1)
S
n
be the north pole of S
n
. By the Poincare lemma there exists
a neighborhood, U, of p in S
n
and a k 1-form,
k1
(U) with
= d on U. Let C

0
(U) be a bump function which is equal
to one on a neighborhood, U
0
of U in p. Then
(5.1.14)
1
= d
is a closed k-form with compact support in S
n
{p}. However stere-
ographic projection gives one a dieomorphism
: R
n
S
n
{p}
220 Chapter 5. Cohomology via forms
(see exercise 1 below), and hence

1
is a closed compactly sup-
ported k-form on R
n
with support in a large rectangle. Thus by
(5.1.14)

= d, for some
k1
c
(R
n
), and by (5.1.14)
(5.1.15) = d( + (
1
)

)
with (
1
)


k1
c
(S
n
{p})
k
(S
n
), so weve proved that for
0 < k < n every closed k-form on S
n
is exact.
We will next discuss some pull-back operations in DeRham the-
ory. Let X and Y be manifolds and f : X Y a C

map. For

k
(Y ), df

= f

d, so if is closed, f

is as well. Moreover,
if = d, f

= df

, so if is exact, f

is as well. Thus we have


linear maps
f

: Z
k
(Y ) Z
k
(X) (5.1.16)
and
f

: B
k
(Y ) B
k
(X) (5.1.17)
and comparing (5.1.16) with the projection
: Z
k
(X) Z
k
(X)/B
k
(X)
we get a linear map
(5.1.18) Z
k
(Y ) H
k
(X) .
In view of (5.1.17), B
k
(Y ) is in the kernel of this map, so by Theo-
rem 1.2.2 one gets an induced linear map
(5.1.19) f

: H
k
(Y ) H
k
(Y ) ,
such that f

is the map (5.1.18). In other words, if is a closed


k-form on Y f

has the dening property


(5.1.20) f

[] = [f

] .
This pull-backoperation on cohomology satises the following
chain rule: Let Z be a manifold and g : Y Z a C

map. Then if
is a closed k-form on Z
(g f)

= f

5.1 The DeRham cohomology groups of a manifold 221


by the chain rule for pull-backs of forms, and hence by (5.1.20)
(5.1.21) (g f)

[] = f

(g

[]) .
The discussion above carries over verbatim to the setting of com-
pactly supported DeRham cohomology: If f : X Y is a proper
C

map it induces a pull-back map on cohomology


(5.1.22) f

: H
k
c
(Y ) H
k
c
(X)
and if f : X Y and g : Y Z are proper C

maps then the chain


rule (5.1.21) holds for compactly supported DeRham cohomology as
well as for ordinary DeRham cohomology. Notice also that if f :
X Y is a dieomorphism, we can take Z to be X itself and g to
be f
1
, and in this case the chain rule tells us that the maps (5.1.19)
and (5.1.22) are bijections, i.e., H
k
(X) and H
k
(Y ) and H
k
c
(X) and
H
k
c
(Y ) are isomorphic as vector spaces.
We will next establish an important fact about the pull-back op-
eration, f

; well show that its a homotopy invariant of f. Recall


that two C

maps
(5.1.23) f
i
: X Y , i = 0, 1
are homotopic if there exists a C

map
F : X [0, 1] Y
with the property F(p, 0) = f
0
(p) and F(p, 1) = f
1
(p) for all p X.
We will prove:
Theorem 5.1.1. If the maps (5.1.23) are homotopic then, for the
maps they induce on cohomology
(5.1.24) f

0
= f

1
.
Our proof of this will consist of proving this for an important
special class of homotopies, and then by pull-back tricks deducing
this result for homotopies in general. Let v be a complete vector eld
on X and let
f
t
: X X , < t <
be the one-parameter group of dieomorphisms it generates. Then
F : X [0, 1] X , F(p, t) = f
t
(p) ,
222 Chapter 5. Cohomology via forms
is a homotopy between f
0
and f
1
, and well show that for this ho-
motopic pair (5.1.24) is true. Recall that for
k
(X)
_
d
dt
f

t

_
(t = 0) = L
v
= (v) d + d(v)
and more generally for all t
d
dt
f

t
=
_
d
ds
f

s+t

_
(s = 0)
=
_
d
ds
(f
s
f
t
)

_
(s = 0)
=
_
d
ds
f

t
f

_
(s = 0) = f

t
_
d
ds
f

_
(s = 0)
= f

t
L
v

= f

t
(v) d + df

t
(v) .
Thus if we set
(5.1.25) Q
t
= f

t
(v)
we get from this computation:
(5.1.26)
d
dt
f

= dQ
t
+Q
t
d
and integrating over 0 t 1:
(5.1.27) f

1
f

0
= dQ +Qd
where
Q :
k
(Y )
k1
(X)
is the operator
(5.1.28) Q =
_
1
0
Q
t
dt .
The identity (5.1.24) is an easy consequence of this chain homo-
topy identity. If is in Z
k
(X), d = 0, so
f

1
f

0
= dQ
and
5.1 The DeRham cohomology groups of a manifold 223
f

1
[] f

0
[] = [f

1
f

0
] = 0 .
Q.E.D.
Well now describe how to extract from this result a proof of The-
orem 5.1.1 for any pair of homotopic maps. Well begin with the
following useful observation.
Proposition 5.1.2. If f
i
: X Y , i = 0, 1, are homotopic C

mappings there exists a C

map
F : X R Y
such that the restriction of F to X [0, 1] is a homotopy between f
0
and f
1
.
Proof. Let C

0
(R), 0, be a bump function which is supported
on the interval,
1
4
t
3
4
and is positive at t =
1
2
. Then
(t) =
_
t

(s) ds
_
_

(s) ds
is a function which is zero on the interval t
1
4
, is one on the interval
t
3
4
, and, for all t, lies between 0 and 1. Now let
G : X [0, 1] Y
be a homotopy between f
0
and f
1
and let F : X R Y be the
map
(5.1.29) F(x, t) = G(x, (t)) .
This is a C

map and since


F(x, 1) = G(x, (1)) = G(x, 1) = f
1
(x)
and
F(x, 0) = G(x, (0)) = G(x, 0) = f
0
(x) ,
it gives one a homotopy between f
0
and f
1
.
224 Chapter 5. Cohomology via forms
Were now in position to deduce Theorem 5.1.1 from the version
of this result that we proved above.
Let

t
: X R X R, < t <
be the one-parameter group of dieomorphisms

t
(x, a) = (x, a +t)
and let v = /t be the vector eld generating this group. For k-
forms,
k
(X R), we have by (5.1.27) the identity

0
= d + d (5.1.30)
where
=
_
1
0

t
_

t
_

_
dt . (5.1.31)
Now let F, as in Proposition 5.1.2, be a C

map
F : X R Y
whose restriction to X[0, 1] is a homotopy between f
0
and f
1
. Then
for
k
(Y )
(5.1.32)

1
F

0
F

= dF

+ F

d
by the identity (5.1.29). Now let : X X R be the inclusion,
p (p, 0), and note that
(F
1
)(p) = F(p, 1) = f
1
(p)
and
(F
0
)(p) = F(p, 0) = f
0
(p)
i.e.,
F
1
= f
1
(5.1.33)
and
F
0
= f
0
. (5.1.34)
5.1 The DeRham cohomology groups of a manifold 225
Thus

1
F

0
F

) = f

1
f

and on the other hand by (5.1.31)

1
F

0
F

) = d

d .
Letting
(5.1.35) Q :
k
(Y )
k1
(X)
be the chain homotopy operator
(5.1.36) Q =

we can write the identity above more succinctly in the form


(5.1.37) f

1
f

c
= dQ +Qd
and from this deduce, exactly as we did earlier, the identity (5.1.24).
This proof can easily be adapted to the compactly supported set-
ting. Namely the operator (5.1.36) is dened by the integral
(5.1.38) Q =
_
1
0

t
_

t
_
F

_
dt .
Hence if is supported on a set, A, in Y , the integrand of (5.1.37)
at t is supported on the set
(5.1.39) {p X , F(p, t) A}
and hence Q is supported on the set
(5.1.40) (F
1
(A) X [0, 1])
where : X[0, 1] X is the projection map, (p, t) = p. Suppose
now that f
0
and f
1
are proper mappings and
G : X [0, 1] Y
a proper homotopy between f
0
and f
1
, i.e., a homotopy between f
0
and f
1
which is proper as a C

map. Then if F is the map (5.1.30) its


restriction to X[0, 1] is also a proper map, so this restriction is also
a proper homotopy between f
0
and f
1
. Hence if is in
k
c
(Y ) and
A is its support, the set (5.1.39) is compact, so Q is in
k1
c
(X).
Therefore all summands in the chain homotopy formula (5.1.37)
are compactly supported. Thus weve proved
226 Chapter 5. Cohomology via forms
Theorem 5.1.3. If f
i
: X Y , i = 0, 1 are proper C

maps
which are homotopic via a proper homotopy, the induced maps on
cohomology
f

i
: H
k
c
(Y ) H
k
c
(X)
are the same.
Well conclude this section by noting that the cohomology groups,
H
k
(X), are equipped with a natural product operation. Namely,
suppose
i

k
i
(X), i = 1, 2, is a closed form and that c
i
= [
i
]
is the cohomology class represented by
i
. We can then dene a
product cohomology class c
1
c
2
in H
k
1
+k
2
(X) by the recipe
(5.1.41) c
1
c
2
= [
1

2
] .
To show that this is a legitimate denition we rst note that since

2
is closed
d(
1

2
) = d
1

2
+ (1)
k
1

1
d
2
= 0 ,
so
1

2
is closed and hence does represent a cohomology class.
Moreover if we replace
1
by another representative,
1
+ d
1
=

,
of the cohomology class, c
1

1

2
=
1

2
+ d
1

2
.
But since
2
is closed,
d
1

2
= d(
1

2
) + (1)
k
1

1
d
2
= d(
1

2
)
so

1

2
=
1

2
+d(
1

2
)
and [

2
] = [
1

2
]. Similary (5.1.41) is unchanged if we replace

2
by
2
+d
2
, so the denition of (5.1.41) depends neither on the
choice of
1
nor
2
and hence is an intrinsic denition as claimed.
There is a variant of this product operation for compactly sup-
ported cohomology classes, and well leave for you to check that
its also well dened. Suppose c
1
is in H
k
1
c
(X) and c
2
is in H
k
2
(X)
(i.e., c
1
is a compactly supported class and c
2
is an ordinary coho-
mology class). Let
1
be a representative of c
1
in
k
1
c
(X) and
2
5.1 The DeRham cohomology groups of a manifold 227
a representative of c
2
in
k
2
(X). Then
1

2
is a closed form in

k
1
+k
2
c
(X) and hence denes a cohomology class
(5.1.42) c
1
c
2
= [
1

2
]
in H
k
1
+k
2
c
(X). Well leave for you to check that this is intrinsically
dened. Well also leave for you to check that (5.1.42) is intrinsically
dened if the roles of c
1
and c
2
are reversed, i.e., if c
1
is in H
k
1
(X)
and c
2
in H
k
2
c
(X) and that the products (5.1.41) and (5.1.42) both
satisfy
(5.1.43) c
1
c
2
= (1)
k
1
k
2
c
2
c
1
.
Finally we note that if Y is another manifold and f : X Y a C

map then for


1

k
1
(Y ) and
2

k
2
(Y )
f

(
1

2
) = f

1
f

2
by (2.5.7) and hence if
1
and
2
are closed and c
i
= [
i
]
(5.1.44) f

(c
1
c
2
) = f

c
1
f

c
2
.
Exercises.
1. (Stereographic projection.) Let p S
n
be the point, (0, 0, . . . , 0, 1).
Show that for every point x = (x
1
, . . . , x
n+1
) of S
n
{p} the ray
tx + (1 t)p , t > 0
intersects the plane, x
n+1
= 0, in the point
(x) =
1
1 x
n+1
(x
1
, . . . , x
n
)
and that the map
: S
n
{p} R
n
, x (x)
is a dieomorphism.
2. Show that the operator
Q
t
:
k
(Y )
k1
(X)
228 Chapter 5. Cohomology via forms
in the integrand of (5.1.38), i.e., the operator,
Q
t
=

t
_

t
_
F

_
has the following description. Let p be a point of X and let q = f
t
(p).
The curve, s f
s
(p) passes through q at time s = t. Let v(q) T
q
Y
be the tangent vector to this curve at t. Show that
(5.1.45) (Q
t
)(p) = (df

t
)
p
(v
q
)
q
.
3. Let U be a star-shaped open subset of R
n
, i.e., a subset of R
n
with the property that for every p U the ray, tp, 0 t < 1, is in U.
(a) Let v be the vector eld
v =

x
i

x
i
and
t
: U U, the map p tp. Show that for every k-form,

k
(U)
= dQ +Qd
where
Q =
_
1
0

t
(v)
dt
t
.
(b) Show that if
=

a
I
(x) dx
I
then
Q =

I,r
__
t
k1
(1)
r1
x
ir
a
I
(tx) dt
_
dx
Ir
(5.1.46)
where
dx
Ir
= dx
i
1


dx
ir
dx
i
k
.
5.1 The DeRham cohomology groups of a manifold 229
4. Let X and Y be oriented connected n-dimensional manifolds,
and f : X Y a proper map. Show that the linear map, L, in the
diagram below
H
n
c
(Y )
f

H
n
c
(X)
I
Y

_
I
X

_
R
L
R
is just the map, t R deg(f)t.
5. Let X and Y be manifolds and let id
X
and id
Y
be the identity
maps of X onto X and Y onto Y . A homotopy equivalence between
X and Y is a pair of maps
f : X Y
and
g : Y X
such that g f is homotopic to i d
X
and f g is homotopic to id
Y
.
Show that if X and Y are homotopy equivalent their cohomology
groups are the same up to isomorphism, i.e., there exist bijections
H
k
(X) H
k
(Y ) .
6. Show that R
n
{0} and S
n1
are homotopy equivalent.
7. What are the cohomology groups of the n-sphere with two
points deleted? Hint: The n-sphere with one point deleted is R
n
.
8. Let X and Y be manifolds and f
i
: X Y , i = 0, 1, 2, C

maps. Show that if f


0
and f
1
are homotopic and f
1
and f
2
are ho-
motopic then f
0
and f
2
are homotopic.
Hint: The homotopy (5.1.20) has the property that
F(p, t) = f
t
(p) = f
0
(p)
for 0 t
1
4
and
F(p, t) = f
t
(p) = f
1
(p)
for
3
4
t < 1. Show that two homotopies with these properties: a
homotopy between f
0
and f
1
and a homotopy between f
1
and f
2
,
are easy to glue together to get a homotopy between f
0
and f
2
.
230 Chapter 5. Cohomology via forms
9. (a) Let X be an n-dimensional manifold. Given points p
i
X,
i = 0, 1, 2 show that if p
0
can be joined to p
1
by a C

curve,
0
:
[0, 1] X, and p
1
can be joined to p
2
by a C

curve,
1
: [0, 1] X,
then p
0
can be joined to p
2
by a C

curve, : [0, 1] X.
Hint: A C

curve, : [0, 1] X, joining p


0
to p
2
can be thought
of as a homotopy between the maps

p
0
: pt X , pt p
0
and

p
1
: pt X , pt p
1
where pt is the zero-dimensional manifold consisting of a single
point.
(b) Show that if a manifold, X, is connected it is arc-wise con-
nected: any two points can by joined by a C

curve.
10. Let X be a connected n-dimensional manifold and
1
(X)
a closed one-form.
(a) Show that if : [0, 1] X is a C

curve there exists a partition:


0 = a
0
< a
1
< < a
n
= 1 of the interval [0, 1] and open sets U
i
in
X such that ([a
i1
, a
i
]) U
i
and such that |U
i
is exact.
(b) In part (a) show that there exist functions, f
i
C

(U
i
) such
that |U
i
= df
i
and f
i
((a
i
)) = f
i+1
((a
i
)).
(c) Show that if p
0
and p
1
are the end points of
f
n
(p
1
) f
1
(p
0
) =
_
1
0

.
(d) Let
(5.1.47)
s
: [0, 1] X , 0 s 1
be a homotopic family of curves with
s
(0) = p
0
and
s
(1) = p
1
.
Prove that the integral
_
1
0

is independent of s
0
. Hint: Let s
0
be a point on the interval, [0, 1].
For =
s
0
choose a
i
s and f
i
s as in parts (a)(b) and show that
for s close to s
0
,
s
[a
i1
, a
i
] U
i
.
5.2 The MayerVictoris theorem 231
(e) A manifold, X, is simply connected if, for any two curves,
i
:
[0, 1] X, i = 0, 1, with the same end-points, p
0
and p
1
, there exists
a homotopy (5.1.42) with
s
(0) = p
0
and
s
(1) = p
1
, i.e.,
0
can be
smoothly deformed into
1
by a family of curves all having the same
end-points. Prove
Theorem 5.1.4. If X is simply-connected H
1
(X) = {0}.
11. Show that the product operation (5.1.41) is associative and
satises left and right distributive laws.
12. Let X be a compact oriented 2n-dimensional manifold. Show
that the map
B : H
n
(X) H
n
(X) R
dened by
B(c
1
, c
2
) = I
X
(c
1
c
2
)
is a bilinear form on H
n
(X) and that its symmetric if n is even and
alternating if n is odd.
5.2 The MayerVictoris theorem
In this section well develop some techniques for computing coho-
mology groups of manifolds. (These techniques are known collec-
tively as diagram chasing and the mastering of these techniques is
more akin to becoming procient in checkers or chess or the Sunday
acrostics in the New York Times than in the areas of mathematics
to which theyre applied.) Let C
i
, i = 0, 1, 2, . . ., be vector spaces
and d : C
i
C
i+1
a linear map. The sequence of vector spaces and
maps
(5.2.1) C
0
d
C
1
d
C
2
d

is called a complex if d
2
= 0, i.e., if for a C
k
, d(da) = 0. For
instance if X is a manifold the DeRham complex
(5.2.2)
0
(X)
d

1
(X)
d

2
(X)
is an example of a complex, and the complex of compactly supported
DeRham forms
(5.2.3)
0
c
(X)
d

1
c
(X)
d

2
c
(X)
232 Chapter 5. Cohomology via forms
is another example. One denes the cohomology groups of the com-
plex (5.2.1) in exactly the same way that we dened the cohomology
groups of the complexes (5.2.2) and (5.2.3) in 5.1. Let
Z
k
= {a C
k
; da = 0}
and
B
k
= {a C
k
; a dC
k1
}
i.e., let a be in B
k
if and only if a = db for some b C
k1
. Then
da = d
2
b = 0, so B
k
is a vector subspace of Z
k
, and we dene
H
k
(C) the k
th
cohomology group of the complex (5.2.1) to be
the quotient space
(5.2.4) H
k
(C) = Z
k
/B
k
.
Given c Z
k
we will, as in 5.1, denote its image in H
k
(C) by [c]
and well call c a representative of the cohomology class [c].
We will next assemble a small dictionary of diagram chasing
terms.
Denition 5.2.1. Let V
i
, i = 0, 1, 2, . . ., be vector spaces and
i
:
V
i
V
i+1
linear maps. The sequence
(5.2.5) V
0

0
V
1

1
V
2

2

is an exact sequence if, for each i, the kernel of
i+1
is equal to the
image of
i
.
For example the sequence (5.2.1) is exact if Z
i
= B
i
for all i, or,
in other words, if H
i
(C) = 0 for all i. A simple example of an exact
sequence that well encounter a lot below is a sequence of the form
(5.2.6) {0} V
1

1
V
2

2
V
3
{0} ,
a ve term exact sequence whose rst and last terms are the vector
space, V
0
= V
4
= {0}, and hence
0
=
3
= 0. This sequence is
exact if and only if
1.
1
is injective,
2. the kernel of
2
equals the image of
1
, and
5.2 The MayerVictoris theorem 233
3.
2
is surjective.
We will call an exact sequence of this form a short exact sequence.
(Well also encounter a lot below an even shorter example of an exact
sequence, namely a sequence of the form
(5.2.7) {0} V
1

1
V
2
{0} .
This is an exact sequence if and only if
1
is bijective.)
Another basic notion in the theory of diagram chasing is the notion
of a commutative diagram. The square diagram of vector spaces and
linear maps
A
f
B
i

j
C
g
D
is commutative if f i = j g, and a more complicated diagram
of vector spaces and linear maps like the diagram below
A
1
A
2
A
3

B
1
B
2
B
3

C
1
C
2
C
3
is commutative if every subsquare in the diagram, for instance the
square,
B
2
B
3

C
2
C
3
is commutative.
We now have enough diagram chasing vocabulary to formulate
the MayerVictoris theorem. For r = 1, 2, 3 let
(5.2.8) {0} C
0
r
d
C
1
r
d
C
2
r
d

be a complex and, for xed k, let
(5.2.9) {0} C
k
1
i
C
k
2
j
C
k
3
{0}
234 Chapter 5. Cohomology via forms
be a short exact sequence. Assume that the diagram below com-
mutes:
(5.2.10)
| | |
0 C
k+1
1
i
C
k+1
2
j
C
k+1
3
0

d
0 C
k
1
C
k
2
C
k
3
0

0 C
k1
1
i
C
k1
2
j
C
k1
3
0

i.e., assume that in the left hand squares, di = id, and in the right
hand squares, dj = jd.
The MayerVictoris theorem addresses the following question: If
one has information about the cohomology groups of two of the three
complexes, (5.2.8), what information about the cohomology groups
of the third can be extracted from this diagram? Lets rst observe
that the maps, i and j, give rise to mappings between these cohomol-
ogy groups. Namely, for r = 1, 2, 3 let Z
k
r
be the kernel of the map,
d : C
k
r
C
k+1
r
, and B
k
r
the image of the map, d : C
k1
r
C
k
r
. Since
id = di, i maps B
k
1
into B
k
2
and Z
k
1
into Z
k
2
, therefore by (5.2.4) it
gives rise to a linear mapping
i

: H
k
(C
1
) H
k
(C
2
) .
Similarly since jd = dj, j maps B
k
2
into B
k
3
and Z
k
2
into Z
k
3
, and so
by (5.2.4) gives rise to a linear mapping
j

: H
k
(C
2
) H
k
(C
3
) .
Moreover, since j i = 0 the image of i

is contained in the kernel of


j

. Well leave as an exercise the following sharpened version of this


observation:
Proposition 5.2.2. The kernel of j

equals the image of i

, i.e., the
three term sequence
(5.2.11) H
k
(C
1
)
i

H
k
(C
2
)
j

H
k
(C
3
)
is exact.
5.2 The MayerVictoris theorem 235
Since (5.2.9) is a short exact sequence one is tempted to conjecture
that (5.2.11) is also a short exact sequence (which, if it were true,
would tell us that the cohomology groups of any two of the complexes
(5.2.8) completely determine the cohomology groups of the third).
Unfortunately, this is not the case. To see how this conjecture can be
violated lets try to show that the mapping j

is surjective. Let c
k
3
be
an element of Z
k
3
representing the cohomology class, [c
k
3
], in H
3
(C
3
).
Since (5.2.9) is exact there exists a c
k
2
in C
k
2
which gets mapped by j
onto c
k
3
, and if c
k
3
were in Z
k
2
this would imply
j

[c
k
2
] = [jc
k
2
] = [c
k
3
] ,
i.e., the cohomology class, [c
k
3
], would be in the image of j

. However,
since theres no reason for c
k
2
to be in Z
k
2
, theres also no reason for
[c
k
3
] to be in the image of j

. What we can say, however, is that


j dc
k
2
= djc
k
2
= dc
k
3
= 0 since c
k
3
is in Z
k
3
. Therefore by the exactness
of (5.2.9) in degree k +1 there exists a unique element, c
k+1
1
in C
k+1
1
with property
(5.2.12) dc
k
2
= ic
k+1
1
.
Moreover, since 0 = d(dc
k
2
) = di(c
k+1
1
) = i dc
k+1
1
and i is injective,
dc
k+1
1
= 0, i.e.,
(5.2.13) c
k+1
1
Z
k+1
1
.
Thus via (5.2.12) and (5.2.13) weve converted an element, c
k
3
, of Z
k
3
into an element, c
k+1
1
, of Z
k+1
1
and hence set up a correspondence
(5.2.14) c
k
3
Z
k
3
c
k+1
1
Z
k+1
1
.
Unfortunately this correspondence isnt, strictly speaking, a map
of Z
k
3
into Z
k+1
1
; the c
k
1
in (5.2.14) isnt determined by c
k
3
alone
but also by the choice we made of c
k
2
. Suppose, however, that we
make another choice of a c
k
2
with the property j(c
k
2
) = c
k
3
. Then the
dierence between our two choices is in the kernel of j and hence,
by the exactness of (2.5.8) at level k, is in the image of i. In other
words, our two choices are related by
(c
k
2
)
new
= (c
k
2
)
old
+i(c
k
1
)
for some c
k
1
in C
k
1
, and hence by (5.2.12)
(c
k+1
1
)
new
= (c
k+1
1
)
old
+dc
k
1
.
236 Chapter 5. Cohomology via forms
Therefore, even though the correspondence (5.2.14) isnt strictly
speaking a map it does give rise to a well-dened map
(5.2.15) Z
k
3
H
k+1
(C
1
) , c
k
3
[c
k+1
3
] .
Moreover, if c
k
3
is in B
k
3
, i.e., c
k
3
= dc
k1
3
for some c
k1
3
C
k1
3
, then
by the exactness of (5.2.8) at level k 1, c
k1
3
= j(c
k1
2
) for some
c
k1
2
C
k1
2
and hence c
k
3
= j(dc
k2
2
). In other words we can take
the c
k
2
above to be dc
k1
2
in which case the c
k+1
1
in equation (5.2.12)
is just zero. Thus the map (5.2.14) maps B
k
3
to zero and hence by
Proposition 1.2.2 gives rise to a well-dened map
(5.2.16) : H
k
(C
3
) H
k+1
(C
1
)
mapping [c
k
3
] [c
k+1
1
]. We will leave it as an exercise to show that
this mapping measures the failure of the arrow j

in the exact se-


quence (5.2.11) to be surjective (and hence the failure of this se-
quence to be a short exact sequence at its right end).
Proposition 5.2.3. The image of the map j

: H
k
(C
2
) H
k
(C
3
)
is equal to the kernel of the map, : H
k
(C
3
) H
k+1
(C
1
).
Hint: Suppose that in the correspondence (5.2.14) c
k+1
1
is in B
k+1
1
.
Then c
k+1
1
= dc
k
1
for some c
k
1
in C
k
1
. Show that
j(c
k
2
i(c
k
1
)) = c
k
3
and
d(c
k
2
i(c
k
1
)) = 0
i.e., c
k
2
i(c
k
1
) is in Z
k
2
and hence j

[c
k
2
i(c
k
1
)] = [c
k
3
].
Lets next explore the failure of the map, i

: H
k+1
(C
1
) H
k+1
(C
2
),
to be injective. Let c
k+1
1
be in Z
k+1
1
and suppose that its cohomol-
ogy class, [c
k+1
1
], gets mapped by i

into zero. This translates into


the statement
(5.2.17) i(c
k+1
1
) = dc
k
2
for some c
k
2
C
k
2
. Moreover since dc
k
2
= i(c
k+1
1
), j(dc
k
2
) = 0. But if
(5.2.18) c
k
3
def
= j(c
k
2
)
5.2 The MayerVictoris theorem 237
then dc
k
3
= dj(c
k
2
) = j(dc
k
2
) = j(i(c
k+1
1
)) = 0, so c
k
3
is in Z
k
3
, and by
(5.2.17), (5.2.18) and the denition of
(5.2.19) [c
k+1
1
] = [c
k
3
] .
In other words the kernel of the map, i

: H
k+1
(C
1
) H
k+1
(C
2
) is
contained in the image of the map : H
k
(C
3
) H
k+1
(C
1
). We will
leave it as an exercise to show that this argument can be reversed to
prove the converse assertion and hence to prove
Proposition 5.2.4. The image of the map : H
k
(C
1
) H
k+1
(C
1
)
is equal to the kernel of the map i

: H
k+1
(C
1
) H
k+1
(C
2
).
Putting together the Propositions 5.2.25.2.4 we obtain the main
result of this section: the MayerVictoris theorem. The sequence of
cohomology groups and linear maps
(5.2.20)

H
k
(C
1
)
i

H
k
(C
2
)
j

H
k
(C
3
)

H
k+1
(C1)
i


is exact.
Remark 5.2.5. In view of the s this sequence can be a very
long sequence and is commonly referred to as the long exact se-
quence in cohomology associated to the short exact sequence of com-
plexes (2.5.9).
Before we discuss the applications of this result, we will introduce
some vector space notation. Given vector spaces, V
1
and V
2
well
denote by V
1
V
2
the vector space sum of V
1
and V
2
, i.e., the set of
all pairs
(u
1
, u
2
) , u
i
V
i
with the addition operation
(u
1
, u
2
) + (v
1
+ v
2
) = (u
1
+ v
1
, u
2
+ v
2
)
and the scalar multiplication operation
(u
1
, u
2
) = (u
1
, u
2
) .
Now let X be a manifold and let U
1
and U
2
be open subsets of X.
Then one has a linear map
(5.2.21)
k
(U
1
U
2
)
i

k
(U
1
)
k
(U
2
)
238 Chapter 5. Cohomology via forms
dened by
(5.2.22) (|U
1
, |U
2
)
where |U
i
is the restriction of to U
i
. Similarly one has a linear
map
(5.2.23)
k
(U
1
)
k
(U
2
)
j

k
(U
1
U
2
)
dened by
(5.2.24) (
1
,
2
)
1
|U
1
U
2

2
|U
1
U
2
.
We claim
Theorem 5.2.6. The sequence
(5.2.25)
{0}
k
(U
1
U
2
)
i

k
(U
1
)
k
(U
2
)
j

k
(U
1
U
2
) {0}
is a short exact sequence.
Proof. If the right hand side of (5.2.22) is zero, itself has to be
zero so the map (5.2.22) is injective. Moreover, if the right hand side
of (5.2.24) is zero,
1
and
2
are equal on the overlap, U
1
U
2
, so
we can glue them together to get a C

k-form on U
1
U
2
by setting
=
1
on U
1
and =
2
on U
2
. Thus by (5.2.22) i() = (
1
,
2
),
and this shows that the kernel of j is equal to the image of i. Hence
to complete the proof we only have to show that j is surjective,
i.e., that every form on
k
(U
1
U
2
) can be written as a dierence,

1
|U
1
U
2

2
|U
1
U
2
, where
1
is in
k
(U
1
) and
2
in in
k
(U
2
).
To prove this well need the following variant of the partition of unity
theorem.
Theorem 5.2.7. There exist functions,

(U
1
U
2
), = 1, 2,
such that support

is contained in U

and
1
+
2
= 1.
Before proving this lets use it to complete our proof of Theo-
rem 5.2.6. Given
k
(U
1
U
2
) let

1
=
_
_
_

2
on U
1
U
2
0 on U
1
U
1
U
2
(5.2.26)
and let
5.2 The MayerVictoris theorem 239

2
=
_

1
on U
1
U
2
0 on U
2
U
1
U
2
.
(5.2.27)
Since
2
is supported on U
2
the form dened by (5.2.26) is C

on U
1
and since
1
is supported on U
1
the form dened by (5.2.27) is C

on U
2
and since
1
+
2
= 1,
1

2
= (
1
+
2
) = on U
1
U
2
.
To prove Theorem 5.2.7, let
i
C

0
(U
1
U
2
), i = 1, 2, 3, . . .
be a partition of unity subordinate to the cover, {U

, = 1, 2} of
U
1
U
2
and let
1
be the sum of the
i
s with support on U
1
and

2
the sum of the remaining
i
s. Its easy to check (using part (b)
of Theorem 4.6.1) that

is supported in U

and (using part (c) of


Theorem 4.6.1) that
1
+
2
= 1.
Now let
(5.2.28) {0} C
0
1
d
C
1
1
d
C
2
1

be the DeRham complex of U
1
U
2
, let
(5.2.29) {0} C
0
3
d
C
1
3
d
C
2
3

be the DeRham complex of U
1
U
2
and let
(5.2.30) {0} C
0
2
d
C
1
2
d
C
2
2
d

be the vector space direct sum of the DeRham complexes of U
1
and
U
2
, i.e., the complex whose k
th
term is
C
k
2
=
k
(U
1
)
k
(U
2
)
with d : C
k
2
C
k+1
2
dened to be the map d(
1
,
2
) = (d
1
, d
2
).
Since C
k
1
=
k
(U
1
U
2
) and C
k
3
=
k
(U
1
U
2
) we have, by Theo-
rem 5.2.6, a short exact sequence
(5.2.31) {0} C
k
1
i
C
k
2
j
C
k
3
{0} ,
and its easy to see that i and j commute with the ds:
(5.2.32) di = id and dj = jd .
240 Chapter 5. Cohomology via forms
Hence were exactly in the situation to which MayerVictoris applies.
Since the cohomology groups of the complexes (5.2.28) and (5.2.29)
are the DeRham cohomology group. H
k
(U
1
U
2
) and H
k
(U
1
U
2
),
and the cohomology groups of the complex (5.2.30) are the vector
space direct sums, H
k
(U
1
) H
k
(U
2
), we obtain from the abstract
MayerVictoris theorem, the following DeRham theoretic version of
MayerVictoris.
Theorem 5.2.8. Letting U = U
1
U
2
and V = U
1
U
2
one has a
long exact sequence in DeRham cohomology:
(5.2.33)


H
k
(U)
i

H
k
(U
1
) H
k
(U
2
)
j

H
k
(V )

H
k+1
(U)
i

.
This result also has an analogue for compactly supported DeRham
cohomology. Let
(5.2.34) i :
k
c
(U
1
U
2
) H
k
c
(U
1
)
k
c
(U
2
)
be the map
(5.2.35) i() = (
1
,
2
)
where
(5.2.36)
i
=
_
on U
1
U
2
0 on U
i
U
1
U
2
.
(Since is compactly supported on U
1
U
2
the form dened by
(5.2.34) is a C

form and is compactly supported on U


i
.) Similarly,
let
(5.2.37) j :
k
c
(U
1
)
k
c
(U
2
)
k
c
(U
1
U
2
)
be the map
(5.2.38) j(
1
,
2
) =
1

2
where:
(5.2.39)
i
=
_

i
on U
i
0 on (U
1
U
2
) U
i
.
As above its easy to see that i is injective and that the kernel of j
is equal to the image of i. Thus if we can prove that j is surjective
well have proved
5.2 The MayerVictoris theorem 241
Theorem 5.2.9. The sequence
(5.2.40)
{0}
k
c
(U
1
U
2
)
i

k
c
(U
1
)
k
c
(U
2
)
j

k
c
(U
1
U
2
) {0}
is a short exact sequence.
Proof. To prove the surjectivity of j we mimic the proof above. Given
in
k
c
(U
1
U
2
) let
=
1
|U
1
(5.2.41)
and

2
=
2
|U
2
. (5.2.42)
Then by (5.2.36) = j(
1
,
2
).
Thus, applying MayerVictoris to the compactly supported ver-
sions of the complexes (5.2.8), we obtain:
Theorem 5.2.10. Letting U = U
1
U
2
and V = U
1
U
2
there exists
a long exact sequence in compactly supported DeRham cohomology
(5.2.43)


H
k
c
(V )
i

H
k
c
(U
1
) H
k
c
(U
2
)
j

H
k
c
(U)

H
k+1
c
(V )
i

.
Exercises
1. Prove Proposition 5.2.2.
2. Prove Proposition 5.2.3.
3. Prove Proposition 5.2.4.
4. Show that if U
1
, U
2
and U
1
U
2
are non-empty and connected
the rst segment of the MayerVictoris sequence is a short exact
sequence
{0} H
0
(U
1
U
2
) H
0
(U
1
) H
0
(U
2
) H
0
(U
1
U
2
) {0} .
5. Let X = S
n
and let U
1
and U
2
be the open subsets of S
n
obtained by removing from S
n
the points, p
1
= (0, . . . , 0, 1) and
p
2
= (0, . . . , 0, 1).
242 Chapter 5. Cohomology via forms
(a) Using stereographic projection show that U
1
and U
2
are dieo-
morphic to R
n
.
(b) Show that U
1
U
2
= S
n
and U
1
U
2
is homotopy equivalent
to S
n1
. (See problem 5 in 5.1.) Hint: U
1
U
2
is dieomorphic to
R
n
{0}.
(c) Deduce from the MayerVictoris sequence that H
i+1
(S
n
) =
H
i
(S
n1
) for i 1.
(d) Using part (c) give an inductive proof of a result that we proved
by other means in 5.1: H
k
(S
n
) = {0} for 1 k < n.
6. Using the MayerVictoris sequence of exercise 5 with cohomol-
ogy replaced by compactly supported cohomology show that
H
k
c
(R
n
{0})

= R
for k = 1 and n and
H
k
c
(R
n
{0}) = {0}
for all other values of k.
5.3 Good covers
In this section we will show that for compact manifolds (and for lots
of other manifolds besides) the DeRham cohomology groups which
we dened in 5.1 are nite dimensional vector spaces and thus, in
principle, computable objects. A key ingredient in our proof of this
fact is the notion of a good cover of a manifold.
Denition 5.3.1. Let X be an n-dimensional manifold, and let
U = {U

, I}
be a covering of X by open sets. This cover is a good cover if for
every nite set of indices,
i
I, i = 1, . . . , k, the intersection
U
1
U
k
is either empty or is dieomorphic to R
n
.
One of our rst goals in this section will be to show that good
covers exist. We will sketch below a proof of the following.
Theorem 5.3.2. Every manifold admits a good cover.
5.3 Good covers 243
The proof involves an elementary result about open convex subsets
of R
n
.
Proposition 5.3.3. If U is a bounded open convex subset of R
n
, it
is dieomorphic to R
n
.
A proof of this will be sketched in exercises 14 at the end of this
section.
One immediate consequence of this result is an important special
case of Theorem 5.3.2.
Theorem 5.3.4. Every open subset, U, of R
n
admits a good cover.
Proof. For each p U let U
p
be an open convex neighborhood of p
in U (for instance an -ball centered at p) . Since the intersection of
any two convex sets is again convex the cover, {U
p
, p U} is a good
cover by Proposition 5.3.3.
For manifolds the proof of Theorem 5.3.2 is somewhat trickier.
The proof requires a manifold analogue of the notion of convexity
and there are several serviceable candidates. The one we will use is
the following. Let X R
N
be an n-dimensional manifold and for
p X let T
p
X be the tangent space to X at p. Recalling that T
p
X
sits inside T
p
R
N
and that
T
p
R
N
= {(p, v) , v R
N
}
we get a map
T
p
X T
p
R
N
R
N
, (p, x) p +x,
and this map maps T
p
X bijectively onto an n-dimensional ane
subspace, L
p
, of R
N
which is tangent to X at p. Let
p
: X L
p
be, as in the gure below, the orthogonal projection of X onto L
p
.
244 Chapter 5. Cohomology via forms

p
(x)
L
p
p
x
Denition 5.3.5. An open subset, V , of X is convex if for every
p V the map
p
: X L
p
maps V dieomorphically onto a convex
open subset of L
p
.
Its clear from this denition of convexity that the intersection of
two open convex subsets of X is an open convex subset of X and
that every open convex subset of X is dieomorphic to R
n
. Hence
to prove Theorem 5.3.2 it suces to prove that every point, p, in X
is contained in an open convex subset, U
p
, of X. Here is a sketch of
how to prove this. In the gure above let B

(p) be the ball of radius


about p in L
p
centered at p. Since L
p
and T
p
are tangent at p the
derivative of
p
at p is just the identity map, so for small
p
maps a
neighborhood, U

p
of p in X dieomorphically onto B

(p). We claim
Proposition 5.3.6. For small, U

p
is a convex subset of X.
Intuitively this assertion is pretty obvious: if q is in U

p
and is
small the map
B

1
p
U

p
q
L
q
is to order
2
equal to the identity map, so its intuitively clear that
its image is a slightly warped, but still convex, copy of B

(p). We
wont, however, bother to write out the details that are required to
make this proof rigorous.
A good cover is a particularly good good cover if it is a nite
cover. Well codify this property in the denition below.
5.3 Good covers 245
Denition 5.3.7. An n-dimensional manifold is said to have nite
topology if it admits a nite covering by open sets, U
1
, . . . , U
N
with
the property that for every multi-index, I = (i
1
, . . . , i
k
), 1 i
1

i
2
< i
K
N, the set
(5.3.1) U
I
= Ui
1
Ui
k
is either empty or is dieomorphic to R
n
.
If X is a compact manifold and U = {U

, I} is a good cover
of X then by the HeineBorel theorem we can extract from U a nite
subcover
U
i
= U

i
,
i
I , i = 1, . . . , N ,
hence we conclude
Theorem 5.3.8. Every compact manifold has nite topology.
More generally, for any manifold, X, let C be a compact subset of
X. Then by HeineBorel we can extract from the cover, U, a nite
subcollection
U
i
= U

i
,
i
I , i = 1, . . . , N
that covers C, hence letting U =

U
i
, weve proved
Theorem 5.3.9. If X is an n-dimensional manifold and C a com-
pact subset of X, then there exists an open neighborhood, U, of C in
X having nite topology.
We can in fact even strengthen this further. Let U
0
be any open
neighborhood of C in X. Then in the theorem above we can replace
X by U
0
to conclude
Theorem 5.3.10. Let X be a manifold, C a compact subset of X
and U
0
an open neighborhood of C in X. Then there exists an open
neighborhood, U, of C in X, U contained in U
0
, having nite topol-
ogy.
We will justify the term nite topology by devoting the rest of
this section to proving
Theorem 5.3.11. Let X be an n-dimensional manifold. If X has
nite topology the DeRham cohomology groups, H
k
(X), k = 0, . . . , n
and the compactly supported DeRham cohomology groups, H
k
c
(X),
k = 0, . . . , n are nite dimensional vector spaces.
246 Chapter 5. Cohomology via forms
The basic ingredients in the proof of this will be the Mayer
Victoris techniques that we developed in 5.2 and the following ele-
mentary result about vector spaces.
Lemma 5.3.12. Let V
i
, i = 1, 2, 3, be vector spaces and
(5.3.2) V
1

V
2

V
3
an exact sequence of linear maps. Then if V
1
and V
3
are nite di-
mensional, so is V
2
.
Proof. Since V
3
is nite dimensional, the image of is of dimension,
k < , so there exist vectors, v
i
, i = 1, . . . , k in V
2
having the
property that
(5.3.3) Image = span{(v
i
) , i = 1, . . . , k}.
Now let v be any vector in V
2
. Then (v) is a linear combination
(v) =
k

i=1
c
i
(v
i
) c
i
R
of the vectors (v
i
) by (5.3.3), so
(5.3.4) v

= v
k

i=1
c
i
v
i
is in the kernel of and hence, by the exactness of (5.3.2), in the im-
age of . But V
1
is nite dimensional, so (V
1
) is nite dimensional.
Letting v
k+1
, . . . , v
m
be a basis of (V
1
) we can by (5.3.4) write v as
a sum, v =

m
i=1
c
i
v
i
. In other words v
1
, . . . , v
m
is a basis of V
2
.
Well now prove Theorem 5.3.4. Our proof will be by induction on
the number of open sets in a good cover of X. More specically let
U = {U
i
, i = 1, . . . , N}
be a good cover of X. If N = 1, X = U
1
and hence X is dieomorphic
to R
n
, so
H
k
(X) = {0} for k > 0
5.3 Good covers 247
and H
k
(X) = R for k = 0, so the theorem is certainly true in
this case. Lets now prove its true for arbitrary N by induction.
Let U be the open subset of X obtained by forming the union of
U
2
, . . . , U
N
. We can think of U as a manifold in its own right, and
since {U
i
, i = 2, . . . , N} is a good cover of U involving only N 1
sets, its cohomology groups are nite dimensional by the induction
assumption. The same is also true of the intersection of U with U
1
.
It has the N 1 sets, U U
i
, i = 2, . . . , N as a good cover, so its
cohomology groups are nite dimensional as well. To prove that the
theorem is true for X we note that X = U
1
U and that one has an
exact sequence
H
k1
(U
1
U)

H
k
(X)
i

H
k
(U
1
) H
k
(U)
by MayerVictoris. Since the right hand and left hand terms are nite
dimensional it follows from Lemma 5.3.12 that the middle term is
also nite dimensional.
The proof works practically verbatim for compactly supported co-
homology. For N = 1
H
k
c
(X) = H
k
c
(U
1
) = H
k
c
(R
n
)
so all the cohomology groups of H
k
(X) are nite in this case, and
the induction N 1 N follows from the exact sequence
H
k
c
(U
1
) H
k
c
(U)
j

H
k
c
(X)

H
k+1
c
(U
1
U) .
Remark 5.3.13. A careful analysis of the proof above shows that
the dimensions of the H
k
(X)s are determined by the intersection
properties of the U
i
s, i.e., by the list of multi-indices, I, for which
th intersections (5.3.1) are non-empty.
This collection of multi-indices is called the nerve of the cover,
U = {U
i
, i = 1, . . . , N}, and this remark suggests that there should
be a cohomology theory which has as input the nerve of U and as
output cohomology groups which are isomorphic to the DeRham
cohomology groups. Such a theory does exist and a nice account of it
can be found in Frank Warners book, Foundations of Dierentiable
Manifolds and Lie Groups. (See the section on

Cech cohomology in
Chapter 5.)
248 Chapter 5. Cohomology via forms
Exercises.
1. Let U be a bounded open subset of R
n
. A continuous function
: U [0, )
is called an exhaustion function if it is proper as a map of U into
[0, ); i.e., if, for every a > 0,
1
([0, a]) is compact. For x U let
d(x) = inf {|x y| , y R
n
U} ,
i.e., let d(x) be the distance from x to the boundary of U. Show
that d(x) > 0 and that d(x) is continuous as a function of x. Conclude
that
0
= 1/d is an exhaustion function.
2. Show that there exists a C

exhaustion function,
0
: U
[0, ), with the property
0

2
0
where
0
is the exhaustion func-
tion in exercise 1.
Hints: For i = 2, 3, . . . let
C
i
=
_
x U ,
1
i
d(x)
1
i 1
_
and
U
i
=
_
x U ,
1
i + 1
< d(x) <
1
i 2
_
.
Let
i
C

0
(U
i
),
i
0, be a bump function which is identically
one on C
i
and let
0
=

i
2

i
+ 1.
3. Let U be a bounded open convex subset of R
n
containing the
origin. Show that there exists an exhaustion function
: U R, (0) = 1 ,
having the property that is a monotonically increasing function of
t along the ray, tx, 0 t 1, for all points, x, in U. Hints:
(a) Let (x), 0 (x) 1, be a C

function which is one outside


a small neighborhood of the origin in U and is zero in a still smaller
5.3 Good covers 249
neighborhood of the origin. Modify the function,
0
, in the previous
exercise by setting (x) = (x)
0
(x) and let
(x) =
_
1
0
(sx)
ds
s
+ 1 .
Show that for 0 t 1
(5.3.5)
d
dt
(tx) = (tx)/t
and conclude from (5.3.4) that is monotonically increasing along
the ray, tx, 0 t 1.
(b) Show that for 0 < < 1,
(x) (y)
where y is a point on the ray, tx, 0 t 1 a distance less than |x|
from X.
(c) Show that there exist constants, C
0
and C
1
, C
1
> 0 such that
(x) =
C
1
d(x)
+C
0
.
Sub-hint: In part (b) take to be equal to
1
2
d(x)/|x|.
4. Show that every bounded, open convex subset, U, of R
n
is dif-
feomorphic to R
n
. Hints:
(a) Let (x) be the exhaustion function constructed in exercise 3
and let
f : U R
n
be the map: f(x) = (x)x. Show that this map is a bijective map of
U onto R
n
.
(b) Show that for x U and v R
n
(df)
x
v = (x)v + d
x
(v)x
and conclude that df
x
is bijective at x, i.e., that f is locally a dieo-
morphism of a neighborhood of x in U onto a neighborhood of f(x)
in R
n
.
250 Chapter 5. Cohomology via forms
(c) Putting (a) and (b) together show that f is a dieomorphism
of U onto R
n
.
5. Let U R be the union of the open intervals, k < x < k + 1,
k an integer. Show that U doesnt have nite topology.
6. Let V R
2
be the open set obtained by deleting from R
2
the points, p
n
= (0, n), n an integer. Show that V doesnt have nite
topology. Hint: Let
n
be a circle of radius
1
2
centered about the point
p
n
. Using exercises 1617 of 2.1 show that there exists a closed C

-
one-form,
n
on V with the property that
_
n

n
= 1 and
_
m

n
= 0
for m = n.
7. Let X be an n-dimensional manifold and U = {U
i
, i = 1, 2} a
good cover of X. What are the cohomology groups of X if the nerve
of this cover is
(a) {1}, {2}
(b) {1}, {2}, {1, 2}?
8. Let X be an n-dimensional manifold and U = {U
i
, i = 1, 2, 3, }
a good cover of X. What are the cohomology groups of X if the
nerve of this cover is
(a) {1}, {2}, {3}
(b) {1}, {2}, {3}, {1, 2}
(c) {1}, {2}, {3}, {1, 2}, {1, 3}
(d) {1}, {2}, {3}, {1, 2}, {1, 3}, {2, 3}
(e) {1}, {2}, {3}, {1, 2}, {1, 3}, {2, 3}, {1, 2, 3}?
9. Let S
1
be the unit circle in R
3
parametrized by arc length:
(x, y) = (cos , sin). Let U
1
be the set: 0 < <
2
3
, U
2
the set:

2
< <
3
2
, and U
3
the set:
2
3
< <

3
.
(a) Show that the U
i
s are a good cover of S
1
.
(b) Using the previous exercise compute the cohomology groups of
S
1
.
5.4 Poincare duality 251
10. Let S
2
be the unit 2-sphere in R
3
. Show that the sets
U
i
= {(x
1
, x
2
, x
3
) S
2
, x
i
> 0}
i = 1, 2, 3 and
U
i
= {(x
1
, x
2
, x
3
) S
2
, x
i3
< 0} ,
i = 4, 5, 6, are a good cover of S
2
. What is the nerve of this cover?
11. Let X and Y be manifolds. Show that if they both have nite
topology, their product, X Y , does as well.
12. (a) Let X be a manifold and let U
i
, i = 1, . . . , N, be a good
cover of X. Show that U
i
R, i = 1, . . . , N, is a good cover of XR
and that the nerves of these two covers are the same.
(b) By Remark 5.3.13,
H
k
(X R) = H
k
(X) .
Verify this directly using homotopy techniques.
(c) More generally, show that for all > 0
(5.3.6) H
k
(X R

) = H
k
(X)
(i) by concluding that this has to be the case in view of the
Remark 5.3.13 and
(ii) by proving this directly using homotopy techniques.
5.4 Poincare duality
In this chapter weve been studying two kinds of cohomology groups:
the ordinary DeRham cohomology groups, H
k
, and the compactly
supported DeRham cohomology groups, H
k
c
. It turns out that these
groups are closely related. In fact if X is a connected, oriented n-
dimensional manifold and has nite topology, H
nk
c
(X) is the vector
space dual of H
k
(X). Well give a proof of this later in this section,
however, before we do well need to review some basic linear alge-
bra. Given two nite dimensional vector space, V and W, a bilinear
pairing between V and W is a map
(5.4.1) B : V W R
252 Chapter 5. Cohomology via forms
which is linear in each of its factors. In other words, for xed w W,
the map
(5.4.2)
w
: V R, v B(v, w)
is linear, and for v V , the map
(5.4.3)
v
: W R, w B(v, w)
is linear. Therefore, from the pairing (5.4.1) one gets a map
(5.4.4) L
B
: W V

, w
w
and since
w
1
+
w
2
(v) = B(v, w
1
+ w
2
) =
w
1
+w
2
(v), this map is
linear. Well say that (5.4.1) is a non-singular pairing if (5.4.4) is
bijective. Notice, by the way, that the roles of V and W can be
reversed in this denition. Letting B

(w, v) = B(v, w) we get an


analogous linear map
(5.4.5) L
B
: V W

and in fact
(5.4.6) (L
B
(v))(w) = (L
B
(w))(v) = B(v, w) .
Thus if
(5.4.7) : V (V

)

is the canonical identication of V with (V



)

given by the recipe


(v)() = (v)
for v V and V

, we can rewrite (5.4.6) more suggestively in
the form
(5.4.8) L
B
= (L
B
)

i.e., L
B
and L
B
are just the transposes of each other. In particular
L
B
is bijective if and only if L
B
is bijective.
Lets now apply these remarks to DeRham theory. Let X be a
connected, oriented n-dimensional manifold. If X has nite topology
the vector spaces, H
nk
c
(X) and H
k
(X) are both nite dimensional.
We will show that there is a natural bilinear pairing between these
5.4 Poincare duality 253
spaces, and hence by the discussion above, a natural linear mapping
of H
k
(X) into the vector space dual of H
n1
c
(X). To see this let
c
1
be a cohomology class in H
nk
c
(X) and c
2
a cohomology class
in H
k
(X). Then by (5.1.42) their product, c
1
c
2
, is an element of
H
n
c
(X), and so by (5.1.8) we can dene a pairing between c
1
and c
2
by setting
(5.4.9) B(c
1
, c
2
) = I
X
(c
1
c
2
) .
Notice that if
1

nk
c
(X) and
2

k
(X) are closed forms
representing the cohomology classes, c
1
and c
2
, then by (5.1.42) this
pairing is given by the integral
(5.4.10) B(c
1
, c
2
) =
_
X

1

2
.
Well next show that this bilinear pairing is non-singular in one
important special case:
Proposition 5.4.1. If X is dieomorphic to R
n
the pairing dened
by (5.4.9) is non-singular.
Proof. To verify this there is very little to check. The vector spaces,
H
k
(R
n
) and H
nk
c
(R
n
) are zero except for k = 0, so all we have to
check is that the pairing
H
n
c
(X) H
0
(X) R
is non-singular. To see this recall that every compactly supported
n-form is closed and that the only closed zero-forms are the constant
functions, so at the level of forms, the pairing (5.4.9) is just the
pairing
(, c)
n
(X) R c
_
X
,
and this is zero if and only if c is zero or is in d
n1
c
(X). Thus at
the level of cohomology this pairing is non-singular.
We will now show how to prove this result in general.
Theorem 5.4.2 (Poincare duality.). Let X be an oriented, con-
nected n-dimensional manifold having nite topology. Then the pair-
ing (5.4.9) is non-singular.
254 Chapter 5. Cohomology via forms
The proof of this will be very similar in spirit to the proof that
we gave in the last section to show that if X has nite topology its
DeRham cohomology groups are nite dimensional. Like that proof,
it involves MayerVictoris plus some elementary diagram-chasing.
The diagram-chasing part of the proof consists of the following
two lemmas.
Lemma 5.4.3. Let V
1
, V
2
and V
3
be nite dimensional vector spaces,
and let V
1

V
2

V
3
be an exact sequence of linear mappings. Then
the sequence of transpose maps
V

V
1
is exact.
Proof. Given a vector subspace, W
2
, of V
2
, let
W

2
= { V

2
; (w) = 0 for w W} .
Well leave for you to check that if W
2
is the kernel of , then W

2
is
the image of

and that if W
2
is the image of , W

2
is the kernel
of

. Hence if Ker = Image , Image

= kernel

.
Lemma 5.4.4 (the ve lemma). Let the diagram below be a com-
mutative diagram with the properties:
(i) All the vector spaces are nite dimensional.
(ii) The two rows are exact.
(iii) The linear maps,
i
, i = 1, 2, 4, 5 are bijections.
Then the map,
3
, is a bijection.
A
1

1
A
2

2
A
3

3
A
4

4
A
5

5
B
1

1
B
2

2
B
3

3
B
4

4
B
5
.
Proof. Well show that
3
is surjective. Given a
3
A
3
there exists
a b
4
B
4
such that
4
(b
4
) =
3
(a
3
) since
4
is bijective. More-
over,
5
(
4
(b
4
)) =
4
(
3
(a
3
)) = 0, by the exactness of the top row.
5.4 Poincare duality 255
Therefore, since
5
is bijective,
4
(b
4
) = 0, so by the exactness of the
bottom row b
4
=
3
(b
3
) for some b
3
B
3
, and hence

3
(
3
(b
3
)) =
4
(
3
(b
3
)) =
4
(b
4
) =
3
(a
3
) .
Thus
3
(a
3

3
(b
3
)) = 0, so by the exactness of the top row
a
3

3
(b
3
) =
2
(a
2
)
for some a
2
A
2
. Hence by the bijectivity of
2
there exists a b
2
B
2
with a
2
=
2
(b
2
), and hence
a
3

3
(b
3
) =
2
(a
2
) =
2
(
2
(b
2
)) =
3
(
2
(b
2
)) .
Thus nally
a
3
=
3
(b
3
+
2
(b
2
)) .
Since a
3
was any element of A
3
this proves the surjectivity of
3
.
One can prove the injectivity of
3
by a similar diagram-chasing
argument, but one can also prove this with less duplication of eort
by taking the transposes of all the arrows in Figure 5.4.1 and noting
that the same argument as above proves the surjectivity of

3
: A

3

B

3
.
To prove Theorem 5.4.2 we apply these lemmas to the diagram
below. In this diagram U
1
and U
2
are open subsets of X, M is U
1
U
2
and the vertical arrows are the mappings dened by the pairing
(5.4.9). We will leave for you to check that this is a commutative
diagram up to sign. (To make it commutative one has to replace
some of the vertical arrows, , by their negatives: .) This is easy
to check except for the commutative square on the extreme left. To
check that this square commutes, some serious diagram-chasing is
required.
//
H
n(k1)
(M)
//
H
nk
(U
1
U
2
)
//
H
nk
(U
1
)

H
nk
(U
2
)
//
H
nk
(M)
//
//
H
k1
c
(M)
OO
//
H
k
c
(U
1
U
2
)
OO
//
H
k
c
(U
1
) H
k
c
(U
2
)
OO
//
H
k
c
(M)
OO
//
Figure 5.4.2
256 Chapter 5. Cohomology via forms
By MayerVictoris the bottom row of this gure is exact and by
MayerVictoris and Lemma 5.4.3 the top row of this gure is exact.
hence we can apply the ve lemma to Figure 5.4.2 and conclude:
Lemma 5.4.5. If the maps
(5.4.11) H
k
(U) H
nk
c
(U)

dened by the pairing (5.4.9) are bijective for U


1
, U
2
and U
1
U
2
,
they are also bijective for M = U
1
U
2
.
Thus to prove Theorem 5.4.2 we can argue by induction as in 5.3.
Let U
1
, U
2
, . . . , U
N
be a good cover of X. If N = 1, then X = U
1
and, hence, since U
1
is dieomorphic to R
n
, the map (5.4.12) is
bijective by Proposition 5.4.1. Now lets assume the theorem is true
for manifolds involving good covers by k open sets where k is less
than N. Let U

= U
1
U
N1
and U

= U
N
. Since
U

= U
1
U
N
U
N1
U
N
it can be covered by a good cover by k open sets, k < N, and hence
the hypotheses of the lemma are true for U

, U

and U

. Thus
the lemma says that (5.4.12) is bijective for the union, X, of U

and
U

.
Exercises.
1. (The push-forward operation in DeRham cohomology.) Let
X be an m-dimensional manifold, Y an n-dimensional manifold and
f : X Y a C

map. Suppose that both of these manifolds are


oriented and connected and have nite topology. Show that there
exists a unique linear map
(5.4.12) f

: H
mk
c
(X) H
nk
c
(Y )
with the property
(5.4.13) B
Y
(f

c
1
, c
2
) = B
X
(c
1
, f

c
2
)
for all c
1
H
mk
c
(X) and c
2
H
k
(Y ). (In this formula B
X
is the
bilinear pairing (5.4.9) on X and B
Y
is the bilinear pairing (5.4.9)
on Y .)
5.4 Poincare duality 257
2. Suppose that the map, f, in exercise 1 is proper. Show that
there exists a unique linear map
(5.4.14) f

: H
mk
(X) H
nk
(Y )
with the property
(5.4.15) B
Y
(c
1
, f

c
2
) = (1)
k(mn)
B
X
(f

c
1
, c
2
)
for all c
1
H
k
c
(Y ) and c
2
H
mk
(X), and show that, if X and
Y are compact, this mapping is the same as the mapping, f

, in
exercise 1.
3. Let U be an open subset of R
n
and let f : U R U be the
projection, f(x, t) = x. Show that there is a unique linear mapping
(5.4.16) f

:
k+1
c
(U R)
k
c
(U)
with the property
(5.4.17)
_
U
f

=
_
UR
f

for all
k+1
c
(U R) and
nk
(U).
Hint: Let x
1
, . . . , x
n
and t be the standard coordinate functions
on R
n
R. By 2.2, exercise 5 every (k +1)-form,
k+1
c
(U R)
can be written uniquely in reduced form as a sum
=

f
I
dt dx
I
+

g
J
dx
J
over multi-indices, I and J, which are strictly increasing. Let
(5.4.18) f

I
__
R
f
I
(x, t) dt
_
dx
I
.
4. Show that the mapping, f

, in exercise 3 satises f

d = df

.
5. Show that if is a closed compactly supported k + 1-form on
U R then
(5.4.19) [f

] = f

[]
where f

is the mapping (5.4.13) and f

the mapping (5.4.17).


258 Chapter 5. Cohomology via forms
6. (a) Let U be an open subset of R
n
and let f : U R

U
be the projection, f(x, t) = x. Show that there is a unique linear
mapping
(5.4.20) f

:
k+
c
(U R

)
k
c
(U)
with the property
(5.4.21)
_
U
f

=
_
UR

for all
k+
c
(U R

) and
nk
(U).
Hint: Exercise 3 plus induction on .
(b) Show that for
k+
c
(U R

)
df

= f

d .
(c) Show that if is a closed, compactly supported k +-form on
X R

(5.4.22) f

[] = [f

]
where f

: H
k+
c
(U R

) H
k
c
(U) is the map (5.4.13).
7. Let X be an n-dimensional manifold and Y an m-dimensional
manifold. Assume X and Y are compact, oriented and connected,
and orient X Y by giving it its natural product orientation. Let
f : X Y Y
be the projection map, f(x, y) = y. Given

m
(X Y )
and p Y , let
(5.4.23) f

(p) =
_
X

where
p
: X X Y is the inclusion map,
p
(x) = (x, p).
(a) Show that the function f

dened by (5.5.24) is C

, i.e., is in

0
(Y ).
5.5 Thom classes and intersection theory 259
(b) Show that if is closed this function is constant.
(c) Show that if is closed
[f

] = f

[]
where f

: H
n
(X Y ) H
0
(Y ) is the map (5.4.13).
8. (a) Let X be an n-dimensional manifold which is compact,
connected and oriented. Combining Poincare duality with exercise 12
in 5.3 show that
H
k+
c
(X R

) = H
k
c
(X) .
(b) Show, moreover, that if f : X R

X is the projection,
f(x, a) = x, then
f

: H
k+
c
(X R

) H
k
c
(X)
is a bijection.
9. Let X and Y be as in exercise 1. Show that the push-forward
operation (5.4.13) satises
f

; (c
1
f

c
2
) = f

c
1
c
2
for c
1
H
k
c
(X) and c
2
H

(Y ).
5.5 Thom classes and intersection theory
Let X be a connected, oriented n-dimensional manifold. If X has
nite topology its cohomology groups are nite dimensional, and
since the bilinear pairing, B, dened by (5.4.9) is non-singular we
get from this pairing bijective linear maps
L
B
: H
nk
c
(X) H
k
(X)

(5.5.1)
and
L

B
: H
nk
(X) H
k
c
(X)

. (5.5.2)
In particular, if : H
k
(X) R is a linear function (i.e., an element
of H
k
(X)

), then by (5.5.1) we can convert into a cohomology class


(5.5.3) L
1
B
() H
nk
c
(X) ,
260 Chapter 5. Cohomology via forms
and similarly if
c
: H
k
c
(X) R is a linear function, we can convert
it by (5.5.2) into a cohomology class
(5.5.4) (L

B
)
1
() H
nk
(X) .
One way that linear functions like this arise in practice is by in-
tegrating forms over submanifolds of X. Namely let Y be a closed,
oriented k dimensional submanifold of X. Since Y is oriented, we
have by (5.1.8) an integration operation in cohomology
I
Y
: H
k
c
(Y ) R,
and since Y is closed the inclusion map,
Y
, of Y into X is proper,
so we get from it a pull-back operation on cohomology
(
Y
)

: H
k
c
(X) H
k
c
(Y )
and by composing these two maps, we get a linear map,
Y
= I
Y

(
Y
)

, of H
k
c
(X) into R. The cohomology class
(5.5.5) T
Y
= L
1
B
(
Y
) H
k
c
(X)
associated with
Y
is called the Thom class of the manifold, Y and
has the dening property
(5.5.6) B(T
Y
, c) = I
Y
(

Y
c)
for c H
k
c
(X). Lets see what this dening property looks like at the
level of forms. Let
Y

nk
(X) be a closed k-form representing
T
Y
. Then by (5.4.9), the formula (5.5.6), for c = [], becomes the
integral formula
(5.5.7)
_
X

Y
=
_
Y

Y
.
In other words, for every closed form,
nk
c
(X) the integral of
over Y is equal to the integral over X of
Y
. A closed form,

Y
, with this reproducing property is called a Thom form for Y .
Note that if we add to
Y
an exact (n k)-form, d
nk1
(X),
we get another representative,
Y
+ , of the cohomology class, T
Y
,
and hence another form with this reproducing property. Also, since
the formula (5.5.7) is a direct translation into form language of the
5.5 Thom classes and intersection theory 261
formula (5.5.6) any closed (n k)-form,
Y
, with the reproducing
property (5.5.7) is a representative of the cohomology class, T
Y
.
These remarks make sense as well for compactly supported coho-
mology. Suppose Y is compact. Then from the inclusion map we get
a pull-back map
(
Y
)

: H
k
(X) H
k
(Y )
and since Y is compact, the integration operation, I
Y
, is a map of
H
k
(Y ) into R, so the composition of these two operations is a map,

Y
: H
k
(X) R
which by (5.5.3) gets converted into a cohomology class
T
Y
= L
1
B
(
Y
) H
nk
c
(X) .
Moreover, if
Y

nk
c
(X) is a closed form, it represents this coho-
mology class if and only if it has the reproducing property
(5.5.8)
_
X

Y
=
_
Y

for closed forms, , in


nk
(X). (Theres a subtle dierence, how-
ever, between formula (5.5.7) and formula (5.5.8). In (5.5.7) has
to be closed and compactly supported and in (5.5.8) it just has to
be closed.)
As above we have a lot of latitude in our choice of
Y
: we can
add to it any element of d
nk1
c
(X). One consequence of this is the
following.
Theorem 5.5.1. Given a neighborhood, U, of Y in X there exists
a closed form,
Y

nk
c
(U) , with the reproducing property
(5.5.9)
_
U

Y
=
_
Y

for closed forms,


k
(U).
Hence in particular,
Y
has the reproducing property (5.5.8) for
closed forms,
nk
(X). This result shows that the Thom form,

Y
, can be chosen to have support in an arbitrarily small neighbor-
hood of Y . To prove Theorem 5.5.1 we note that by Theorem 5.3.8
we can assume that U has nite topology and hence, in our deni-
tion of
Y
, we can replace the manifold, X, by the open submanifold,
262 Chapter 5. Cohomology via forms
U. This gives us a Thom form,
Y
, with support in U and with the
reproducing property (5.5.9) for closed forms
nk
(U).

Lets see what Thom forms actually look like in concrete examples.
Suppose Y is dened globally by a system of independent equations,
i.e., suppose there exists an open neighborhood, O, of Y in X, a C

map, f : O R

, and a bounded open convex neighborhood, V , of


the origin in R
n
such that
(i) The origin is a regular value of f .
(ii) f
1
(

V ) is closed in X . (5.5.10)
(iii) Y = f
1
(0) .
Then by (i) and (iii) Y is a closed submanifold of O and by (ii) its
a closed submanifold of X. Moreover, it has a natural orientation:
For every p Y the map
df
p
: T
p
X T
0
R

is surjective, and its kernel is T


p
Y , so from the standard orientation
of T
0
R

one gets an orientation of the quotient space,


T
p
X/T
p
Y ,
and hence since T
p
X is oriented, one gets, by Theorem 1.9.4, an
orientation on T
p
Y . (See 4.4, example 2.) Now let be an element
of

c
(X). Then f

is supported in f
1
(

V ) and hence by property


(ii) of (5.5.10) we can extend it to X by setting it equal to zero
outside O. We will prove
Theorem 5.5.2. If
(5.5.11)
_
V
= 1 ,
f

is a Thom form for Y .


To prove this well rst prove that if f

has property (5.5.7) for


some choice of it has this property for every choice of .
Lemma 5.5.3. Let
1
and
2
be forms in

c
(V ) with the property
(5.5.11). Then for every closed k-form,
k
c
(X)
_
X
f

1
=
_
X
f

2
.
5.5 Thom classes and intersection theory 263
Proof. By Theorem 3.2.1,
1

2
= d for some
1
c
(V ),
hence, since d = 0
(f

1
f

2
) = df

= d(f

) .
Therefore, by Stokes theorem, the integral over X of the expression
on the left is zero.
Now suppose = (x
1
, . . . , x

) dx
1
dx

, for in C

0
(V ). For
t 1 let
(5.5.12)
t
= t

_
x
1
t
, ,
x

t
_
dx
1
dx

.
This form is supported in the convex set, tV , so by Lemma 5.5.3
(5.5.13)
_
X
f

t
=
_
X
f


for all closed forms
k
c
(X). Hence to prove that f

has the
property (5.5.7) it suces to prove
(5.5.14) Lim
t0
_
f

t
=
_
Y

Y
.
Well prove this by proving a stronger result.
Lemma 5.5.4. The assertion (5.5.14) is true for every k-form

k
c
(X).
Proof. The canonical form theorem for submersions (see Theorem 4.3.6)
says that for every p Y there exists a neighborhood U
p
of p in Y ,
a neighborhood, W of 0 in R
n
, and an orientation preserving dieo-
morphism : (W, 0) (U
p
, p) such that
(5.5.15) f =
where : R
n
R

is the canonical submersion, (x


1
, . . . , x
n
) =
(x
1
, . . . , x

). Let U be the cover of O by the open sets, O Y and


the U
p
s. Choosing a partition of unity subordinate to this cover it
suces to verify (5.5.14) for in
k
c
(OY ) and in
k
c
(U
p
). Lets
rst suppose is in
k
c
(OY ). Then f(supp) is a compact subset
of R

{0} and hence for t small f(supp) is disjoint from tV , and


264 Chapter 5. Cohomology via forms
both sides of (5.5.14) are zero. Next suppose that is in
k
c
(U
p
).
Then

is a compactly supported k-form on W so we can write it


as a sum

h
I
(x) dx
I
, h
I
C

0
(W)
the Is being strictly increasing multi-indices of length k. Let I
0
=
( + 1,
2
+ 2, . . . , n). Then
(5.5.16)

= t

(
x
1
t
, ,
x

t
)h
I
0
(x
1
, . . . , x
n
) dx
r
dx
n
and by (5.5.15)

(f

t
) =

and hence since is orientation preserving


_
Up
f

t
= t

_
R
n

_
x
1
t
, ,
x

t
_
h
I
0
(x
1
, . . . , x
n
) dx
=
_
R
n
(x
1
, . . . , x

)h
I
0
(tx
1
, . . . , tx

, x
+1
, . . . , x
n
) dx
and the limit of this expression as t tends to zero is
_
(x
1
, . . . , x

)h
I
0
(0, . . . , 0 , x
+1
, . . . , x
n
) dx
1
. . . dx
n
or
_
h
I
(0, . . . , 0 , x
+1
, . . . , x
n
) dx
+1
dx
n
. (5.5.17)
This, however, is just the integral of

over the set


1
(0) W.
By (5.5.14) maps this set dieomorphically onto Y U
p
and by
our recipe for orienting Y this dieomorphism is an orientation-
preserving dieomorphism, so the integral (5.5.17) is equal to the
integral of over Y .
Well now describe some applications of Thom forms to topological
intersection theory. Let Y and Z be closed, oriented submanifolds of
X of dimensions k and where k + = n, and lets assume one
of them (say Z) is compact. We will show below how to dene an
intersection number, I(Y, Z), which on the one hand will be a
topological invariant of Y and Z and on the other hand will actually
5.5 Thom classes and intersection theory 265
count, with appropriate -signs, the number of points of intersection
of Y and Z when they intersect non-tangentially. (Thus this notion
is similar to the notion of degree f for a C

mapping f. On the
one hand degree f is a topological invariant of f. Its unchanged
if we deform f by a homotopy. On the other hand if q is a regular
value of f, degree f counts with appropriate -signs the number
of points in the set, f
1
(q).)
Well rst give the topological denition of this intersection num-
ber. This is by the formula
(5.5.18) I(Y, Z) = B(T
Y
, T
Z
)
where T
Y
H

(X) and T
Z
H
k
c
(X) and B is the bilinear pairing
(5.4.9). If
Y

(X) and
Z

k
c
(X) are Thom forms representing
T
Y
and T
Z
, (5.5.18) can also be dened as the integral
(5.5.19) I(Y, Z) =
_
X

Y

Z
or by (5.5.9), as the integral over Y ,
(5.5.20) I(Y, Z) =
_
Y

Z
or, since
Y

Z
= (1)
k

Z

Y
, as the integral over Z
(5.5.21) I(X, Y ) = (1)
k
_
Z

Y
.
In particular
(5.5.22) I(Y, Z) = (1)
k
I(Z, Y ).
As a test case for our declaring I(Y, Z) to be the intersection number
of Y and Z we will rst prove:
Proposition 5.5.5. If Y and Z dont intersect, then I(Y, Z) = 0.
Proof. If Y and Z dont intersect then, since Y is closed, U = XY
is an open neighborhood of Z in X, therefore since Z is compact there
exists by Theorem 5.5.1 a Thom form,
Z
in

c
(U). Thus

Z
= 0,
and so by (5.5.20) I(Y, Z) = 0.
266 Chapter 5. Cohomology via forms
Well next indicate how one computes I(Y, Z) when Y and Z in-
tersect non-tangentially, or, to use terminology more in current
usage, when their intersection is transversal. Recall that at a point
of intersection, p Y Z, T
p
Y and T
p
Z are vector subspaces of T
p
X.
Denition 5.5.6. Y and Z intersect transversally if for every p
Y Z, T
p
Y T
p
Z = {0}.
Since n = k + = dimT
p
Y + dimT
p
Z = dimT
p
X, this condition
is equivalent to
(5.5.23) T
p
X = T
p
Y T
p
Z ,
i.e., every vector, u T
p
X, can be written uniquely as a sum,
u = v + w, with v T
p
Y and w T
p
Z. Since X, Y and Z are
oriented, their tangent spaces at p are oriented, and well say that
these spaces are compatibly oriented if the orientations of the two
sides of (5.5.23) agree. (In other words if v
1
, . . . , v
k
is an oriented
basis of T
p
Y and w
1
, . . . , w

is an oriented basis of T
p
Z, the n vec-
tors, v
1
, . . . , v
k
, w
1
, . . . , w

, are an oriented basis of T


p
X.) We will
dene the local intersection number, I
p
(Y, Z), of Y and Z at p to be
equal to +1 if X, Y and Z are compatibly oriented at p and to be
equal to 1 if theyre not. With this notation well prove
Theorem 5.5.7. If Y and Z intersect transversally then Y Z is a
nite set and
(5.5.24) I(Y, Z) =

pY Z
I
p
(Y, Z).
To prove this we rst need to show that transverse intersections
look nice locally.
Theorem 5.5.8. If Y and Z intersect transversally, then for every
p Y Z, there exists an open neighborhood, V
p
, of p in X, an open
neighborhood, U
p
, of the origin in R
n
and an orientation preserving
dieomorphism

p
: V
p
U
p
which maps V
p
Y dieomorphically onto the subset of U
p
dened
by the equations: x
1
= = x

= 0, and maps V Z onto the subset


of U
p
dened by the equations: x
+1
= = x
n
= 0.
5.5 Thom classes and intersection theory 267
Proof. Since this result is a local result, we can assume that X is R
n
and hence by Theorem 4.2.7 that there exists a neighborhood, V
p
, of p
in R
n
and submersions f : (V
p
, p) (R

, 0) and g : (V
p
, p) (R
k
, 0)
with the properties
V
p
Y = f
1
(0) (5.5.25)
and
v
p
Z = g
1
(0) . (5.5.26)
Moreover, by (4.3.4)
T
p
Y = (df
p
)
1
(0)
and
T
p
Z = (dg
p
)
1
(0) .
Hence by (5.5.23), the equations
(5.5.27) df
p
(v) = dg
p
(v) = 0
for v T
p
X imply that v = 0. Now let
p
: V
p
R
n
be the map
(f, g) : V
p
R

R
k
= R
n
.
Then by (5.5.27), d
p
is bijective, therefore, shrinking V
p
if necessary,
we can assume that
p
maps V
p
dieomorphically onto a neighbor-
hood, U
p
, of the origin in R
n
, and hence by (5.5.25) and (5.5.26),

p
maps V
p
Y onto the set: x
1
= = x

= 0 and maps V
p
Z
onto the set: x
+1
= = x
n
= 0. Finally, if isnt orientation
preserving, we can make it so by composing it with the involution,
(x
1
, . . . , x
n
) (x
1
, x
2
, . . . , x
n1
, x
n
).
From this result we deduce:
Theorem 5.5.9. If Y and Z intersect transversally, their intersec-
tion is a nite set.
Proof. By Theorem 5.5.8 the only point of intersection in V
p
is p
itself. Moreover, since Y is closed and Z is compact, Y Z is compact.
268 Chapter 5. Cohomology via forms
Therefore, since the V
p
s cover Y Z we can extract a nite subcover
by the HeineBorel theorem. However, since no two V
p
s cover the
same point of Y Z, this cover must already be a nite subcover.
We will now prove Theorem 5.5.7. Since Y is closed, the map,
Y
:
Y X is proper, so by Theorem 3.4.2 there exists a neighborhood,
U, of Z in X such that U Y is contained in the union of the open
sets, V
p
, above. Moreover by Theorem 5.5.1 we can choose
Z
to
be supported in U and by Theorem 5.3.2 we can assume that U
has nite topology, so were reduced to proving the theorem with X
replaced by U and Y replaced by Y U. Let
O =
_
_
V
p
_
U ,
let
f : O R

be the map whose restriction to V


p
U is
p
where is, as in
(5.5.15), the canonical submersion of R
n
onto R

, and nally let V


be a bounded convex neighborhood of R

, whose closure is contained


in the intersection of the open sets,
p
(V
p
U). Then f
1
(

V ) is a
closed subset of U, so if we replace X by U and Y by Y U, the data
(f, O, V ) satisfy the conditions (5.5.10). Thus to prove Theorem 5.5.7
it suces by Theorem 5.5.2 to prove this theorem with

Y
=
p
(Y )f

on V
p
O where
p
(Y ) = +1 or 1 depending on whether the orien-
tation of Y V
p
in Theorem 5.5.2 coincides with the given orientation
of Y or not. Thus
I(Y, Z) = (1)
k
I(Z, Y )
= (1)
k

p
(Y )
_
Z

Z
f

= (1)
k

p
(Y )
_
Z

p
(1)
k

p
(Y )
_
ZVp
(
p

Z
)

.
5.5 Thom classes and intersection theory 269
But
p

Z
maps an open neighborhood of p in U
p
Z dieomor-
phically onto V , and is compactly supported in V so by (5.5.11)
_
ZUp
(
p

Z
)

=
p
(Z)
_
V
=
p
(Z)
where
p
(Z) = +1 or 1 depending on whether
p

Z
is orien-
tation preserving or not. Thus nally
I(Y, Z) =

(1)
k

p
(Y )
p
(Z) .
We will leave as an exercise the task of unraveling these orientations
and showing that
(1)
k

p
(Y )
p
(Z) = I
p
(Y, Z)
and hence that I(Y, Z) =

p
I
p
(Y, Z).
Exercises.
1. Let X be a connected, oriented n-dimensional manifold, W a
connected, oriented -dimensional manifold, f : X W a C

map,
and Y a closed submanifold of X of dimension k = n. Suppose Y
is a level set of the map, f, i.e., suppose that q is a regular value
of f and that Y = f
1
(q). Show that if is in

c
(Z) and its integral
over Z is 1, then one can orient Y so that
Y
= f

is a Thom form
for Y .
Hint: Theorem 5.5.2.
2. In exercise 1 show that if Z X is a compact oriented -
dimensional submanifold of X then
I(Y, Z) = (1)
k
deg(f
Z
) .
3. Let q
1
be another regular value of the map, f : X W, and
let Y
1
= f
1
(q). Show that
I(Y, Z) = I(Y
1
, Z) .
4. (a) Show that if q is a regular value of the map, f
Z
: Z W
then Z and Y intersect transversally.
270 Chapter 5. Cohomology via forms
(b) Show that this is an if and only if proposition: If Y and Z
intersect transversally then q is a regular value of the map, f
Z
.
5. Suppose q is a regular value of the map, f
Z
. Show that p is
in Y Z if and only if p is in the pre-image (f
Z
)
1
(q) of q and
that
I
p
(X, Y ) = (1)
k

p
where
p
is the orientation number of the map, f
Z
, at p, i.e.,
p
= 1
if f
Z
is orientation-preserving at p and
p
= 1 if f
Z
is
orientation-reversving at p.
6. Suppose the map f : X W is proper. Show that there exists
a neighborhood, V , of q in W having the property that all points of
V are regular values of f.
Hint: Since q is a regular value of f there exists, for every p
f
1
(q) a neighborhood, U
p
of p, on which f is a submersion. Con-
clude, by Theorem 3.4.2, that there exists a neighborhood, V , of q
with f
1
(V )

U
p
.
7. Show that in every neighborhood, V
1
, of q in V there exists a
point, q
1
, whose pre-image
Y
1
= f
1
(q
1
)
intersects Z transversally. (Hint: Exercise 4 plus Sards theorem.)
Conclude that one can deform Y an arbitrarily small amount so
that it intersects Z transversally.
8. (Intersection theory for mappings.) Let X be an oriented, con-
nected n-dimensional manifold, Z a compact, oriented -dimensional
submanifold, Y an oriented manifold of dimension k = n and
f : Y X a proper C

map. Dene the intersection number of f


with Z to be the integral
I(f, Z) =
_
Y
f

Z
.
(a) Show that I(f, Z) is a homotopy invariant of f, i.e., show that
if f
i
: Y X, i = 0, 1 are proper C

maps and are properly homo-


topic, then
I(f
0
, Z) = I(f
1
, Z) .
5.6 The Lefshetz theorem 271
(b) Show that if Y is a closed submanifold of X of dimension k =
n and
Y
: Y X is the inclusion map
I(
Y
, Z) = I(Y, Z) .
9. (a) Let X be an oriented, connected n-dimensional manifold
and let Z be a compact zero-dimensional submanifold consisting of
a single point, z
0
X. Show that if is in
n
c
(X) then is a Thom
form for Z if and only if its integral is 1.
(b) Let Y be an oriented n-dimensional manifold and f : Y X
a C

map. Show that for Z = {z


0
} as in part a
I(f, Z) = deg(f) .
5.6 The Lefshetz theorem
In this section well apply the intersection techniques that we devel-
oped in 5.5 to a concrete problem in dynamical systems: counting
the number of xed points of a dierentiable mapping. The Brouwer
xed point theorem, which we discussed in 3.6, told us that a C

map of the unit ball into itself has to have at least one xed point.
The Lefshetz theorem is a similar result for manifolds. It will tell us
that a C

map of a compact manifold into itself has to have a xed


point if a certain topological invariant of the map, its global Lefshetz
number, is non-zero.
Before stating this result, we will rst show how to translate the
problem of counting xed points of a mapping into an intersection
number problem. Let X be an oriented, compact n-dimensional man-
ifold and f : X X a C

map. Dene the graph of f in X X to


be the set
(5.6.1)
f
= {(x, f(x)) ; x X} .
Its easy to see that this is an n-dimensional submanifold of XX
and that this manifold is dieomorphic to X itself. In fact, in one
direction, there is a C

map
(5.6.2)
f
: X
f
,
f
(x) = (x, f(x)) ,
and, in the other direction, a C

map
(5.6.3) :
f
X , (x, f(x)) x,
272 Chapter 5. Cohomology via forms
and its obvious that these maps are inverses of each other and hence
dieomorphisms. We will orient
f
by requiring that
f
and be
orientation-preserving dieomorphisms.
An example of a graph is the graph of the identity map of X onto
itself. This is the diagonal in X X
(5.6.4) = {(x, x) , x X}
and its intersection with
f
is the set
(5.6.5) {(x, x) , f(x) = x} ,
which is just the set of xed points of f. Hence a natural way to
count the xed points of f is as the intersection number of
f
and
in X X. To do so we need these three manifolds to be oriented,
but, as we noted above,
f
and acquire orientations from the
identications (5.6.2) and, as for X X, well give it its natural
orientation as a product of oriented manifolds. (See 4.5.)
Denition 5.6.1. The global Lefshetz number of X is the intersec-
tion number
(5.6.6) L(f) = I(
f
, ) .
In this section well give two recipes for computing this number:
one by topological methods and the other by making transversality
assumptions and computing this number as a sum of local intersec-
tion numbers a la (5.5.24). Well rst show what one gets from the
transversality approach.
Denition 5.6.2. The map, f, is a Lefshetz map if
f
and in-
tersect transversally.
Lets see what being Lefshetz entails. Suppose p is a xed point of
f. Then at q = (p, p)
f
(5.6.7) T
q
(
f
) = (d
f
)
p
T
p
X = {(v, df
p
(v)) , v T
p
X}
and, in particular, for the identity map,
(5.6.8) T
q
() = {(v, v) , v T
p
X} .
Therefore, if and
f
are to intersect transversally, the intersection
of (5.6.7) and (5.6.8) inside T
q
(X X) has to be the zero space. In
other words if
(5.6.9) (v, df
p
(v)) = (v, v)
5.6 The Lefshetz theorem 273
then v = 0. But the identity (5.6.9) says that v is a xed point of
df
p
, so transversality at p amounts to the assertion
(5.6.10) df
p
(v) = v v = 0 ,
or in other words the assertion that the map
(5.6.11) (I df
p
) : T
p
X T
p
X
is bijective. Well now prove
Proposition 5.6.3. The local intersection number I
p
(
f
, ) is 1 if
(5.6.11) is orientation-preserving and 1 if not.
In other words I
p
(
f
, ) is the sign of det(I df
p
). To prove this
let e
1
, . . . , e
n
be an oriented basis of T
p
X and let
(5.6.12) df
p
(e
i
) =

a
j,i
e
j
.
Now set
v
i
= (e
i
, 0) T
q
(X X)
and
w
i
= (0, e
i
) T
q
(X X) .
Then by the deifnition of the product orientation on X X
(5.6.13) v
1
, . . . , v
n
, w
1
, . . . , w
n
is an oriented basis of T
q
(X X) and by (5.6.7)
(5.6.14) v
1
+

a
j,i
w
j
. . . , v
n
+

a
j,n
w
j
is an oriented basis of T
q

f
and
(5.6.15) v
1
+w
1
, . . . , v
n
+w
n
is an oriented basis of T
q
. Thus I
p
(
f
, ) = +1 or 1 depending
on whether or not the basis
v
1
+

a
j,i
w
j
, . . . , v
n
+

a
j,n
w
j
, v
1
+w
1
, . . . , v
n
+w
n
274 Chapter 5. Cohomology via forms
of T
q
(X X) is compatibly oriented with the basis (5.6.12). Thus
I
p
(
f
, ) = +1 or 1 depending on whether the determinant of the
2n 2n matrix relating these two bases:
(5.6.16)
_
I A
I , I
_
, A = [a
i.j
]
is positive or negative. However, its easy to see that this determinant
is equal to det(I A) and hence by (5.6.12) to det(I df
p
). Hint: By
elementary row operations (5.6.16) can be converted into the matrix
_
I , A
0 , I A
_
.

Lets summarize what weve shown so far.


Theorem 5.6.4. The map, f : X X, is a Lefshetz map if and
only if, for every xed point, p, the map
I df
p
: T
p
X T
p
X ()
is bijective. Moreover for Lefshetz maps
(5.6.17) L(f) =

pf
(p)
L
p
(f)
where L
p
(f) = +1 if () is orientation-preserving and 1 if its
orientation-reversing.
Well next describe how to compute L(f) as a topological invariant
of f. Let

be the inclusion map of


f
into X X and let T


H
n
(X X) be the Thom class of . Then by (5.5.20)
L(f) = I

f
(

)
and hence since the mapping,
f
: X X X dened by (5.6.2) is
an orientation-preserving dieomorphism of X onto
f
(5.6.18) L(f) = I
X
(

f
T

) .
To evaluate the expression on the right well need to know some
facts about the cohomology groups of product manifolds. The main
result on this topic is the K unneth theorem, and well take up the
5.6 The Lefshetz theorem 275
formulation and proof of this theorem in 5.7. First, however, well
describe a result which follows from the K unneth theorem and which
will enable us to complete our computation of L(f).
Let
1
and
2
be the projection of XX onto its rst and second
factors, i.e., let

i
: X X X i = 1, 2
be the map,
i
(x
1
, x
2
) = x
i
. Then by (5.6.2)

1

f
= i d
X
(5.6.19)
and

2

f
= f . (5.6.20)
Lemma 5.6.5. If
1
and
2
are in
n
(X) then
(5.6.21)
_
XX

2
=
__
X

1
___
X

2
_
.
Proof. By a partition of unity argument we can assume that
i
has
compact support in a parametrizable open set, V
i
. Let U
i
be an open
subset of R
n
and
i
: U
i
V
i
an orientation-preserving dieomor-
phism. Then

i
=
i
dx
1
dx
n
with
i
C

0
(U
i
), so the right hand side of (5.6.21) is the product
of integrals over R
n
:
(5.6.22)
_

1
(x) dx
_

2
(x) dx.
Moreover, since X X is oriented by its product orientation, the
map
: U
1
U
2
V
1
V
2
mapping (x, y) to (
1
(x) ,
2
(y)) is an orientation-preserving dieo-
morphism and since
i
=
i

2
) =

2
=
1
(x)
2
(y) dx
1
dx
n
dy
1
dy
n
and hence the left hand side of (5.6.21) is the integral over R
2n
of
the function,
1
(x)
2
(y), and therefore, by integration by parts, is
equal to the product (5.6.22).
276 Chapter 5. Cohomology via forms
As a corollary of this lemma we get a product formula for coho-
mology classes:
Lemma 5.6.6. If c
1
and c
2
are in H
n
(X) then
(5.6.23) I
XX
(

1
c
1

2
c
2
) = I
X
(c
1
)I
X
(c
2
) .
Now let d
k
= dimH
k
(X) and note that since X is compact,
Poincare duality tells us that d
k
= d

when = n k. In fact it
tells us even more. Let

k
i
, i = 1, . . . , d
k
be a basis of H
k
(X). Then, since the pairing (5.4.9) is non-singular,
there exists for = n k a dual basis

j
, j = 1, . . . , d

of H

(X) satisfying
(5.6.24) I
X
(
k
i

j
) =
ij
.
Lemma 5.6.7. The cohomology classes
(5.6.25)

k
s
, k + = n
for k = 0, . . . , n and 1 r, s d
k
, are a basis for H
n
(X X).
This is the corollary of the K unneth theorem that we alluded to
above (and whose proof well give in 5.7). Using these results well
prove
Theorem 5.6.8. The Thom class, T

, in H
n
(X X) is given ex-
plicitly by the formula
(5.6.26) T

k+=n
(1)

d
k

i=1

k
i

i
.
Proof. We have to check that for every cohomology class, c H
n
(X
X), the class, T

, dened by (5.6.26) has the reproducing property


(5.6.27) I
XX
(T

c) = I

c)
where

is the inclusion map of into X X. However the map

: X X X , x (x, x)
5.6 The Lefshetz theorem 277
is an orientation-preserving dieomorphism of X onto , so it suf-
ces to show that
(5.6.28) I
XX
(T

c) = I
X
(

c)
and by Lemma 5.6.7 it suces to verify (5.6.28) for cs of the form
c =

k
s
.
The product of this class with a typical summand of (5.6.26), for
instance, the summand
(5.6.29) (1)

i
, k

= n,
is equal, up to sign to,

k
s

i
.
Notice, however, that if k = k

this product is zero: For k < k

, k

+
is greater than k + and hence greater than n. Therefore

r
H
k

+
(X)
is zero since X is of dimension n, and for k > k

is greater than
and
k
s

i
is zero for the same reason. Thus in taking the product
of T

with c we can ignore all terms in the sum except for the terms,
k

= k and

= . For these terms, the product of (5.6.29) with c is


(1)
k

k
i

k
3

i
.
(Exercise: Check this. Hint: (1)

(1)

2
= 1.) Thus
T

c = (1)
k

k
i

k
s

i
and hence by Lemma 5.6.5 and (5.6.24)
I
XX
(T

c) = (1)
k

i
I
X
(
k
i

r
)I
X
(
k
s

i
)
= (1)
k

ir

is
= (1)
k

rs
.
278 Chapter 5. Cohomology via forms
On the other hand for c =

k
s

c =

k
s
= (
1

r
(
2

k
s
=

r

k
s
since

=
2

= id
X
.
So
I
X
(

c) = I
X
(

r

k
s
) = (1)
k

rs
by (5.6.24). Thus the two sides of (5.6.27) are equal.
Were now in position to compute L(f) , i.e., to compute the
expression I
X
(

f
T

) on the right hand side of (5.6.18). Since

i
,
i = 1, . . . , d

is a basis of H

(X) the linear mapping


(5.6.30) f

: H

(X) H

(X)
can be described in terms of this basis by a matrix, [f

ij
] with the
dening property
f

i
=

ji

j
.
Thus by (5.6.26), (5.6.19) and (5.6.20)

f
T

f
(1)

k+=n

1
u
k
i

i
=

(1)

i
(
1

f
)

k
i
(
2

f
)

i
=

(1)

k
i
f

i
=

(1)

ji

k
i

j
.
Thus by (5.6.24)
I
X
(

f
T

) =

(1)

ji
I
X
(
k
i

j
)
=

(1)

ji

ij
=
n

=0
(1)

i
f

ii
_
.
5.6 The Lefshetz theorem 279
But

i
f

i,i
is just the trace of the linear mapping (5.6.30) (see ex-
ercise 12 below), so we end up with the following purely topological
prescription of L(f).
Theorem 5.6.9. The Lefshetz number, L(f) is the alternating sum
(5.6.31)

(1)

Trace (f

where Trace (f

is the trace of the mapping


f

: H

(X) H

(X) .
Exercises.
1. Show that if f
0
: X X and f
1
: X X are homotopic C

mappings L(f
0
) = L(f
1
).
2. (a) The Euler characteristic, (X), of X is dened to be the
intersection number of the diagonal with itself in X X, i.e., the
self-intersection number
I(, ) = I
XX
(T

, T

) .
Show that if a C

map, f : X X is homotopic to the identity,


L
f
= (X).
(b) Show that
(5.6.32) (X) =
n

=0
(1)

dimH

(X) .
(c) Show that (X) = 0 if n is odd.
3. (a) Let S
n
be the unit n-sphere in R
n+1
. Show that if g :
S
n
S
n
is a C

map
L(g) = 1 + (1)
n
(deg) (g) .
(b) Conclude that if deg(g) = (1)
n+1
, then g has to have a xed
point.
280 Chapter 5. Cohomology via forms
4. Let f be a C

mapping of the closed unit ball, B


n+1
, into itself
and let g : S
n
S
n
be the restriction of f to the boundary of B
n+1
.
Show that if deg(g) = (1)
n+1
then the xed point of f predicted
by Brouwers theorem can be taken to be a point on the boundary
of B
n+1
.
5. (a) Show that if g : S
n
S
n
is the antipodal map, g(x) = x,
then deg(g) = (1)
n+1
.
(b) Conclude that the result in #4 is sharp. Show that the map
f : B
n+1
B
n+1
, f(x) = x,
has only one xed point, namely the origin, and in particular has no
xed points on the boundary.
6. Let v be a vector eld on X. Since X is compact, v generates
a one-parameter group of dieomorphisms
(5.6.33) f
t
: X X , < t < .
(a) Let

t
be the set of xed points of f
t
. Show that this set
contains the set of zeroes of v, i.e., the points, p X where v(p) = 0.
(b) Suppose that for some t
0
, f
t
0
is Lefshetz. Show that for all t, f
t
maps

t
0
into itself.
(c) Show that for |t| < , small, the points of

t
0
are xed points
of f
t
.
(d) Conclude that

t
0
is equal to the set of zeroes of v.
(e) In particular, conclude that for all t the points of

t
0
are xed
points of f
t
.
7. (a) Let V be a nite dimensional vector space and
F(t) : V V , < t <
a one-parameter group of linear maps of V onto itself. Let A =
dF
dt
(0).
Show that F(t) = exp tA. (See 2.1, exercise 7.)
(b) Show that if I F(t
0
) : V V is bijective for some t
0
, then
A : V V is bijective. Hint: Show that if Av = 0 for some v
V {0}, F(t)v = v.
5.6 The Lefshetz theorem 281
8. Let v be a vector eld on X and let (5.6.33) be the one-
parameter group of dieomorphisms generated by v. If v(p) = 0
then by part (a) of exercise 6, p is a xed point of f
t
for all t.
(a) Show that
(df
t
) : T
p
X T
p
X
is a one-parameter group of linear mappings of T
p
X onto itself.
(b) Conclude from #7 that there exists a linear map
(5.6.34) L
v
(p) : T
p
X T
p
X
with the property
(5.6.35) exp tL
v
(p) = (df
t
)
p
.
9. Suppose f
t
0
is a Lefshetz map for some t
0
. Let a = t
0
/N where
N is a positive integer. Show that f
a
is a Lefshetz map. Hints:
(a) Show that
f
t
0
= f
a
f
a
= f
N
a
(i.e., f
a
composed with itself N times).
(b) Show that if p is a xed point of f
a
, it is a xed point of f
t
0
.
(c) Conclude from exercise 6 that the xed points of f
a
are the
zeroes of v.
(d) Show that if p is a xed point of f
a
,
(df
t
0
)
p
= (df
a
)
N
p
.
(e) Conclude that if (df
a
)
p
v = v for some v T
p
X {0}, then
(df
t
0
)
p
v = v.
10. Show that for all t, L(f
t
) = (X). Hint: Exercise 2.
11. (The Hopf theorem.) A vector eld v on X is a Lefshetz vector
eld if for some t
0
, f
t
0
is a Lefshetz map.
(a) Show that if v is a Lefshetz vector eld then it has a nite
number of zeroes and for each zero, p, the linear map (5.6.34) is
bijective.
282 Chapter 5. Cohomology via forms
(b) For a zero, p, of v let
p
(v) = +1 if the map (5.6.34) is orientation-
preserving and 1 if its orientation-reversing. Show that
(X) =

v(p)=0

p
(v) .
Hint: Apply the Lefshetz theorem to f
a
, a = t
0
/N, N large.
12. (The trace of a linear mapping: a quick review.)
For A = [a
i,j
] an n n matrix dene
trace A =

a
i,i
.
(a) Show that if A and B are n n matrices
trace AB = trace BA.
(b) Show that if B is an invertible n n matrix
trace BAB
1
= trace A.
(c) Let V be and n-dimensional vector space and L : V V a
liner map. Fix a basis v
1
, . . . , v
n
of V and dene the trace of L to be
the trace of A where A is the dening matrix for L in this basis, i.e.,
Lv
i
=

a
j,i
v
j
.
Show that this is an intrinsic denition not depending on the basis
v
1
, . . . , v
n
.
5.7 The K unneth theorem
Let X be an n-dimensional manifold and Y an r-dimensional mani-
fold, both of these manifolds having nite topology. Let
: X Y X
be the projection map, (x, y) = x and
: X Y Y
5.7 The K unneth theorem 283
the projection map (x, y) y. Since X and Y have nite topology
their cohomology groups are nite dimensional vector spaces. For
0 k n let

k
i
, 1 i dimH
k
(X) ,
be a basis of H
k
(X) and for 0 r let

j
, 1 j dimH

(Y )
be a basis of H

(Y ). Then for k + = m the product,

k
i

j
, is
in H
m
(X Y ). The K unneth theorem asserts
Theorem 5.7.1. The product manifold, X Y , has nite topology
and hence the cohomology groups, H
m
(XY ) are nite dimensional.
Moreover, the products over k + = m

k
i

j
, 0 i dimH
k
(X) , 0 j dimH

(Y ) , (5.7.1)
are a basis for the vector space H
m
(X Y ).
The fact that X Y has nite topology is easy to verify. If U
i
,
i = 1, . . . , M, is a good cover of X and V
j
, j = 1, . . . , N, is a good
cover of Y the products of these open sets, U
i
U
j
, 1 i M , 1
j N is a good cover of XY : For every multi-index, I, U
I
is either
empty or dieomorphic to R
n
, and for every multi-index, J, V
J
is
either empty or dieomorphic to R
r
, hence for any product multi-
index (I, J) , U
I
V
J
is either empty or dieomorphic to R
n
R
r
.
The tricky part of the proof is verifying that the products, (5.7.1)
are a basis of H
m
(X Y ), and to do this it will be helpful to state
the theorem above in a form that avoids our choosing specied bases
for H
k
(X) and H

(Y ). To do so well need to generalize slightly the


notion of a bilinear pairing between two vector space.
Denition 5.7.2. Let V
1
, V
2
and W be nite dimensional vector
spaces. A map B : V
1
V
2
W is a bilinear map if it is linear in
each of its factors, i.e., for v
2
V
2
the map
v V
1
B(v
1
, v
2
)
is a linear map of V
1
into W and for v
1
V
1
so is the map
v V
2
B(v
1
, v) .
284 Chapter 5. Cohomology via forms
Its clear that if B
1
and B
2
are bilinear maps of V
1
V
2
into W
and
1
and
2
are real numbers the function

1
B
1
+
2
B
2
: V
1
V
2
W
is also a bilinear map of V
1
V
2
into W, so the set of all bilinear
maps of V
1
V
2
into W forms a vector space. In particular the set of
all bilinear maps of V
1
V
2
into R is a vector space, and since this
vector space will play an essential role in our intrinsic formulation of
the K unneth theorem, well give it a name. Well call it the tensor
product of V

1
and V

2
and denote it by V

1
V

2
. To explain where
this terminology comes from we note that if
1
and
2
are vectors in
V

1
and V

2
then one can dene a bilinear map
(5.7.2)
1

2
: V
1
V
2
R
by setting (
1

2
)(v
1
, v
2
) =
1
(v
1
)
2
(v
2
). In other words one has a
tensor product map:
(5.7.3) V

1
V

2
V

1
V

2
mapping (
1
,
2
) to
1

2
. We leave for you to check that this is a
bilinear map of V

1
V

2
into V

1
V

2
and to check as well
Proposition 5.7.3. If
1
i
, i = 1, . . . , m is a basis of V

1
and
2
j
,
j = 1, . . . , n is a basis of V

2
then
1
i

2
j
, 1 i m, 1 j n, is
a basis of V

1
V

2
.
Hint: If V
1
and V
2
are the same vector space you can nd a proof
of this in 1.3 and the proof is basically the same if theyre dierent
vector spaces.
Corollary 5.7.4. The dimension of V

1
V

2
is equal to the dimen-
sion of V

1
times the dimension of V

2
.
Well now perform some slightly devious maneuvers with duality
operations. First note that for any nite dimensional vector space,
V , the pairing
(5.7.4) V V

R, (v, ) (v)
is a non-singular bilinear pairing, so, as we explained in 5.4 it gives
rise to a bijective linear mapping
(5.7.5) V (V

)

.
5.7 The K unneth theorem 285
Next note that if
(5.7.6) L : V
1
V
2
W
is a bilinear mapping and : W R a linear mapping (i.e., an
element of W

), then the composition of and L is a bilinear mapping


L : V
1
V
2
R
and hence by denition an element of V

1
V

2
. Thus from the bilinear
mapping (5.7.6) we get a linear mapping
(5.7.7) L

: W

1
V

2
.
Well now dene a notion of tensor product for the vector spaces
V
1
and V
2
themselves.
Denition 5.7.5. The vector space, V
1
V
2
is the vector space dual
of V

1
V

2
, i.e., is the space
(5.7.8) V
1
V
2
= (V

1
V

2
)

.
One implication of (5.7.8) is that there is a natural bilinear map
(5.7.9) V
1
V
2
V
1
V
2
.
(In (5.7.3) replace V
i
by V

i
and note that by (5.7.5) (V

i
)

= V
i
.)
Another is the following:
Proposition 5.7.6. Let L be a bilinear map of V
1
V
2
into W.
Then there exists a unique linear map
(5.7.10) L
#
: V
1
V
2
W
with the property
(5.7.11) L
#
(v
1
v
2
) = L(v
1
, v
2
)
where v
1
v
2
is the image of (v
1
, v
2
) with respect to (5.7.9).
Proof. Let L
#
be the transpose of the map L

in (5.7.7) and note


that by (5.7.5) (W

= W.
286 Chapter 5. Cohomology via forms
Notice that by Proposition 5.7.6 the property (5.7.11) is the den-
ing property of L
#
, it uniquely determines this map. (This is in fact
the whole point of the tensor product construction. Its purpose is to
convert bilinear objects into linear objects.)
After this brief digression (into an area of mathematics which some
mathematicians unkindly refer to as abstract nonsense) lets come
back to our motive for this digression: an intrinsic formulation of the
K unneth theorem. As above let X and Y be manifolds of dimension
n and r, respectively, both having nite topology. For k + = m one
has a bilinear map
H
k
(X) H

(Y ) H
m
(X Y )
mapping (c
1
, c
2
) to

c
1

c
2
, and hence by Proposition 5.7.6 a linear
map
(5.7.12) H
k
(X) H

(Y ) H
m
(X Y ) .
Let
H
m
1
(X Y ) =

k+=m
H
k
(X) H

(Y ) .
The maps (5.7.12) can be combined into a single linear map
(5.7.13) H
m
1
(X Y ) H
m
(X Y )
and our intrinsic version of the K unneth theorem asserts
Theorem 5.7.7. The map (5.7.13) is bijective.
Here is a sketch of how to prove this. (Filling in the details will
be left as a series of exercises.) Let U be an open subset of X which
has nite topology and let
H
m
1
(U) =

k+=m
H
k
(U) H

(Y )
and
H
m
2
(U) = H
m
(U Y ) .
As weve just seen theres a K unneth map
: H
m
1
(U) H
m
2
(U) .
5.7 The K unneth theorem 287
Exercises.
1. Let U
1
and U
2
be open subsets of X, both having nite topology,
and let U = U
1
U
2
. Show that there is a long exact sequence:

H
m
1
(U) H
m
1
(U
1
)H
m
1
(U
2
) H
m
1
(U
1
U
2
)

H
m+1
1
(U)
Hint: Take the usual MayerVictoris sequence:

H
k
(U) H
k
(U
1
)H
k
(U
2
) H
k
(U
1
U
2
)

H
k+1
(U)
tensor each term in this sequence with H

(Y ) and sum over k + =


m.
2. Show that for H
2
there is a similar sequence. Hint: Apply
MayerVictoris to the open subsets U
1
Y and U
2
Y of M.
3. Show that the diagram below commutes. (This looks hard but
is actually very easy: just write down the denition of each arrow in
the language of forms.)

H
m
2
(U)H
m
2
(U
1
) H
m
2
(U
2
)H
m
2
(U
1
U
2
)

H
m+1
2
(U)
k

H
m
1
(U)H
m
1
(U
1
) H
m
1
(U
2
)H
m
1
(U
1
U
2
)

H
m+1
1
(U)
4. Conclude from Exercise 3 that if the K unneth map is bijective
for U
1
, U
2
and U
1
U
2
it is bijective for U.
5. Prove the K unneth theorem by induction on the number of
open sets in a good cover of X. To get the induction started, note
that
H
k
(X Y )

= H
k
(Y )
if X = R
n
. (See 5.3, exercise 11.)

Вам также может понравиться